Sei sulla pagina 1di 141

5

MATEMATIC
PROGRAMA COLAR PENTRU CLASELE DE EXCELEN
IX

ARGUMENT

Studiul matematicii prin clasele de excelen, urmrete n principal
crearea unui cadru organizat, n care elevii talentai la matematic, provenii din
diferite medii colare, s poat intra n contact, i n timp relativ scurt, s
formeze un grup performant. Aceti elevi, beneficiind de o pregtire pe msura
potenialului lor intelectual, vor contribui ulterior la formarea unei elite
romneti n domeniul matematicii.
Realizarea unei programe pentru clasele de excelen, precum i modul
n care se va lucra pe aceast program, constituie o noutate pentru
nvmntul romnesc. Din acest motiv elaborarea prezentei programe trebuie
neleas ca o etap necesar unui nceput de drum.
Un colectiv de cadre didactice din nvmntul preuniversitar i
universitar din CRTCP Cluj, cu experien n domeniul pregtirii elevilor
capabili de performane superioare, au format o echip care a realizat programa
i manualul care conine exerciii i probleme extrem de utile pentru
desvrirea pregtirii acestor elevi.
n selectarea coninuturilor programei s-a inut cont de tendinele actuale
n formularea subiectelor la concursurile i olimpiadele colare, dar i de
tradiiile colii romneti de matematic. Numeroasele cri i reviste adresate
vrfurilor au costituit o important surs bibliografic n tratarea temelor.
Temele propuse constituie o extindere fireasc a programei analitice obligatorii
de matematic i parcurgerea lor este necesar pentru abordarea unor probleme
mai dificile. Anumite teme vor fi tratate pe percursul mai multor ani de studiu
(evident cu o problematic corespunztoare) asigurndu-se astfel continuitatea
i coerena procesului de nvare. Mai trebuie precizat c la elaborarea
programei echipa a avut n vedere faptul c matematica nu este un produs finit,
ci un proces intelectual n care, pe suportul unor cunotine solide, primeaz
iniiativa personal. Astfel, aceast program ofer posibiliti autentice de
opiune pentru profesori i elevi.
Programa se adreseaz elevilor claselor V-IX i a fost conceput pentru
un numr de 2 ore/sptmn (n cele 30 de sptmni ale anului colar n care
se lucreaz cu clasele sau grupele de excelen). Obiectivele cadru ale
programei obligatorii de matematic au suferit cteva modificri. Obiectivul
cadru Dezvoltarea capacitilor de explorare/ investigare i de rezolvare de
probleme a fost nlocuit cu obiectivele cadru: Dezvoltarea capacitii de a
emite judeci de valoare pentru rezolvarea problemelor inventiv i euristic-
6
creative i Dezvoltarea capacitii de a face conexiuni cognitive n cadrul
disciplinei, la nivelul ariei curriculare i interarii. Raiunea acestei schimbri o
constituie tocmai specificul activitii intelectuale matematice la nivel de
performane superioare, prin focalizarea ateniei dinspre cmpul larg al
problemelor de matematic spre cele de tip inventiv i euristic-creative. O
atenie sporit se d i capacitii elevilor de a face conexiuni cognitive n
cadrul disciplinei, a ariei curriculare i interarii n scopul dobndirii unei
imagini de ansamblu a matematicii elementare ca parte a unui sistem aflat n
permanent evoluie i interaciune cu lumea nconjurtoare.
Programa este construit pe baza obiectivelor cadru ale predrii-nvrii
matematicii i are urmtoarele componente:
- obiective de referin i sugestii de activiti de nvare;
- coninuturi.




OBIECTIVE CADRU:

1. Cunoaterea i nelegerea conceptelor, a terminologiei i a
procedurilor de calcul
2. Dezvoltarea capacitii de a emite judeci de valoare pentru
rezolvarea problemelor inventiv i euristic-creative
3. Dezvoltarea capacitii de a face conexiuni cognitive n cadrul
disciplinei, la nivelul ariei curriculare i interarii
4. Dezvoltarea capacitii de a comunica utiliznd limbajul
matematic
5. Dezvoltarea interesului i a motivaiei pentru studiul i aplicarea
matematicii in contexte variate



7
OBIECTIVE DE REFERIN I EXEMPLE
DE ACTIVITI DE NVARE
1. Cunoaterea i nelegerea conceptelor, a terminologiei i a
procedurilor de calcul

Obiective de referin
La sfritul clasei a IX-a elevul va
fi capabil:
1.1. s recunoasc operaiile cu
mulimi i s opereze cu acestea



1.2. s opereze cu principii i metode
matematice











1.3. s opereze cu ecuaii i funcii
speciale



1.4. s utilizeze metode specifice de
rezolvare a problemelor de geometrie





1.5. s utilizeze metode geometrice n
rezolvarea problemelor de algebr

Exemple de activiti de nvare
Pe parcursul clasei a IX-a se
recomand urmtoarele activiti:
- exerciii de stabilire a unor identiti
n calculul cu mulimi
- demonstrarea principiului includerii
i excluderii

- exerciii de aplicare a principiului lui
Dirichlet n algebr i geometrie
- aplicaii ale principiului induciei
matematice n geometria vectorial,
plan i n spaiu
- demonstrarea unor inegaliti
utiliznd semnul funciei de gradul al
doilea
- exerciii de aplicare a principiului
includerii i excluderii
- rezolvarea unor inegaliti prin
metoda lui Sturm

- rezolvri de ecuaii diofantice
- exerciii cu funciile speciale: parte
ntreag, parte fracionar, minim,
maxim

- probleme legate de reele laticeale n
plan i spaiu
- rezolvarea problemelor de geometrie
utiliznd metoda vectorial
- probleme de concuren i colinia-
ritate

- inegaliti rezolvate geometric
- probleme de maxim i minim rezol-
vate geometric
8





1.6. s utilizeze metode matematice n
rezolvarea problemelor puse de alte
discipline

- ecuaii funcionale rezolvate geome-
tric
- sisteme de ecuaii rezolvate geome-
tric

- rezolvarea de probleme de geometrie
plan i n spaiu care i au originea n
fizic
- probleme pe tabla de ah, probleme
de partiionri i colorri


2. Dezvoltarea capacitii de a emite judeci de valoare pentru
rezolvarea problemelor inventiv i euristic - creative

Obiective de referin
La sfritul clasei a IX-a elevul va
fi capabil:
2.1. s analizeze, s elaboreze un plan
de rezolvare i s rezolve probleme
atipice i/ sau dificile din domeniile
studiate




2.2. s formuleze situaii-problem,
reciproce, generalizri pe baza proble-
melor atipice i/ sau dificile din
domeniile studiate











Exemple de activiti de nvare
Pe parcursul clasei a IX-a se
recomand urmtoarele activiti:
- nelegerea problemei
- elaborarea, n urma unei investigaii,
a unui plan de rezolvare
- obinerea soluiei prin aplicarea
planului
- verificarea rezultatului obinut i
analiza rezolvrii

- depistarea modului n care condiiile
iniiale ale problemei intervin n
soluionarea acesteia prin ntrebri
deschise de tipul: Ce s-ar ntmpla
dac din ipotez am elimina
condiia?, Ce s-ar ntmpla dac
n ipotez am modifica condiia cu
condiia?, Ce condiii de lucru
trebuie adugate pentru ca problema s
aib soluie unic?
- formularea unor ipoteze care s
conduc la reciproce, generalizri i
demonstrarea acestora


9
2.3. s identifice metode de lucru
valabile pentru clase de probleme

- identificarea unor algoritmi de re-
zolvare valabili pentru clase de
probleme
- analizarea eficienei metodelor
descoperite prin rezolvri de probleme
din aceeai sfer cognitiv


3. Dezvoltarea capacitii de a face conexiuni cognitive n cadrul
disciplinei, la nivelul ariei curriculare i interarii

Obiective de referin
La sfritul clasei a IX-a elevul va fi
capabil:
3.1. s utilizeze raionamente
inductive, deductive, prin analogie i
mixte n rezolvarea problemelor
atipice i/ sau dificile din domeniile
studiate





3.2. s-i nsueasc o gndire
reflexiv, independent, flexibil i
abstract specific matematicii
Exemple de activiti de nvare
Pe parcursul clasei a IX-a se
recomand urmtoarele activiti:
- exerciii care se rezolv fcnd
analogii ntre sisteme de elemente,
inclusiv utiliznd modelarea
- exerciii care se rezolv pornind de
la ipotez spre concluzie i invers, i
prin metode combinate
- extrapolarea soluiilor unei probleme
pentru rezolvarea altora din aceeai
sfer cognitiv

- probleme care necesit reflecii
asupra justeei mai multor soluii
ipotetice nainte de a aciona
- discuii care conduc la diferenierea
elementelor importante din enunul
problemei
- exerciii dintr-un domeniu al mate-
maticii care se rezolv cu metodele
specifice altui domeniu
- modelarea matematic a unor proble-
me sau enunuri matematice, din
cotidian sau puse de alte discipline




10
4. Dezvoltarea capacitii de a comunica utiliznd limbajul
matematic

Obiective de referin
La sfritul clasei a IX-a elevul va fi
capabil:
4.1. s-i nsueasc treptat exigenele
unei exprimri riguroase specifice
problemelor inventiv-creative din
domeniile studiate


4.2. s formuleze rezultate matematice
noi: generalizri, reciproce i metode
de rezolvare pentru clase de probleme
din domeniile studiate

Exemple de activiti de nvare
Pe parcursul clasei a IX-a se
recomand urmtoarele activiti:
- descrierea n scris, sau oral a
ncercrilor de rezolvare a problemelor
- redactarea unor demonstraii sau
rezolvri de probleme utiliznd
terminologia adecvat

- discuii pornind de la o problem n
scopul formulrii de noi rezultate
matematice
- s redacteze noi rezultate matematice
i s le demonstreze

5. Dezvoltarea interesului i a motivaiei pentru studiul i aplicarea
matematicii in contexte variate

Obiective de referin
La sfritul clasei a IX-a elevul va fi
capabil:
5.1. s argumenteze importana
metodelor matematice n rezolvarea
unor probleme cotidiene sau puse de
alte discipline

5.1. s manifeste disponibilitate, perse-
veren i gndire creativ pentru
rezolvarea problemelor atipice i/ sau
dificile


Exemple de activiti de nvare
Pe parcursul clasei a IX-a se
recomand urmtoarele activiti:
- identificarea rolului matematicii n
rezolvarea problemelor cotidiene sau
puse de alte discipline prin exemple


- rezolvarea de probleme persevernd
pe aceeai idee de lucru
- activiti n grup de rezolvare de
probleme care s permit fiecrui
participant s aib un rol activ
- activiti care s permit exprimarea
ideilor n scopul generrii de ipoteze
de lucru multiple i inedite
- utilizarea unor softuri educaionale
pentru nvarea matematicii
11
CONINUTURI



1. Mulimi, funcii, ecuaii
- Exerciii de teoria mulimilor
- Ecuaii diofantice. Metode elementare de rezolvare
- Funcii speciale

2. Cteva principii de rezolvare a problemelor de matematic
- Principiul lui Dirichlet
- Principiul induciei matematice n geometrie
- Principiul trinomului n stabilirea unor inegaliti
- Principiul includerii i excluderii

3. Metode de rezolvare a problemelor de matematic
- Metoda vectorial n rezolvarea problemelor de geometrie
- Metoda geometric n rezolvarea problemelor de algebr
- Metoda apropierii de extrem sau metoda variaiei pariale a lui
Sturm

4. Geometrie
- Reele laticeale n plan i spaiu
- Probleme de concuren i coliniaritate rezolvate vectorial
- Baricentre i centre de greutate
- Probleme de geometrie combinatoric






CLASA a IX-a


1. Exerciii de teoria mulimilor (Gh.Lobon)
1.1. Operaii cu mulimi. Proprieti

2. Principiul lui Dirichlet (E.Jecan)
2.1. Principiul lui Dirichlet n algebr
2.2. Principiul lui Dirichlet n geometrie

3. Reele laticeale n plan i spaiu (E.Jecan)
3.1. Noiuni teoretice
3.2. Probleme rezolvate

4. Ecuaii diofantice. Metode elementare de rezolvare a ecuaiilor
diofantice (E.Jecan)
4.1. Ecuaii diofantice de gradul nti
4.2. Metoda descompunerii
4.3. Metoda inegalitilor n rezolvarea ecuaiilor diofantice
4.4. Metoda aritmeticii modulare

5. Inducia matematic n geometrie (E.Jecan)

6. Metoda vectorial n geometrie (E.Jecan)
6.1. Consideratii teoretice
6.2. Probleme rezolvate

7. Probleme de concuren i coliniaritate rezolvate vectorial (E.Jecan)

8. Metoda geometric n rezolvarea problemelor de algebr (E.Jecan)

9. Funcii speciale (E.Jecan)
9.1. Definiii i proprieti ale funciilor speciale
9.2. Aplicaii ale funciilor speciale
9.3. Probleme propuse i soluii
9.4. Ecuaii funcionale

10. Principiul trinomului n stabilirea unor inegaliti (E.Jecan)
10.1. Consideratii teoretice

11. Principiul includerii i excluderii (Gh. Lobon)

14

12. Metoda apropierii de extrem, sau metoda variaiei pariale a lui Sturm
(E.Jecan)

13. Baricentre i centre de greutate (E.Jecan, V. Lupor)
13.1. Definiia matematic a baricentrului
13.2. Baricentrul unui sistem de puncte materiale n care apar mase
negative
13.3. Coordonate baricentrice n plan

14. Probleme de geometrie combinatoric (V. Pop)
14.1. Partiii ale mulimii numerelor naturale



Coordonator Vasile Pop
Viorel Lupor



15
1. Exerciii de teoria mulimilor
n cadrul acestei teme vom prezenta probleme din teoria mulimilor i
metode de rezolvare pline de imaginaie care s l stimuleaz pe elev. n teoria
naiv a mulimilor, ca i n unele teorii axiomatice ale mulimilor, ideea de
mulime este considerat ca o noiune primar, fapt pentru care nu se d o
definiie. Vom nelege mulimea ca pe o colecie de obiecte, fr a avea, ns,
pretenia c prin aceasta am enunat o definiie.

1.1. Operaii cu mulimi. Proprieti
1.1.1. Definiie: Date fiind dou mulimi, A i B, numim reuniunea lor
i o notm prin B A , mulimea care conine acele elemente i numai acelea
care aparin cel puin uneia dintre mulimile A sau B.
B A x dac i numai dac xA sau xB

1.1.2. Proprieti ale reuniunii a dou mulimi:

(1) Reuniunea a dou mulimi include fiecare dintre termenii reuniunii.
(2) Dac o mulime include ambii termeni ai reuniunii, atunci ea include
reuniunea.
(3) Reuniunea este comutativ A B B A =
(4) Reuniunea este asociativ ( ) ( ) C B A C B A =
(5) Reuniunea este idempotent A A A =
(6) Mulimea nul are rol de element neutru fa de reuniune
A A A = =
(7) Reuniunea este izoton n ambele argumente B A i D C rezult
D B C A

Demonstraie:
(7) Deoarece D B B (1) i B A din ipotez, din tranzitivitatea
incluziunii rezult c D B A . Analog se arat c D B C . Utilizm (1)
i rezult c D B C A .

1.1.3. Propoziie: B A dac i numai dac B B A =

Demonstraie: Din (1) rezult c B A A . Deci presupunnd
I B A = , deducem B A .
Reciproc, s presupunem B A . Deoarece B B , utilizm (2), rezult
c B B A . Incluziunea contrar B A B rezult din (1) deci
B B A .

16
1.1.4. Definiie: Fiind date dou mulimi A i B, numim intersecia lor
i o notm A B mulimea care conine acele elemente i numai acelea care
aparin att lui A ct i lui B.

x A B dac i numai dac x A sau x B

1.1.5. Proprieti ale interseciei:

(1) Intersecia a dou mulimi este inclus n fiecare dintre termenii interseciei.
(2) Dac o mulime este inclus n fiecare termen al interseciei, atunci ea este
inclus n intersecie.
(3) Intersecia este comutativ A B = B A.
(4) Intersecia este asociativ (A B ) C = A (B C)
(5) Intersecia este idempotent A A = A
(6) Mulimea vid are rol de anulator fa de intersecie = = A A
(7) Intersecia este izoton n ambele argumente A B i C D A C B
D
(8) Intersecia este distributiv fa de reuniune
a) ( ) ( ) ( ) C A B A C B A =
b) ( ) ( ) ( ) A C A B A C B =

1.1.6. Propoziie (proprietile de absorbie):
a) ( ) A B A A =
b) ( ) A B A A =

Demonstraie: Vom demonstra numai egalitatea a doua. Deoarece A B
A (din 1.1.5.(1)), aplicnd proprietatea: A B dac i numai dac A B = A,
rezult ( ) A A B A = , din care obinem egalitatea cerut folosind
comutativitatea.

1.1.7. Observaie: Tot sub numele de absorbie se ntlnesc i
identitile:
(A B) A = A (A B) A = A
A (B A) = A A (B A) = A
(B A) A = A (B A) A = A
care se obin din cele de mai sus prin comutativitate.

1.1.8. Propoziie: Reuniunea este distributiv fa de intersecie:
a) A (B C) = (A B) (A C)
b) (B C) A = (B A) (C A)

17
1.1.9. Definiie: Mulimile A i B se numesc disjuncte dac A B = .

1.1.10. Definiie: Fiind date dou mulimi A i B numim diferena lor i
o notm prin A \ B sau A B mulimea care conine acele elemente ale lui A
care nu se gsesc n B i numai acelea.

1.1.11. Propoziie: Au loc urmtoarele relaii:
(1) A \ B A
(2) A (B \ A) = A B
(3) (A \ B) \ C = (A \ C) \ B = A \ (B C)
(4) A \ (B C) = (A \ B) (A \ C)
(5) A \ (B C) = (A \ B) (A \ C)
(6) (A B) \ C = (A \ C) (B \ C)
(7) (A B) \ C = (A \ C) (B \ C)
(8) A \ = A, \ A =
(9) A \ A =

Demonstraie:
(2) Deoarece B \ A B , din proprietatea de izotonie a reuniunii rezult
c A(B \ A) A B. Reciproc, fie x A B, adic x A sau x B. Dac
x A, atunci x A (B \ A). Dac x A, atunci x B, deci x B \ A i,
prin urmare x A (B \ A), rezult c A B A (B \ A).
(4) x A \ (B C) dac i numai dac x A i x B C x A i (x B
sau x C) (x A i x B) i (x A i x C) x A \ B i x A \ C
dac i numai dac x (A \ B) (A \ C).

1.1.12. Definiie: Mulimea A B = (A \ B) (B \ A) se numete
diferena simetric a mulimilor A i B.

1.1.13. Lem: x A B (x A i x B) sau (x B i x A)

Demonstraie: Echivalena rezult din definiie.

1.1.14. Proprieti ale diferenei simetrice:
(1) Diferena simetric este comutativ A B = B A
(2) Diferena simetric este asociativ (A B) C = A (B C)
(3) Mulimea vid este elementul neutru fa de diferena simetric
A = A = A
(4) A A =


18
1.1.15. Propoziie: Intersecia este distributiv fa de diferena
simetric:
a) A (B C) = (A B) (A C)
b) (A B) C = (A C) (B C).

Demonstraie: Vom demonstra relaia de la a) deoarece relaia de la b)
rezult din prima folosind comutativitatea. Din definiie rezult c (A B) (A
C) = ( ) ( ) ( ) ( ) ( ) ( ) B A C A C A B A \ \ . Folosind 1.1.11. (5) obinem
(A B) (A C) =
( ) ( ) ( ) ( ) ( ) ( ) ( ) ( ) ( ) B C A A C A C B A A B A \ \ \ \
Deoarece (A B) A i (A C) A , rezult c (A B) \ A =
i (A C) \ A = , deci (A B) (A C) = ( ) ( ) ( ) ( ) B C A C B A \ \ .
Observm c (A B) \ C = A (B \ C) i rezult
(A B) (A C) = ( ) ( ) ( ) ( ) B C A C B A \ \
Folosind distributivitatea interseciei fa de reuniune, obinem
(A B) (A C) = A ( ) ( ) ( ) B C C B \ \ = A (B C).

1.1.16. Definiie: Dac B T, mulimea T \ B se numete
complementara lui B fa de T i se noteaz C
T
B
.

1.1.17. Notaie: n cazul particular n care mulimea T se subnelege,
vom nota prescurtat CB n loc de C
T
B
. n continuare vom lucra n P(T)
(mulimea prilor mulimii T) i vom adopta convenia de notaie.

1.1.18. Lem: Dac B P(T) i x T , atunci x CB dac i numai
dac x B.

Demonstraie: x CB x T \ B x T i x B x B.

1.1.19. Proprietile complementarei:

(1) Au loc formulele lui Morgan:
C (A B) = CA CB
C (A B) = CA CB
(2) Complementara este antiton: A B CA CB
(3) Complementara este involutiv CCA = A
(4) A CA = i A CA = T
(5) A B = i A B = T B = CA

19
Demonstraie:
(1) C(A B) = T \ (A B) = (T \ A) (T \ B) = CA CB
C (A B) = T \ (A B) = (T \ A) (T \ B) = CA CB
(2) A B A = A B CA = C (A B) = CA CB CA = CB.
(5) Fie x T . Dac x B din A B = deducem x A . Dac x B ,
din A B = T deducem x A. Deci x B x A, adic B = CA.

1.1.20. Definiie: Se numete produsul cartezian al mulimilor A i B i
se noteaz cu A B mulimea tuturor perechilor ordonate (a, b), unde a A i
b B.
A B = {(a, b) \ a A i b B}.

1.1.21. Proprietile produsului cartezian:
(1) Produsul cartezian este izoton: A B i C D A C B D.
(2) Produsul cartezian este distributiv fa de reuniune:
(A B) C = (A C) (B C)
C (A B) = (C A) (C B)
(3) Produsul cartezian este distributiv fa de intersecie:
(A B) C = (A C) (B C)
C (A B) = (C A) (C B)
(4) Produsul cartezian este distributiv fa de diferen:
(A \ B) C = (A C) \ (B C)
C (A \ B) = (C A) \ (C B)
(5) Produsul cartezian este distributiv fa de diferena simetric:
(A B) C = (A C) (B C)
C (A B) = (C A) (C B)

Demonstraie:
(2) Vom demonstra prima identitate. Deoarece A A B i B A B din
1.1.21. (1) deducem c A C (A B) C i B C (A B) C.
Aplicm 1.1.2.(2) i obinem (A C) (B C) (A B) C.
Reciproc , fie x (A B) C , adic x = (a, c) cu a A B i c C .
Dar a A B este echivalent cu a A sau a B. Dac a A, atunci x B
C deci x (A C) (B C).
(6) Demonstrm prima egalitate:
(A B) C = ( ) ( ) ( ) C A B B A \ \ = ( ) ( ) ( ) ( ) C A B C B A \ \ =
( ) ( ) ( ) ( ) ( ) ( ) C A C B C B C A \ \ = ( ) ( ) C B C A .


20
1.2. Probleme rezolvate (1.1)

R1.2.1.1 Fie mulimea M = {a + 1, a + 2, , a + 25}, unde a N. S se
determine cea mai mare valoare a lui a, astfel nct M s se poat mpri n trei
submulimi, dou cte dou disjuncte, cu proprietatea c suma elementelor din
fiecare submulime este cel mult
3
3 + s
, unde s este suma elementelor mulimii
M.
(A. Ghioca)

Soluie: Avem s = 25a + 325. Dac o astfel de mprire este posibil, atunci
una dintre cele trei submulimi va avea cel puin 9 elemente. Cum suma celor
mai mici 9 elemente din M este 9a + 45, este necesar ca 9a + 45
3
3 + s
, adic
a 96. Pentru a = 96 se poate construi urmtorul exemplu: M = X Y Z,
unde:
X = {97, 98, 99, , 105}, cu suma elementelor 909
Y = {108, 109, 110, 111, 112, 117, 120, 121}, cu suma elementelor 908
Z = {106, 107, 113, 114, 115, 116, 118, 119}, cu suma elementelor 908
ntruct pentru a = 96 avem
3
3 + s
= 909 +
3
1
, condiia cerut n enun este
ndeplinit. n concluzie, valoarea maxim a lui a este 96.


R1.2.1.2 Fie A o mulime de numere reale care verific:
a) 1 A
b) x A x
2
A
c) x
2
4x + 4 A x A
S se arate c 2000+ 2001 A
(L. Dragomir)

Soluie: Fie x A; din b) obinem x
2
A, deci ( ) | |
2
2
2 2 + x A, iar din c)
rezult c (x + 2) A. Din 1 A i (x + 2) A reiese prin inducie c A conine
toate numerele impare pozitive.
Astfel 2001 = ( ) | |
2
2 2 2001 + A, deci 2 2001 + A.
Avem 2 2001 + A i (x + 2) A, prin urmare 2000 2001 + A.

R1.2.1.3 S se determine mulimile nevide A R* cu proprietile:

21
(a) A are cel mult 5 elemente;
(b) x A
x
1
A i (1 x) A.
(M. ena)

Soluie: Fie x A. Din ipotez avem
x
1
, 1 x A; dar atunci i 1
x
1
=
=
x
x 1
A,
1 x
x
A,
x 1
1
A.
Aadar dac notm
)
`

=
1
,
1
, 1 ,
1
, /
x
x
x
x
x
x
x A x B avem c B A.
Deoarece A are cel mult 5 elemente, rezult c printre elementele lui B, cel
puin dou dintre ele sunt egale. Studiind toate posibilitile ca dou elemente
ale lui B s fie egale i innd seama c nu putem avea 0 A, rezult x = 2, x =
2
1
, i x = 1. Dar fiecare dintre aceste valori ale lui x genereaz o aceeai
mulime B i anume
)
`

= 2 ,
2
1
, 1 B . Rezult c exist o singur mulime A care
verific enunul, i anume
)
`

= = 2 ,
2
1
, 1 B A .
R1.2.1.4 Fie mulimile:
A = {(x, y) R R / x + y 1 = 0}
B = {(x, y) R R / x
3
+ y
3
2x
2
2y
2
xy + 2x + 2y 1 = 0} Determinai:
a) A \ B
b) B \ A.
(M. Ursu)
Soluie: Fie (x, y) B, atunci perechea (x, y) verific:
x
3
+ y
3
2x
2
2y
2
xy + 2x + 2y 1 = 0 (x + y 1)(x
2
+ y
2
xy x y + 1) =
0 (x + y 1)
( )
0
4
1 3
2
1
2
2
=
(
(


+
|
.
|

\
| +

y y
x x + y 1 = 0
sau (x, y) A{(1, 1)}, de unde deducem c B = A{(1, 1)}, ceea ce conduce
la rspunsul A \ B = i B \ A = {(1, 1)}.

R1.2.1.5 Se dau mulimile:
A = {x \ x = 3n 2, n N}
B = { x \ x = 1003 2m, m N}
C = { x \ x = 6p + 1, p Z, 0 p 166}.

22
S se arate c A B = C.
(N. Matei)

Soluie: Fie x A B i s artm c x C. ntr-adevr, dac x A B
atunci x A i x B , adic exist numerele naturale m i n astfel ca x = 3n
2 = =1003 2m, de unde rezult
2
1
502

=
n
n m ; m i n fiind numere
naturale trebuie ca
2
1 n
s fie ntreg:
2
1 n
= p.Rezult n = 2p + 1, M = 501
3p;2p + 1 1, 501 3p 1 , adic 0 p 166, x = 3n 2 = 3(2p + 1) 2 = 6p
+ 1, deci x C i A B C.
Reciproc. Fie x C i s artm c x A B. Dac x C, atunci exist p
ntreg, 0 p 166 , astfel ca x = 6p + 1 = 3(2p + 1) 2 = 3n 2 , unde am
notat 2p + 1 = = n. Rezult c x A. Avem i x = 6p + 1 = 1003 2(501 3p) =
1003 2m, unde am notat 501 3p = m i deci x B. Rezult x A B, deci
C A B i prin urmare A B = C.

R1.2.1.6 M este o submulime a mulimii {1, 2, 3, , 15} astfel nct
produsul oricror trei elemente distincte ale lui M nu este ptrat perfect. S se
determine numrul maxim de elemente ale lui M.
(O.I.M. Bulgaria)

Soluie: Observm c produsul celor trei elemente n fiecare dintre
mulimile {1, 4, 9}, {2,6,12}, {3, 5, 15}, {7, 8, 149} este un ptrat perfect. De
aici rezult c nici una dintre aceste mulimi nu este submulime a lui M. Cum
acestea sunt disjuncte , rezult c M 11. Dac 10 M , atunci M 10.
Presupunem c 10 M. Atunci una dintre mulimile {2, 5}, {6, 15}, {1, 4, 9},
{7, 8, 14} nu poate fi submulime a lui M. Dac {3, 12} M din nou avem M
10. Presupunem c {3, 12} M. Atunci nici una dintre mulimile {1} ,
{4}, {9}, {2, 6}, {5, 15}, {7, 8, 14} nu poate fi submulime a lui M i avem M
9. Urmeaz c n orice caz M 10. n final este uor de verificat c M = {1,
4, 5, 6, 7, 10, 11, 12, 13, 14} are proprietatea cerut. De aici valoarea maxim
pentru M 10.



23
2. Principiul lui Dirichlet

Matematica elementar ne ofer numeroase surse de inspiraie bazate pe
cunotine simple. Se pot realiza rezultate matematice valoroase folosind
mijloace la nivelul nvmntului gimnazial, ba chiar i a celui primar. Exist
probleme n matematica care nu cer aproape nici o cultur special, putnd fi
abordate cu mijloacele gndirii cotidiene, dar care prezint dificulti trecute cu
eforturi cerebrale remarcabile, cu imaginaie i ingeniozitate.
Un exemplu strlucit l constituie Principiul cutiei sau Principiul lui
Dirichlet, dup numele matematicianului care a adus rigoare i exigen n
analiza matematic. Faptul c acest principiu a fost introdus de ctre un
matematician cu profund sim al fineii n raionament, ne d de gndit asupra
posibilitii de modelare logico-matematic a unor fapte i evenimente
cotidiene, cu anse de realizri neateptate i pe arii largi de aplicabilitate.
Vom da n continuare cteva formulri ale principiului aplicat n algebr
i n geometrie, urmat de exemple.

2.1 Principiul lui Dirichlet n algebr

Una din formulrile principiului este urmtoarea:
Dac repartizm nk+1 obiecte n k cutii, atunci n cel puin o cutie vor
fi cel puin n+1 obiecte.
Observm c n acest enun este pus n eviden existena unui numr
minim de obiecte repartizate n aceeai cutie.
ntr-o alt formulare, folosim noiunea de partiie a unei mulimi:
Fie A o mulime nevid, iar
n
A A A ,..., ,
2 1
o partiie a lui A, adic
A A
n
i
i
=
=
U
1
, i =
j i
A A pentru j i . Dac avem n+1 elemente din A,
1 2 1
, ,..., ,
+ n n
a a a a , atunci exist o submulime
i
A a partiiei care s conin cel
puin dou elemente ale mulimii { } ,..., ,
1 2 1 + n
a a a .


2.2. Principiul lui Dirichlet n geometrie

Pentru aplicaiile geometrice dm trei formulri ale principiului.
A. Dac figurile
n
F F F ,..., ,
2 1
cu ariile respective
n
S S S ,..., ,
2 1
sunt incluse n
figura F cu aria S i ks S S S
n
> + + + ...
2 1
,
atunci 1 + k din figurile
n
F F F ,..., ,
2 1
au un punct comun.

24
B. Fie n plan o figura F de arie S i n figuri
i
F de arie n i S
i
, 1 , = . Dac

=
>
n
i
i
S S
1
atunci cele n figuri
i
F nu pot acoperi figura F
n
;
Dac cele n figuri
i
F acoper pe F, atunci S S
n
i
i

=1
.
Urmtoarea formulare a principiului lui Dirichlet are ca suport teoretic
noiunea de msur.
C. Fie X o mulime i fie ( ) X P mulimea prilor sale. O funcie
( ) [ ) , 0 : X P m se numete msur pe X dac are proprietile:
o
1

m()=0.
o
2

). ( ) ( ) ( ) ( B A m B m A m B A m + =
Fie ( ). ,..., ,
2 1
X P A A A
n
S notm ( )
U
L n i i
i i i k
k
k
A A A I

=
1
2 1
1
... , n k , 1 = .
2.2.1 Propoziie:

= =
=
n
k
k
n
k
k
I m A m
1 1
) ( ) ( .
Soluie: Pentru 2 = n ,obinem: ), ( ) ( ) ( ) (
2 1 2 1 2 1
A A m A A m A m A m + = +
deci proprietatea
o
2 din definiia msurii. S presupunem relaia adevrat
pentru n i s demonstrm pentru n+1. Fie ( )
U
L 1 1
'
1
2 1
...
+
=
n i i
i i i k
k
k
A A A I
Avem ) ( `
1 1 +
=
k n k k
I A I I

rezult:
( ) ( ) ( ) ( )
1 1 1
'
+ +
+ =
n k n k k k
A I m A I m I m I m , n k , 1 = ; dnd valori, avem:
( ) ( ) ( ) ( )
1 1 1 1
'
1
I A m A m I m I m
n n
+ =
+ +


( ) ( ) ( ) ( )
1 1 1
'
+ +
+ =
n n n n n n
A I m A I m I m I m


Adunnd egalitile obinem: ( ) ( ) ( ) ( )
1 1
1 1
'
+ +
= =
+ =
n n n
n
k
k
n
k
k
A I m A m I m I m
Dar
'
1 1 + +
=
n n n
I A I , deci

+
= =
+
+ =
1
1 1
1
). ( ) ( `) (
n
k
n
k
n k k
A m I m I m Folosind ipoteza
induciei, avem

=
+
=
=
n
k
n
k
k k
A m I m
1
1
1
) ( ) ' ( .
2.2.2. Corolar. Dac =
+1 p
I , atunci ( )

<
= =
U
n
k
k
n
k
k
A m p A m
1 1


25
Soluie: Avem evident
n p
I I I I
+
L L
1 2 1
, deci
). ( ... ) ( ) ( .... ) ( ) (
1 2 1 n p p
I m I m I m I m I m
+
Prin urmare =
+1 p
I ,
rezult c . 1 , 0 ) ( + = p k I m
k
Folosind relaia din propoziia anterioar,
obinem:
U
n
k
k
n
k
n
k
k k
A m p I m p I m A m
1
1
1 1
) ( ) ( ) ( ) (
= = =
= =

.
2.2.3. Corolar. Dac

= =
>
n
k
n
k
k k
A m p A m
1 1
) ( ) (
U
, exist
1 3 2 1
,..., , ,
+ p
i i i i cu
, ... 1
1 2 1
n i i i
p

+
astfel nct
+1 2 1 p
i i i
A A A L .
Observaii
1. Dac X este finit se poate lua = ) ( A m numrul elementelor mulimii A.
2. Dac , R X = , pentru b a < definim [ ] ( ) . , a b b a m =
3. Dac ,
3
R X = putem lua ca msur volumul.

Prezentm in continuare cteva aplicaii la variantele amintite.


Bibliografie

M. Ganga, Teme i probleme de matematica,Ed tehnica, Bucureti, 1991, pag. 95-108
A. Ghioca, N. Teodorescu, Culegere de probleme, Ed. SSM, 1997, Bucureti , pag 53-
68
D. Buneag, I. Maftei, Teme pentru cercurile i concursurile de matemetica ale
elevilor, Ed. Scrisul Romnesc, Craiova ,1983.
M. Ganga, Probleme elementare de matematica, Ed. Math-press , Ploieti, 2003, pag.
172-183
M. Cocuz, Culegere de probleme de matematica, Ed. Academiei, 1984, pag. 125
N. Teodorescu, A. Constantinescu, Probleme din gazeta matematica , Ed. Tehnica
1984


26
2.3. Probleme rezolvate (2.1)

R2.1.1. Se d o mulime M format din n numere ntregi. S se arate c exist
o submulime a lui M astfel nct suma elementelor sale s fie divizibil cu n.
(Gh. Szlsy)
Soluie: Fie M = {
n
a a a ,..., ,
2 1
}. Considerm sumele urmtoare:
, ... ,..., ,
2 1 2 1 2 1 1 n n
a a a S a a S a S + + + = + = = n total n numere. Dac unul
din numere, de exemplu
k
S , se divide la n, atunci submulimea cutat
este{
n
a a a ,..., ,
2 1
}. Dac nici unul din numerele
k
S ,

n k 1
,
nu se divide
la n, atunci resturile lor la mprirea prin n, sunt 1,2,,(n-1). Cum sunt n
numere
k
S , atunci cel puin dou vor da acelai rest la mprirea prin n. Fie
j i
S S , , i 1 , n j , j i < , dou astfel de numere. Atunci n S S
i j
M i deci
submulimea
j i i
a a a ,..., ,
2 1 + +
,este cea cutat.

R2.1.2 S se demonstreze c pentru orice numr impar a exist un numr
natural b, astfel nct 1 2
b
este divizibil cu a.
Soluie: Fie numerele , 1 2 ,..., 1 2 , 1 2
1 0

a
n numr de 1 + a . Prin
mprirea acestora la a se vor obine a resturi. Conform principiului lui
Dirichlet, exist (cel puin) dou numere ce dau acelai rest la mprirea cu a.
Fie aceste numere 1 2
k
i 1 2
p
, cu . p k < Atunci diferena lor se divide prin
a, deci ) 1 2 ( 2 ) 1 2 ( ) 1 2 ( =
k p k k p
a M . Cum a este impar, rezult
a
k m
M 1 2

i lum k m b = .

R2.1.3. Fie 1 2 + n numere reale mai mari dect 1 i mai mici ca
n
2 . S se
arate c exist trei ntre ele care pot fi lungimile laturilor unui triunghi.
Soluie: Puterile lui 2 mai mici dect
n
2 realizeaz o partiie a
intervalului ) 2 , 1 (
n
n n intervale, astfel: [ ) [ ) [ )
n n
2 , 2 ,..., 2 , 2 , 2 , 2 ), 2 , 1 (
1 3 2 2

Conform principiului cutiei, exist un interval din cele n, ce conine cel puin
trei din cele 1 2 + n numere. Fie a,b,c aceste numere situate n[ )
1
2 , 2
+ k k
,
. 1 0 n k Din
1 1
2 2 , 2 2
+ +
< <
k k k k
b a ,
1
2 2
+
<
k k
c , rezult de exemplu
1 1
2 2
+ +
< +
k k
b a , deci b a c + < i analoagele, prin urmare a, b, c pot fi
laturile unui triunghi.

R2.1.4. S se arate c oricare ar fi numerele ntregi a, b, c, d, numrul

27
)( )( )( )( )( )
2 2 2 2 2 2 2 2 2 2 2 2
( d c d b c b d a c a b a abcd E = este multiplu
de 7.
Soluie: Considerm mulimile } { . / 7 7 Z n k n k Z + = + Reuniunea lor
este mulimea Z, deci
U
6
0
7
=
+ =
k
k Z Z . Dac unul din numerele a, b, c, d se afl
n mulimea 7Z, rezult c . 7 M E Dac nici unul din numerele a, b, c, d nu este
n 7Z, atunci ele se afl n oricare din celelalte ase mulimi. Grupm aceste
mulimi astfel: { } } { } { . 4 7 , 3 7 , 5 7 , 2 7 , 6 7 , 1 7 + + + + + + Z Z Z Z Z Z
n modelul nostru acestea vor fi cutiile. Avem patru numere i trei cutii, prin
urmare cel puin dou numere vor fi in aceeai grup (cutie). Dac numerele au
aceeai forma (de exemplu 2 7 , 2 7 + + p k ) atunci diferena lor este divizibil cu
7, deci 7 M E . Dac numerele sunt de forme diferite
(de exemplu 6 7 , 1 7 + + p k )atunci suma lor se divide cu 7, prin urmare 7 M E .

Probleme rezolvate (2.2)

R2.3.1. S se arate c oricare ar fi 1000 de puncte ntr-un disc de raz 1 = r ,
exist un disc de raz
9
1
`= r ce acoper cel puin 11 puncte.
(Pop Vasile)
Soluie: Condiia ca 11 puncte s se afle intr-un disc de raz
9
1
este
echivalent cu intersecia nevid a cercurilor de raz
9
1
cu centrele n punctele
respective. S artm deci c 11 din discurile de raz
9
1
cu centrele in cele 1000
de puncte, au intersecia nevid. Reuniunea acestor discuri este inclus in discul
de raza ` r r + , deci ,
9
1
1
2
1000
1

+ <

=
U
i
i
D m iar ( )
2
1
9
1
1000

=

i
i
D m i avem:
( )

>
= =
U
1000
1
1000
1
10
i
i
i
i
D m D m . Folosind corolarul 2.2.2, exist 11 discuri cu
intersecia nevid.


28
R2.3.2. Pe suprafaa unui poligon de arie 13 se aeaz 10 poligoane de arie 6.
S se arate c exist 4 poligoane ce se intersecteaz dup o arie mai mare ca
70
1
. (Pop Vasile)
Soluie: Avem din ipotez , 13
10
1
=

=
U
i
i
P m ( ) 60
10
1
=

= i
i
P m i scriind relaia
din propoziia 2.2.1, avem:
U U
10
1
4 10 4 3
10
1
2
), ( 7 ) ( 3 ) ( ... ) ( ) ( ) ( ) ( 60
= =
+ + + + + + =
i
i
i
i
I m P m I m I m I m I m P m deci
, 21 ) ( 7
4
I m rezult . 3 ) (
4
I m Dar exist 210 intersecii de cte 4 poligoane.
Exist deci o intersecie
4 3 2 1 i i i i
P P P P cu
.
70
1
210
3
) (
4 3 2 1
=
i i i i
P P P P m

R2.3.3. n interiorul ptratului de latur 1 sunt aezate cteva cercuri avnd
suma lungimilor egal cu 10. S se arate c exist o dreapt care s intersecteze
cel puin 4 din aceste cercuri.

Soluie: Proiectm cercurile pe una din laturile ptratului. Proiecia
fiecrui cerc este un segment cu lungimea egal cu diametrul cercului respectiv.
Suma acestor segmente este

10
.Cum 1 , 3
10
>

conform principiului lui


Dirichlet, exist cel puin patru segmente care au un punct comun.
Perpendiculara ridicat n acest punct pe latura ptratului intersecteaz cel puin
patru cercuri.

R2.3.4. Un ptrat este secionat de zece drepte astfel nct fiecare dreapt
mparte ptratul n dou patrulatere ale cror arii sunt n raportul
3
2
. Artai c
cel puin trei dintre aceste drepte sunt concurente.

Soluie: Fiecare din cele zece drepte mparte ptratul n dou trapeze
avnd raportul ariilor
3
2
=
BEFC
AEFD
S
S
(fig2.1).

29
fig 2.1 fig 2.2
Dac [ ] MN este linia mijlocie a ptratului, atunci
NP l S MP l
AD FD AE
S
BEFC AEFD
= =
+
= ,
2
) (
, unde l este latura ptratului.
Avem:
3
2
= =
PN
MP
S
S
BEFC
AEFD
. Deci fiecare din cele 10 drepte mpart segmentul
[ ] MN sau [ ] PQ n acelai raport (fig 2.2). Pe segmentul [ ] MN punctele
1
P i
2
P mpart segmentul n raportul .
3
2
;
3
2
2
2
1
1
= =
M P
NP
N P
MP
La fel pe segmentul
[ ] PQ , punctele
3
P ,
4
P mpart segmentul n acelai raport. Oricare din cele 10
drepte trece prin unul din punctele
1
P ,
2
P ,
3
P ,
4
P , i conform principiului lui
Dirichlet rezult c printr-un punct trec cel puin trei drepte.

R2.3.5. S se arate c n orice poligon convex cu 21 de laturi exist doua
diagonale care formeaz ntre ele unghi mai mic de
0
1 .
Soluie: Poligonul are 189
2
18 21
=

diagonale. Considerm un punct O


n planul poligonului i ducem prin acest punct paralele la diagonalele
poligonului. Se formeaz n jurul lui O, 378 189 2 = unghiuri. Cum suma
msurilor unghiurilor n jurul unui punct este de
0
360 , deducem c cel puin
unul din cele 378 de unghiuri are msura mai mic dect
0
1 . Diagonalele
paralele cu laturile ce formeaz unghiul sunt cele cerute n enun.



30
R2.3.6. ntr-un cub de latur 1 se consider o linie frnt neintersectat de
lungime 300. S se arate c exist un plan paralel cu una din fee, care taie linia
in cel puin 101 puncte.
Soluie: Fie
n
L L L ,..., ,
2 1
segmentele liniei frnte i proieciile ortogonale
pe muchiile cubului . , ,
3 2 1
r r r
p p p Avem ( ) ( ) ( ) ( )
i r i r i r i
L p m L p m L p m L m
3 2 1
+ + <
de unde prin sumare, obinem:
( ) ( ) ( ) ( )

+ + < =
=
i r i r i r
n
i
i
L p m L p m L p m L m
3 2 1
1
300 . Deci exist o sum de
proiecii mai mare ca 100. Fie ( ) . 100
1
1
>

=
n
i
i r
L p m Dar , 1
1
1

=
U
n
i
i r
L p m deci
( )

>
= =
U
n
i
i r
n
i
i r
L p m L p m
1 1
1 1
100 i folosind corolarul 2.2.3. exist
101 2 1
,..., ,
i i i
L L L cu
= A L p L p L p
i r i r i r
101 1 2 1 1 1
... . Un plan paralel cu aceast direcie de
proiecie dus printr-un punct din A taie liniile
101 2 1
,..., ,
i i i
L L L (n 101 puncte).




31
3. Reele laticeale n plan i spaiu

Aceast tem ncearc s fac o deschidere spre teoria punctelor din
plan i spaiu de coordonate numere ntregi. Pentru a ptrunde n esena teoriei
sunt necesare cunotine elementare, majoritatea acumulate n gimnaziu.
Probleme de genul celor prezentate n continuare apar din ce n ce mai des la
concursurile de matematic, frumuseea lor constnd n simplitatea ideilor care
conduc la rezolvarea lor.
n continuare, vom introduce noiunile cu care operm n cadrul temei
amintite.

3.1. Noiuni teoretice
Fie n plan un sistem ortogonal xOy. Elementele mulimii
{(x,y) | x Z i y Z} se numesc puncte laticeale . Multimea
{(x,y) | x Z sau y Z } se numete reea laticeal n plan . Dreptele
0
x x = ,
0
y y = ,
0
x ,
0
y Z se numesc dreptele reelei , iar punctele laticeale se mai
numesc nodurile reelei. n mod asemntor se definesc reelele spaiale.
Fie n spaiu un sistem ortogonal. Mulimea {(x,y,z) | xZ sau yZ sau zZ} se
numete reea laticeal n spaiu.
Planele
0
x x = ,
0
y y = ,
0
z z = ,
0
x ,
0
y ,
0
z Z se numesc planele reelei.
Dreptele

=
=
0
0
y y
x x
,

=
=
0
0
z z
y y
,

=
=
0
0
x x
z z
, Z
0 0 0
, , z y x se numesc
dreptele reelei.
Punctele (
0 0 0
, , z y x ) cu Z
0 0 0
, , z y x se numesc nodurile reelei sau puncte
laticeale n spaiu.
Dreptele reelei situate n unul din planele reelei determin o retea laticeal
n plan , iar nodurile reelei situate pe o dreapt din reea, determin o diviziune
unitar a acestei drepte.
Vom prezenta i cteva probleme rezolvate, n care se aplic cunotine
elementare referitoare la distana dintre dou puncte n plan i spaiu, numere
raionale i iraionale precum i noiuni de teoria numerelor.



Bibliografie

D. Buneag, I. Maftei, Teme pentru cercurile i concursurile de
matematica ale elevilor, Ed. Scrisul Romnesc , Craiova 1983 pagina
29.

32
V. Pop , Reele ce trec prin elementele unei mulimi G.M. 5-6 , 2001,
pag. 193-198.
M. Pimsner, S. Popa, Probleme de geometrie elementar. E.D.P.1979.
A.M.Iaglom, I. M. Iaglom, Probleme neelementare tratate elementar. E.
T. Pag. 38-39
M. Ganga, Probleme elementare de matematic, Ed. Math press, Ploieti
2003.


33
3.2. Probleme rezolvate (3.1)

R3.2.1. S se demonstreze c oricare ar fi numrul natural n, exist n plan un
cerc care conine n interiorul su exact n puncte laticeale.
(Steinhana,Sierpinski)
Soluie. S demonstrm pentru nceput c nu exist dou puncte
laticeale M(a,b) i N(c,d) , M N care s aib aceeai distan la punctul P
|
.
|

\
|
3
1
, 3 ,
a,b,c,d Z . Presupunem c exist astfel de puncte, nseamn c PM=PN sau
( ) ( ) ( ) ( ) d b b a d c a c d c b a + + = |
.
|

\
|
+ = |
.
|

\
|
+
3
2
3 2
3
1
3
3
1
3
2 2 2 2
2
2
2
2
Dac a c ( ) Q 3 2 a c , contradicie, deci a c = , rezult
( ) ( ) . 0
3
2
0
3
2
2 2
= |
.
|

\
|
+ = b d b d b d b d Deoarece +
3
2
b d Z rezult
b d = , deci M=N. S finalizm raionamentul problemei.
Fie M
1
punctul laticeal cel mai apropiat de P, M
2
urmtorul .a.m.d. Din
demonstraia de mai sus , rezult M
1
M
2
M
n
i
PM
1
< PM
2
< <PM
n
< Cercul cu centrul n P i cu raz cuprins ntre
PM
n
i PM
1 + n
conine n interiorul su exact n puncte laticeale.
R3.2.2. S se demonstreze c dac pentru orice numr natural n exist n plan
un cerc cu centrul de coordonate (a,b), care conine n interiorul su exact n
puncte laticeale, atunci a i b nu pot fi simultan numere raionale.
Soluie. Fie punctul A(x,y) unde ,
q
p
x = = y
q
r
, cu p, q ,r , q 0 .
Atunci punctele laticeale B( ) p r , i C( ) p r, verific egalitatea AB=AC,
deoarece
.
2 2 2 2
|
|
.
|

\
|
+
|
|
.
|

\
|
=
|
|
.
|

\
|
+
|
|
.
|

\
|

q
r
p
q
p
r
q
r
p
q
p
r Aceasta nseamn c pentru
orice punct de coordonate raionale exist dou puncte laticeale distincte egal
deprtate de acel punct . Vom arta c dac cerina problemei este satisfcut
pentru un numr natural n, atunci ea nu are loc pentru n+1.
Dac a,bQ , atunci exist dou puncte laticeale distincte, egal deprtate
de punctul de coordonate ( ) b a, .
Dac cercul cu centrul ( ) b a, care trece prin aceste dou puncte conine
n interiorul su n puncte laticeale, atunci orice cerc concentric cu el i de raz

34
mai mare va conine n interiorul su cel puin n+2 puncte laticeale. Prin urmare
nu exist un cerc cu centrul ( ) b a, , care s conin exact n+1 puncte. Deci Q a
sau Q b .
R3.2.3.S se demonstreze c pentru orice numr natural n, exist n spaiul
3-dimensional o sfer care conine n interiorul su exact n puncte laticeale.

Soluie. Vom face un raionament asemntor cu cel de la problema
R-3.2.1. Fie dou puncte laticeale diferite ( )
1 1 1
, , z y x i ( )
2 2 2
, , z y x .
Vom demonstra c distanele de la aceste dou puncte la punctul de coordonate
( ) 5 , 3 , 2 sunt diferite.Presupunem c aceste distane sunt egale. Rezult
( ) ( ) ( ) ( ) ( ) ( ) + + = + +
2
2
2
2
2
2
2
1
2
1
2
1
5 3 2 5 3 2 z y x z y x
( ) ( ) + + + + = + + + + 5 3 2 2 5 3 2 2
2 2 2
2
2
2
2
2
2 1 1 1
2
1
2
1
2
1
z y x z y x z y x z y x
( ) ( ) ( ) ( ) | |. 5 3 2 2
2 1 2 1 2 1
2
2
2
2
2
2
2
1
2
1
2
1
z z y y x x z y x z y x + + = + + + + .
Cum
i i
y x , Z ,
___
3 , 1 = i , deducem ( ) ( ) ( ) . 0 5 3 2
2 1 2 1 2 1
= + + z z y y x x
Artm c , 0 5 3 2 = + + c b a a,b,c Z implic . 0 = = = c b a
Avem ( ) 6 2 3 2 5 3 2 5
2 2 2
ab b a c b a c + + = + = . Dac 0 ab rezult
ab
b a c
2
3 2 5
6
2 2 2

= , contradicie. Prin urmare 0 = a sau 0 = b i deducem c
0 = = = c b a . Revenind, obinem
2 1 2 1 2 1
, , z z y y x x = = = , contradicie cu
ipoteza problemei. Un raionament similar cu cel de la problema R-3.2.1.
conduce la rezolvarea problemei.

R3.2.4.S se demonstreze c dac un cerc, avnd raza de lungime un numr
natural, trece prin dou puncte laticeale situate la distana 1 unul fa de cellalt,
atunci pe circumferina cercului nu se mai afl nici un alt punct laticeal.


35
Solutie. Dac A i B sunt puncte laticeale situate la distana 1 ntre ele,
putem considera un sistem cartezian n care A( ) 0 , 0 i B( ) 0 , 1 . Dac r N este
raza cercului ce trece prin A i B, atunci centrul cercului este C |
.
|

\
|
0
;
2
1
y , cu
2 2
0
4
1
r y = + deci .
4
1
2
0
= r y Putem lua oricare din valori.
Fie deci C .
4
1
,
2
1
2
|
|
.
|

\
|
r Dac M( ) y x, este un punct laticeal prin care mai trece
cercul (C,r), atunci
0
4
1
2
4
1
2
1
2 2 2 2
2
2
2
= + =
|
|
.
|

\
|
+
|
.
|

\
|
r y y x x r r y x .Deoarece x,y
Z, rezult
2 2
1 4 k r = , ( )( ) 1 2 2 = + k r k r k Z , ecuaie ce nu are soluii n
numere ntregi, deoarece k r + 2 i k r 2 au aceeai paritate. Deci cercul nu
mai poate trece prin alt punct laticeal diferit de A i B.

R3.2.5.S se gseasc toate poligoanele regulate care pot avea toate vrfurile
n puncte laticeale.
Soluie. n mod evident ptratul verific cerinele problemei.
Vom arta c este singurul poligon regulat cu vrfurile n nodurile unei reele
laticeale.
Pentru nceput vom face demonstratia n cazul 6 , 5 , 3 = n .
Fie A, B, C trei vrfuri consecutive ale unui poligon regulat cu 3, 5 sau 6 laturi.
Atunci msura unghiului C B A

este ,
3


5
3
sau .
6
2
Presupunem c A,B,C
sunt

36
Fig. 3.1.

puncte laticeale (Fig. 3.1.). Atunci una din dreptele reelei (orizontal sau
vertical) trece prin interiorul unghiului ABC , fie aceasta d. Notm cu A i
C picioarele perpendicularelor din A i C pe d, iar cu i msurile
unghiurilor ' ABA i ' CBC , deci .
3
2
,
5
3
,
3
)
`

+


Avem tg Q =
'
'
BA
AA
, tg = Q
'
'
BC
CC
tg( ) Q

+
= +



tg tg
tg tg
1
.

Dar , 3
3
=

tg ,
5 2 10
1 5
5
3
+
+
=

tg 3
3
2
=

tg , contradicie cu
( ) Q + tg , prin urmare nu exist poligoane regulate cu 3, 5 sau 6 laturi, cu
vrfurile n puncte laticeale. Pentru 6 > n fie A
1
A
2
A
n
poligonul regulat
nscris n reea avnd latura cea mai mic dintre toate poligoanele cu aceast
proprietate i fie O centrul su.
Fie O un nod al reelei. Considerm vectorii
i
B O' =
1 + i i
A A pentru
1 ,..., 2 , 1 = n i i
n
B O' =
1
A A
n
, rezult B
1
B
2
B
n
este un poligon regulat cu
vrfurile puncte laticeale .

37
Avem OB
i
B
1 + i
~ OA
i
A
1 + i
i
3
) (
1

<
+ i i
OA A m
1 1
'
+ +
= <
i i i i i
A A B O B B ,
fapt ce contrazice minimalitatea lui A
i
A
1 + i
.


R3.2.6. Fie P un poligon convex n plan care conine n puncte laticeale n
interior, k puncte laticeale pe laturi sau vrfuri, iar vrfurile sale sunt puncte
laticeale.
S se demonstreze c Aria (P) 1
2
+ =
k
n . (Teorema lui Pick)
Soluie. S demonstrm formula pentru cazul 3 , 0 = = k n .
Acest caz corespunde situaiei n care P este un triunghi cu vrfurile n nodurile
reelei i care nu mai conine alte noduri pe laturi sau n interior. Artm c aria
triunghiului este .
2
1
Fig.3.2.6. a)



38
Fig.3.2.6. b)
Fie ABC un astfel de triunghi. El poate s nu aib nici o latur coninut n
dreptele reelei (Fig.3.2. a), sau poate avea o latur coninut n dreptele
reelei(Fig.3.2.b). n ambele situaii, simetricul lui B fa de mijlocul lui [AC]
este un punct laticeal, pe care-l notm cu D i ABCD este paralelogram cu
S
ABCD ABC
S = 2 .
Dac translatm paralelogramul de-a lungul axelor reelei, obinem o reea de
paralelograme care acoper planul, iar toate vrfurile acestei reele de
paralelograme vor fi dispuse n nodurile reelei de ptrate, n interiorul i pe
laturile reelei de paralelograme nefiind situat nici un nod al reelei de ptrate.
Vom arta c aria unui astfel de paralelogram este 1.
n figura 3.2.a), S =
ABCD
S 1 =
DECF
, iar n figura3.2.b), S =
ABCD
S . 1 =
BEDF
Deci
S .
2
1
=
ABC
Pentru demonstraia n cazul general, s descompunem poligonul P n
triunghiuri cu vrfurile n puncte laticeale i care nu mai conin puncte laticeale
pe laturi sau n interior. Vom calcula numrul m de triunghiuri exprimnd n
dou moduri suma unghiurilor lor. Pe de o parte suma unghiurilor este
180
0
m , pe de alt parte ea este egal cu suma unghiurilor poligonului i a
unghiurilor din jurul punctelor interioare, deci ea este 180
0
( ) n k +
0
360 2 .
Rezult c 2 2 + = k n m i cum aria unui triunghi este
2
1
, cele m triunghiuri
au aria
2
m
, deci Aria(P) 1
2
+ =
k
n .

39
R3.2.7. n fiecare ptrat al unei reele laticeale este scris cte un numr
real. Se consider dou figuri n plan, fiecare fiind format dintr-un numr finit
de ptrate laticeale. Figurile pot fi translatate n orice poziie din plan. Se tie c
pentru orice translaie a primei figuri, suma numerelor acoperite de figura
translatat este pozitiv. S se arate c exist o translaie a celeilalte figuri,
astfel ca suma numerelor acoperite s fie pozitiv.

Soluie. Notm cu A figura format din ptratele A
1
,A
2
, ,A
n
i cu B
figura format cu ptratele B
1
, B
2
, , B
m
.
Se observ c dac translatm ptratul A
i
cu vectorul de poziie al ptratului
B
j
, se obine ptratul P
ij
, acelai cu translatatul ptratului B
j
cu vectorul de
poziie al ptratului A
i
. (centrul ptratului P
ij
este ( )
j B Ai
r r + . Pentru orice
translaie a figurii A dup orice ptrat B
j
, obinem figura format din ptrate
P
j 1
, P
j 2
, , P
nj
i notm C
ij
numrul scris n ptratul P
ij
. Avem conform
ipotezei

=
>
n
i
ij
C
1
0, pentru orice m j , 1 = , care prin adunare dau

= =
|
.
|

\
|
m
j
n
i
ij
C
1 1
>0 sau echivalent

= =
|
|
.
|

\
|
n
i
m
j
ij
C
1 1
>0, deci cel puin o sum de forma

=
n
j
ij
C
1
este pozitiv. Ea reprezint suma numerelor din ptratele P
1
0
i
, P
2
0
i
, ,
P
m i
0
obinut translatnd figura B dup vectorul de poziie al ptratului A
0
i
.


R3.2.8. S se arate c din orice nod al unei reele laticeale se poate ajunge
n orice alt nod prin sriturile unui cal de ah.

Soluie. Dintr-o poziie dat ( ) Z ZX y x , se pot face opt tipuri de
srituri, (se poate ajunge n alte opt noduri) dup schema ( ) ( ) 2 , 1 , y x y x ,
( ) ( ) 1 , 2 , y x y x . Sriturile contrare ( ) 2 , 1 + + y x i ( ) 2 , 1 y x se
compenseaz , deci este suficient s ne rezumm la umrtoarele patru tipuri de
srituri.
( ) ( ) 2 , 1 ,
1
+ + y x y x
S
, ( ) ( ) 2 , 1 ,
2
+ y x y x
S

( ) ( ) 1 , 2 ,
3
+ + y x y x
S
, ( ) ( ) 1 , 2 ,
4
+ y x y x
S


40
Dac se fac
1
a srituri de tipul S
1
,
2
a de tipul S
2
,
3
a de tipul S
3
i
4
a de tipul
S
4
, din nodul ( ) y x, ajungem n ( ) ' , ' y x unde:

+ + + =
+ + + =
4 3 2 1
4 3 2 1
2 2 '
2 2 '
a a a a y y
a a a a x x


Trebuie artat c pentru orice x, y, x, y , exist Z
4 3 2 1
, , , a a a a astfel ca
s fie verificate relaiile de mai sus.
Avem
( )
( )

+ =
+ =
4 2 2 2
4 2 1 1
5 4
3
1
4 5
3
1
a a z a
a a z a
unde
( ) ( )
( ) ( )

+ =
=
y y x x z
y y x x z
' ' 2
' 2 '
2
1

Numerele Z
3 2
, a a , trebuie astfel alese, nct parantezele s se divid cu 3. Se
observ c
2 1
z z (mod 3),
2 2
5 a a ,
4 4
4 a a ,
2 2
4 a a ,
4 4
5 a a =
(mod 3) ceea ce evident putem realiza lund de exemplu
1 2
z a , 0
4
a , (fr
a folosi srituri de tipul S
4
).

41
4. Ecuaii diofantice. Metode elementare de rezolvare a ecuaiilor
diofantice


Titlul temei vine de la Diofant, numit i ,,tatl algebrei care i-a
desfurat activitatea n oraul Alexandria care era n antichitate centrul
universitar al nvmntul matematic. El este cunoscut pentru cartea sa
,,Aritmetica, lucrare n care trateaz ecuaiile algebrice i teoria numerelor.
Cartea este o colecie de 150 de probleme cu soluii aproximative al unor ecuaii
determinate de grad cel mult trei i coninnd i ecuaii nedeterminate.
Vom numi n continuare ecuaie diofantic, o ecuaie de forma
0 ) ,..., , (
2 1
=
n
x x x f (*) unde f este o funcie de n variabile i n2. Dac f este
funcie polinomial cu coeficieni ntregi, (*) se numete ecuaie diofantic
algebric. Un n-uplu ) ,..., , (
0 0
2
0
1 n
x x x
n
care satisface (*) se numete soluie a
ecuaiei (*) . O ecuaie care are una sau mai multe soluii se numete solvabil.
Relativ la ecuaiile diofantice se pun urmtoarele trei probleme.
1. Este ecuaia solvabil ?
2. n caz de solvabilitate, este numrul soluiilor finit sau infinit ?
3. n caz de solvabilitate, s se determine toate soluiile ecuaiei.

Dintre ecuaiile diofantice clasice vom prezenta ecuaiile
diofantice liniare.

4.1. Ecuaiile diofantice de gradul nti

O ecuaie diofantic de gradul nti cu dou necunoscute este de forma
ax + by = c (1) unde a,b,c Z, ab 0. Perechea ) , (
0 0
y x
2
care verific (1) o
numim soluie particular.

4.1.1. Teorem: Condiia necesar i suficient ca ecuaia (1) s admit
soluie este ca d/c, unde d=(a,b).
Demonstraie. Dac exist ) , (
0 0
y x care verific (1), atunci
c by ax = +
0 0
i prin urmare d/c, deci condiia este necesar. Dac d/c, exist
1
c R, astfel nct c=d
1
c . Deoarece d=(a,b), exist u,vZ, astfel nct au + bv
= d (2). nmulind ambii membrii ai egalitii (2) cu
1
c , obinem a(
1
c u) +
b(
1
c v) = c, prin urmare (
1
c u,
1
c v) este o soluie particular a ecuaiei (1).

42
4.1.2. Teorem: Dac ecuaia diofantic ax + by = c are soluia
particular ) , (
0 0
y x i d=(a,b), soluia general e ecuaiei este dat de:
x = x
0
+ t
d
b
, y = y
0
t
d
a
, t
Demonstraie. Dac ) , (
0 0
y x este o soluie particular, atunci a
0
x +
b
0
y = c (3). Pentru o pereche arbitrar (x,y)Z
2
, avem ax + by = c (4).
Scznd cele dou relaii, obinem: ) ( ) (
0 0
y y b x x a = (5) Cum a=a
1
d,
b=b
1
d, 1 ) , (
1 1
= b a , (5) devine: ) ( ) (
0 1 0 1
y y b x x a = (6)
Rezult c ) (
0 1 1
x x a b i cum 1 ) , (
1 1
= b a , deducem ca
0 1
x x b , prin urmare
exist t, astfel nct t b x x
1 0
= . nlocuind n (6) obinem t a y y
1 0
= .
Deci t b x x
1 0
+ = i t a y y
1 0
= . Reciproc, dac ) , (
0 0
y x este soluie particular
(verific (3)) i t b x x
1 0
+ = , t a y y
1 0
= , t, atunci (6) implic (5). n fine
din (5) i (3) rezult (4), deci ) , ( y x este o soluie a ecuaiei.

4.1.3. Corolar: Fie
2 1
, a a numere ntregi prime ntre ele. Dac
) , (
0 0
y x este o soluie a ecuaiei b y a x a = +
2 1

atunci toate soluiile ei sunt date de

=
+ =
t a y y
t a x x
1 0
2 0
, unde t

4.1.4. Exemple:
a) S se determine cel mai mare divizor comun al numerelor ntregi 1215
i 2755 i s se exprime acestea ca o combinaie liniar a celor dou numere.
b) S se rezolve n ecuaia 1215x 2755y = 560.

Soluie
a) d = (1215,-2755) = (2755,1215). Aplicnd algoritmul lui Euclid,
avem: 2755 = 12152 + 325
1215 = 3253 + 240
325 = 2401 + 85
240 = 852 + 70
85 = 701 + 15
70 = 154 + 10
15 = 101 + 5
43
10 = 52 , deci d=5
Pentru a afla u,v astfel nct v u d ) 2755 ( 1215 + = folosim algoritmul de
mai sus.
Avem: 325 = 12755 + (2)1215
240 = (-3)2755 + 71215
85 = 325 - 2401 = 42755 - 91215
70 = 240 - 852 = (-11)2755 + 251215
15 = 152755 - 341215
10 = -712755 + 1611215
5 = 862755 - 1951215
Deci 5 = 1215(-195) + (-2755)(-86).
b) Avem 560 5 , deci ecuaia are soluii. Cum 560 = 5112, soluia
particular este
0
x = 112 195 ,
0
y = 112 86 , iar soluia general
t x x + = 551
0
, t y y + = 243
0
, t .
Considerm cazul ecuaiilor diofantice de gradul nti cu n necunoscute,
1 n :
b x a x a x a
n n
= + +
1 2 2 1 1
... (7) , unde
n
a a a ,... ,
2 1
, b sunt numere ntregi
fixate i
n
a a a ,... ,
2 1
sunt numere nenule. Principalul rezultat referitor la aceast
ecuaie, care se mai numete ecuaie diofantic liniar, este :

4.1.5.Teorem Condiia necesar i suficient ca ecuaia (7) s
admit soluii este ca b d , unde d = (
n
a a a ,... ,
2 1
).
Demonstraie. O soluie a ecuaiei (7) este un sistem ordonat
) ,..., , (
0 0
2
0
1 n
x x x pentru care are loc egalitatea b x a x a x a
n n
= + + + ) ... (
0 0
2 2
0
1 1
(8)
Presupunem c b nu este divizibil cu d, atunci egalitatea (8) ne este posibil,
deoarece partea stng din (8) este divizibil cu d, iar partea dreapt nu. Dac
b d , adic
1
b d b = , atunci exist 1
1
...
1 1
2 1
= + + +
n
x x x
, astfel nct
d u a u a u a
n n
= + + ...
2 2 1 1
. nmulind ambii membri cu
1
b , obinem:
b b u a b u a b u a
n n
= + + + ) ( ... ) ( ) (
1 1 2 2 1 1 1
i am pus n eviden o soluie
particular a ecuaiei (7), . ,..., ,
1
0
1 2
0
2 1 1
0
1
b u x b u x b u x
n n
= = =
Pentru aflarea mulimii soluiilor ecuaiei, avem urmtoarea:

4.1.6. Teorem: Rezolvarea unei ecuaii diofantice de gradul nti cu n
necunoscute se reduce la rezolvarea unei ecuaii diofantice de gradul nti cu
44
dou necunoscute i a unei ecuaii diofantice de gradul nti cu n-1 necunoscute.
Soluia general depinde de n-1 parametri ntregi.
Demonstraie Fie ecuaia (7) care satisface condiia b d , d =
(
n
a a a ,... ,
2 1
). Notm ),..., , ( ),..., , ( ), , (
1 3 2 3 2 1 2 k k k
a d d a d d a d d

= = =
) , (
1 n n n
a d d

= . tim c d = d
n
. Notm y d x a x a x a
n n n 1 1 1 2 2 1 1
...

= + + + .
Ecuaia (7) se scrie b x a y d
n n n
= +
1
(9). Deoarece (
n n
a d ,
1
) = d i b d ,
ecuaia (9) are soluii i dac (
0
0
,
n
x y ) este o soluie particular a ecuaiei, atunci
soluia general este :
t
d
d
x x t
d
a
y y
n n 1
0 0
,

= + = , t
Pentru fiecare valoare fixat a lui t, gsim o valoare pentru
n
x i una pentru y.
Pentru a gsi o soluie a ecuaiei (7), trebuie s gsim, pentru t fixat,
1 2 1
,... ,
n
x x x astfel nct:
) ( ...
0 1 1 1 2 2 1 1
t
d
a
y d x a x a x a
n
n n n
+ = + +

(10)
Deoarece
1 1 2 1
) ,..., , (

=
n n
d a a a i ) (
0 1 1
t
d
a
y d d
n
n n
+

, rezult c ecuaia (10)
are soluii.
Am vzut n 4.1.2. c dac ecuaia diofantic de gradul nti cu dou
necunoscute are soluii, soluia general depinde de un parametru. Presupunem
c soluia general a ecuaiei cu n-1 necunoscute depinde de n-2 parametri:
2 2 1
,..., ,
n
t t t . Rezult c soluia general a ecuaiei (7) depinde de n-1 parametri:
2 2 1
,..., ,
n
t t t i t.
Demonstraia dat ne propune i o metod de determinare a soluiei
generale a ecuaiei (7) pentru 2 n .
Determinm
n
x n funcie de un parametru
1 n
t .
1
1 0

=
n
n
n n
t
d
d
x x ,
1 n
t
Din (10) gsim
1 n
x care depinde de un nou parametru
2 n
t (i de
1 n
t ),
.a.m.d., pn ajungem la
) ,... , (
1 3 2 2 2 1 1
= +
n
t t t A x a x a , de unde gsim

1
x i
2
x depinznd de nc un parametru
1
t .

4.1.7. Exemplu S se rezolve n ecuaia diofantic
14 2 10 6 4
4 3 2 1
= + + x x x x .
45
Soluie 2 ) 2 , 10 , 6 , 4 ( = = d i 14 2 , deci ecuaia are soluii,
2 ) 10 , 6 , 4 (
3
= = d . Notm
1 2 2 1
2 10 6 4 y x x x = + i ecuaia dat devine
14 2 2
4 1
= + x y sau 7
4 1
= + x y .
Notm
3 1
t y = i rezult
3 4
7 t x = , t. Obinem
3 3 2 1
2 10 6 4 t x x x = + sau
3 3 2 1
5 3 2 t x x x = + . Notm
2 2 1
3 2 y x x = i obinem
3 3 2
5 t x y = + . Notm
2 3
t x = , deci
2 3 2
5t t y = . Din
2 3 2 1
5 3 2 t t x x = ,
2 3 2 3 2 3
5 ) 5 ( 3 ) 10 2 ( 2 t t t t t t = , obinem
1 2 3 1
3 10 2 t t t x + = ,
1 2 3 2
2 5 t t t x + = .
Soluia general este:

3 2 1 1
2 10 3 t t t x + =

3 2 1 2
5 2 t t t x + =

2 3
t x =

3 4
7 t x = ,
3 2 1
, , t t t .


n continuare vom prezenta principalele metode elementare de studiu a
ecuaiilor diofantice.

4.2. Metoda descompunerii
Metoda const n scrierea ecuaiei ) ,..., , (
2 1 n
x x x f =0 sub forma
) ,..., , (
2 1 1 n
x x x f ) ,..., , (
2 1 2 n
x x x f a x x x f
n k
= ) ,..., , (
2 1
, a. Folosind
descompunerea n factori primi a lui a, obinem un numr finit de descompuneri
n k factori ntregi
k
a a a ,..., ,
2 1
. Din fiecare descompunere obinem un sistem.

i n i
a x x x f = ) ,..., , (
2 1
, i{ } k ,..., 2 , 1 .
Rezolvarea acestor sisteme ne conduce la mulimea de soluii. Vom
exemplifica metoda prin cteva exemple:



4.3. Metoda inegalitilor n rezolvarea ecuaiilor diofantice

Probleme rezolvate

4.3.1. S se determine toate perechile (x, y) de numere ntregi care verific
ecuaia
2 3 3
) ( y x y x + = + .

46
Soluie Observm pentru nceput c orice pereche de forma (
0 0
, x x ), cu
0
x este soluie a ecuaiei. Dac 0 + y x ecuaia devine
y x y xy x + = +
2 2
, care este echivalent cu 2 ) 1 ( ) 1 ( ) (
2 2 2
= + + y x y x .
Din aceast egalitate n numere ntregi deducem 1 ) 1 (
2
x i 1 ) 1 (
2
y , de
unde
x [0,2], y [0,2]. Obinem soluiile (0,1), (1,0), (1,2), (2,1), (2,2).

R4.3.2. Determinai toate cvadruplele de numere naturale nenule (x, y, z, w)
pentru care
2 2 2 2
) 1 ( 2 ) 1 ( 2 2 w z y z x xy z y x = + + + + + +
(Titu Andreescu)

Soluie Avem
2 2 2 2 2
1 2 ) 1 ( 2 ) 1 ( 2 2 ) 1 ( w z z y z x xy z y x z y x < + + + + + + = + + +
i
2 2 2 2
1 2 ) 1 ( 2 ) 1 ( 2 2 ) 1 ( w z z y z x xy z y x z y x > + + + + + + + + + = + + +
Din
2 2 2
) 1 ( ) 1 ( + + + < < + + z y x w z y x rezult c
2 2
) ( z y x w + + = , deci
2 2 2 2
) ( ) 1 ( 2 ) 1 ( 2 2 z y x z y z x xy z y x + + = + + + + + + , de unde obinem x = y.
nlocuind n ecuaie, avem
2 2 2
4 4 w xz z x = + + , sau
2 2
) 2 ( w z x = + . Soluiile
ecuaiei sunt (m,m,n,2m+n), m, nN { } 0 .

R4.3.3. Determinai toate tripletele (x, y, z) de numere naturale nenule astfel
nct
2 )
1
1 )(
1
1 )(
1
1 ( = + + +
z y x
.
(Olimpiad Anglia)

Soluie Fr a restrnge generalitatea problemei putem presupune c
z y x . Obinem
3
)
1
1 ( )
1
1 )(
1
1 )(
1
1 (
z z y x
+ + + + , adic
3
)
1
1 ( 2
z
+ , care
implic 3 z . Dac
z = 1, atunci 1 )
1
1 )(
1
1 ( = + +
y x
, imposibil pentru x, y N { } 0 . Dac z = 2,
ajungem la
3
4
)
1
1 )(
1
1 ( = + +
y x
. Cum y x , obinem
3
4
)
1
1 (
2
+
y
, deci y<7.
47
Cum 1
1
1 > +
x
egalitatea
3
4
)
1
1 )(
1
1 ( = + +
y x
ne conduce la
3
4 1
1 < +
y
, sau y > 3,
deci y{4,5,6}. nlocuind, obinem soluiile (7,6,4), (9,5,2), (15,4,2).
Dac z = 3, atunci
2
3
)
1
1 )(
1
1 ( = + +
y x
. Deoarece y x , avem
2
3
)
1
1 (
2
+
y
., de
unde
y < 5. Avem i 3 = z y , care ne conduce la soluiile (8,3,3) i (5,4,3). Soluiile
ecuaiei sunt toate permutrile circulare ale tripletelor (7,6,2), (9,5,2), (15,4,2),
(8,3,3) i (5,4,3).



4.4. Metoda aritmeticii modulare

Consideraiile simple de aritmetic modular se dovedesc foarte utile n
demonstraia faptului c anumite ecuaii nu au soluii sau la reducerea
posibilitilor de alegere a soluiilor acestora.
Amintim cteva noiuni generale cu privire la congruene.
Definiia 1. Dou numere ntregi a i b se numesc congruente modulo
m, unde m, dac ) ( b a m . Se scrie ) (mod m b a i se citete : a congruent
cu b modulo m .
Definiia 2. Dou numere ntregi a i b se numesc congruente modulo
m, unde m { } 0 , dac prin mprire la m dau acelai rest, iar pentru m = 0
dac sunt egale.
Cteva proprieti ale relaiei de congruen:
1. Relaia de congruen este o relaie de echivalen, adic este reflexiv,
simetric i tranzitiv.
2. Dou relaii de congruen se pot aduna, scdea sau nmuli membru cu
membru.
3. Ambii membri ai unei relaii de congruen pot fi ridicai la aceeai
putere ntreag pozitiv.
4. Ambii membri ai unei relaii de congruen pot fi nmulii cu orice
numr ntreg pozitiv, nmulind sau nu n acelai timp i modulul.
5. Orice relaie de congruen n raport cu un modul dat este o relaie de
congruen n raport cu un modulul care este divizor al modului dat.
6. Ambii membrii ai unei relaii de congruen pot fi simplificai cu orice
factor prim cu modulul.
48
Relaia de congruen fiind o relaie de echivalen pe mulimea numerelor
ntregi, determin pe aceasta o partiie, adic o mprire n clase nevide,
disjuncte i a cror reuniune acoper mulimea.

4.5. Metoda induciei matematice

Inducia matematic este o metod util n demonstrarea unor afirmaii
care depind de mulimea numerelor naturale. Fie
0
)) ( (
n
n P un ir de propoziii.
Metoda induciei matematice ne ajut s demonstrm c propoziia ) (n P este
adevrat pentru orice 0 n , unde
0
n este un numr natural fixat.
Inducia matematic (forma slab): Presupunem c
) (
0
n P este adevrat
Pentru orice
0
n k , din faptul c ) (k P este adevrat rezult c
) 1 ( + k P este adevrat.
Atunci propoziia ) (n P este adevrat pentru orice
0
n n .
Inducia matematic (cu pasul s): Fie s un numr natural fixat. Presupunem
c:
) (
0
n P , ) 1 (
0
+ n P ,..., ) 1 (
0
+ s n P sunt adevrate ;
Pentru orice
0
n k , din faptul c ) (k P este adevrat rezult c
) ( s k P + este adevrat.
Atunci propoziia ) (n P este adevrat pentru orice
0
n n .
Inducia matematic (forma tare): Presupunem c:
) (
0
n P este adevrat
Pentru orice
0
n k , din faptul c ) (m P este adevrat pentru orice m
cu k m n
0
, rezult c ) 1 ( + k P este adevrat.
Atunci propoziia ) (n P este adevrat pentru orice
0
n n .
Urmtoarele exemple ilustreaz utilizarea metodei induciei matematice n
studiul ecuaiilor diofantice.




Bibliografie

Petru Minu, Teoria numerelor, Ed. Crengua Gldu, Iai 1997.
49
T. Andreescu, D. Andrica, O introducere n studiul ecuaiilor diofanice,
Ed. Gil, Zalu, 2002.
M. Cocuz, Culegere de probleme de matematic,Ed. Academiei,1984
L.Panaitopol, M.E. Panaitopol, M. Lascu, Inducia
matematic,Ed.GIL,Zalu,2001
I.Cucurezeanu, Probleme de aritmetic i teoria numerelor, Ed.
Tehnic, Bucureti, 1976

50
4.5. Probleme rezolvate (4.1)

R4.2.1 Fie p i q numere prime. Rezolvai n numere ntregi ecuaia
q p y x +
= +
1 1 1
.
Soluie Ecuaia este echivalent cu
2 2
) )( ( q p pq y pq x = .
Considernd toi divizorii pozitivi ai numrului
2 2
q p , obinem sistemele

=
=
2 2
1
q p pq y
pq x
;

=
=
2
pq pq y
p pq x
;

=
=
q p pq y
q pq x
2

=
=
2
2
q pq y
p pq x
;

=
=
pq pq y
pq pq x
;

=
=
p pq y
pq pq x
2

=
=
q pq y
q p pq x
2
;

=
=
2
2
p pq y
q pq x
;

=
=
1
2 2
pq y
q p pq x

Obinem soluiile: )) 1 ( , 1 ( pq pq pq + + , )) 1 ( ), 1 ( ( q pq q p + + ,
)) 1 ( ), 1 ( ( p pq p q + + , )) ( ), ( ( q p q q p p + + , ) 2 , 2 ( pq pq , )) 1 ( ), 1 ( ( q p q pq + + ,
)) 1 ( ), 1 ( ( p q p pq + + , )) ( ), ( ( q p p q p q + + , ) 1 ), 1 ( ( pq pq pq + + .

R4.2.2. Rezolvai n numere ntregi x, y ecuaia 2 6 3 8 6
2 2
= + + + + y x y xy x .

Soluie Ecuaia se scrie echivalent 2 ) 3 4 )( 2 ( = + + + y x y x .
Descompunerea lui 2 n produs de doi factori pozitivi ne conduce la sistemele:

= + +
= +
2 3 4
1 2
y x
y x
cu soluia ( 1 , 3 )

= + +
= +
1 3 4
2 2
y x
y x
cu soluia ( 2 , 6 )

= + +
= +
2 3 4
1 2
y x
y x
cu soluia ( 2 , 3 )

= + +
= +
1 3 4
2 2
y x
y x
cu soluia ( 1 , 0 )



51
R4.2.3. Gsii toate tripletele (x, y, z) de numere naturale astfel nct
p xyz z y x = + + 3
3 3 3
, unde p este un numr prim mai mare dect 3.
(Titu Andreescu, Dorin Andrica)
Soluie Folosind identitatea:
) )( ( 3
2 2 2 3 3 3
zx yz xy z y x z y x xyz z y x + + + + = + +
Deoarece 1 + + z y x , rezult p z y x = + + i 1
2 2 2
= + + zx yz xy z y x .
nmulind a doua ecuaie cu 2 obinem 2 ) ( ) ( ) (
2 2 2
= + + x z z y y x .
Fr s restrngem generalitatea, putem presupune z y x . Dac x> y> z,
cum x, y, z N, avem x-y 1, y-z 1 i x-z 2, de unde rezult
2 6 ) ( ) ( ) (
2 2 2
+ + x z z y y x . nseamn c 1 + = = z y x sau
z y x = = 1 . Numrul p este prim i cum p>3, el este de forma 1 3 + k sau
2 3 + k . Din 1 + = = z y x rezult soluia
|
.
|

\
| +
3
2
,
3
1
,
3
1 p p p
i
permutrile
|
.
|

\
| +
3
1
,
3
2
,
3
1 p p p
,
|
.
|

\
| +
3
1
,
3
1
,
3
2 p p p
.
n cel de-al doilea caz, z y x = = 1 i obinem soluiile
|
.
|

\
| + +
3
1
,
3
1
,
3
2 p p p
,
|
.
|

\
| + +
3
1
,
3
2
,
3
1 p p p
,
|
.
|

\
| + +
3
2
,
3
1
,
3
1 p p p
.


Probleme rezolvate (4.4)

R4.4.1. Artai c ecuaia
2 2 2 2
) 2001 ( ... ) 2 ( ) 1 ( y x x x = + + + + + + nu are
soluii n .
Soluie Folosind substituia 1001 = z x , ecuaia devine
2 2 2 2 2 2
) 1000 ( ... ) 1 ( ) 1 ( ... ) 1000 ( y z z z z z = + + + + + + + + , sau
2 2 2 2 2
) 1000 ... 2 1 ( 2 2001 y z = + + + + , de unde
2 2
6
2001 1001 1000
2 2001 y z =

+ care se mai scrie
2 2
) 1 666 )( 2 999 )( 1 999 ( 2001 y z = + + + + . Membrul nti este congruent cu
2(mod 3), deci nu poate fi ptrat perfect.

R4.4.2. Artai c ecuaia 4
2 5
= y x nu are soluii n numere ntregi.
(Balcaniada de matematic)

52
Soluie. Considerm ecuaia modulo 11. Din Teorema lui Fermat avem
c pentru orice ntreg a nedivizibil cu un numr prim p, are loc congruena
) (mod 1
1
p a
p

. Avem deci ) 11 (mod 1


10
x dac x nu este divizibil cu 11 i
) 11 (mod 0
10
x dac 11 M x . Din
2 5 10
) (x x = rezult 1
5
= x , 0 sau ) 11 (mod 1 .
Dac ) 11 (mod 1
5
x rezult ) 11 (mod 6 ) 5 11 ( 4
5
= x . Dac
) 11 (mod 0
5
x rezult ) 11 (mod 7 ) 4 11 ( 4
5
= x . Dac ) 11 (mod 1
5
x
rezult ) 11 (mod 8 ) 3 11 ( 4
5
= x .
Pe de alt parte, analiznd toate situaiile pentru i y + = 12 11 , i
{0,1,,10}, obinem 5 , 4 , 3 , 1 , 0
2
y sau ) 11 (mod 9 , deci ecuaia nu are soluii.

R4.4.3. Aflai toate perechile (x, y) de numere naturale care verific ecuaia
7 2 3 =
y x
.

Soluie Pentru y = 1 rezult x = 2. Pentru y = 2, avem 11 3
x
care nu are
soluii. Pentru 3 y , considerm ecuaia modulo 8. Cum ) 8 (mod 0 2
y
,
deducem c ) 8 (mod 7 3
x
. Dar pentru x par ) 8 (mod 1 3
x
i pentru x impar
) 8 (mod 3 3
x
, deci ecuaia nu are soluii pentru 3 y .

Probleme rezolvate (4.5)

R4.5.1. Artai c pentru orice numr natural n, urmtoarea ecuaie are
soluie n mulimea numerelor ntregi:
n
z y x 59
2 2 2
= + + .
(Dorin Andrica)

Soluie Vom utiliza inducia matematic cu pasul s = 2 i 1
0
= n .
Observm c pentru (
1 1 1
, , z y x ) = (1,3,7) i (
2 2 2
, , z y x )=(14,39,42) avem
59
2
1
2
1
2
1
= + + z y x i
2 2
2
2
2
2
2
59 = + + z y x , deci ecuaia
n
z y x 59
2 2 2
= + + are
soluii n pentru n{1,2}. Pentru pasul de inducie definim (
n n n
z y x , , ),
3 n , prin
n n
x x
2
2
59 =
+
,
n n
y y
2
2
59 =
+
,
n n
z z
2
2
59 =
+
, ( ) 1 n . Atunci
) ( 59
2 2 2 2 2
2
2
2
2
2 k k k k k k
z y x z y x + + = + +
+ + +
, deci
k
k k k
z y x 59
2 2 2
= + + implic
2 2
2
2
2
2
2
59
+
+ + +
= + +
k
k k k
z y x .

R4.5.2. S se arate c pentru orice 3 n , ecuaia
53
1
1
...
1 1
2 1
= + + +
n
x x x
are soluii n mulimea numerelor naturale
distincte.

Soluie Pentru n = 3 avem 1
6
1
3
1
2
1
= + + . Presupunem c pentru 3 k
are loc relaia 1
1
...
1 1
2 1
= + + +
n
x x x
, unde
k
x x x ,... ,
2 1
sunt numere naturale
distincte. Prin nmulirea cu
2
1
obinem
2
1
2
1
...
2
1
2
1
2 1
= + + +
k
x x x
, de unde
deducem 1
2
1
...
2
1
2
1
2
1
2 1
= + + + +
k
x x x
. Am demonstrat propoziia pentru
numerele naturale distincte
k
x x x 2 ,..., 2 , 2 , 2
2 1
, deci pentru orice 3 n ecuaia
are soluii numere naturale distincte.

R4.5.3. Demonstrai c ecuaia
2 2 2
) 1 ( y x x = + + are o infinitate de soluii n
mulimea numerelor naturale.

Soluie. Ecuaia este verificat pentru 3
1
= x , 5
1
= y . Definim irurile
1
) (
n n
x ,
1
) (
n n
y , prin relaiile de recuren:

+ + =
+ + =
+
+
2 3 4
1 2 3
1
1
n n n
n n n
y x y
y x x
, cu 3
1
= x , 5
1
= y
Pentru pasul de inducie, presupunem c perechea (
n n
y x , ) este soluie
pentru ecuaia dat. innd seama de
2 2 2
) 1 (
n n n
y x x = + + , obinem
2 2 2 2
1
2
1
) 2 3 4 ( ) 2 2 3 ( ) 1 2 3 ( ) 1 ( + + = + + + + + = + +
+ + n n n n n n n n
y x y x y x x x . Deci
2
1
2
1
2
1
) 1 (
+ + +
= + +
n n n
y x x , adic (
1 1
,
+ + n n
y x ) este soluie a ecuaiei.


54
5. Inducia matematic n geometrie

Metoda induciei matematice este frecvent utilizat n diferite ramuri ale
matematicii i constituie unul dintre cele mai puternice mijloace de
demonstraie n matematic dup afirmaia academicianului Miron Nicolescu.
n paragraful 4.4 sunt prezentate trei forme ale induciei matematice:
forma slab, inducia cu pasul 2 i forma tare. Urmtoarele exemple ilustreaz
utilizarea metodei induciei matematice la rezolvarea unor probleme de
geometrie.




Bibliografie

D. Andrica, C. Varga, D. Vcreu, Teme i probleme alese de
geometrie, Ed. Plus, Bucureti, 2002, pag 283-316.
L. Nicolescu, A. Bumbcea, Metode de rezolvare a problemelor de
geometrie, Ed. Univ. Bucureti, 1998, pag278-291.
L.Panaitopol, M.E. Panaitopol, M. Lascu, Inducia
matematic,Ed.GIL,Zalu,2001
M. Pimsner, S. Popa, Probleme de geometrie elementar. E.D.P.1979.
L.I. Golovina, I.M. Iaglom, Inducia n geometrie, Ed.Tehnic, 1954.


55
Probleme rezolvate (5.1)

R5.1.1. Fie O un punct de pe dreapta l, iar
n
OP OP OP , , ,
2 1
K , sunt n vectori de
lungime 1, astfel nct punctele P
1
,, P
n
se afl ntr-un plan care conine
dreapta l i sunt toate de aceeai parte a acesteia. S se demonstreze c dac n
este impar, atunci 1
2 1
+ + +
n
OP OP OP L ,
unde OM reprezint lungimea vectorului OM .
(O.I.M. 1973 propus de Cehoslovacia)

Soluie:
Demonstrm prin metoda induciei, forma slab.
Pentru n=1, 1
1
OP , adevrat. Presupunem proprietatea adevrat pentru 2n
1 vectori i o demonstrm pentru 2n + 1 vectori. Renumerotnd eventual
punctele P
1
, P
2
,
, P
2n+1
, putem presupune c cel mai mare unghi
J i
OP P i,j=1, 2n+1 este
1 2 1 +

n
OP P . Din ipoteza inductiv rezult
1
2 3 2
+ + +
n
OP OP OP L
Fie .
1 2 1
OR OP OP
n
= +
+
OR OP P
n
= +
+1 2 1
. Dac
n
OP OP OP OS
2 3 2
+ + + = L i
OS OR OT + = , atunci T, S ( )
1 2
int
+

h
ROP sau T, S ( )
1
int ROP de unde
OTOS, deoarece ROS este ascuit. Rezult 1 OT i inducia este ncheiat.

R5.1.2. S se demonstreze c pentru orice n 4 exist un poligon convex cu n
laturi, nu toate egale, astfel nct suma distanelor de la orice punct interior la
laturi s fie constant.
(Baraj 1974, Dan Schwartz)
Soluie:
S observm mai nti c triunghiul echilateral i dreptunghiul cu proprietatea
c suma distanelor oricrui punct interior la laturi este constant, P(4) este deci
adevrat. Pentru a demonstra P(5) pornim de la triunghiul echilateral ABC pe
care-l tiem cu dou drepte paralele dup o direcie care nu este paralel cu nici
o latur a triunghiului ABC, ca n figur.



56
M
P
B
A
N
C
Q
A
1
D
1
C
1
B
1
N
1
M
1
P
1
Q
1

Fig. 5.1 Fig. 5.2

Pentagonul BMNQP are proprietatea cerut. Pentru P(6) considerm
dreptunghiul A
1
B
1
C
1
D
1
pe care-l tiem cu dou drepte paralele dup o direcie
neparalel cu laturile dreptunghiului; atunci hexagonul B
1
M
1
N
1
D
1
P
1
Q
1
are
proprietatea cerut. Am demonstrat c proprietatea este adevrat pentru
n=4,5,6. Vom demonstra implicaia P(n)P(n+2) (inducie cu pasul 2).
Considerm un poligon convex cu n laturi care satisface condiia problemei.
Repetnd procedeul de mai sus, tiem dou vrfuri ale poligonului cu dou
drepte paralele dup o direcie neparalel cu nici o latur a poligonului i
obinem un poligon cu n+2 laturi, nu toate egale. Suma distanelor unui punct
interior la laturi este egal cu suma distanelor la laturile vechiului poligon, care
este constant, plus suma distanelor la cele dou laturi noi, care sunt paralele,
deci i aceast sum este constant.

R5.1.3. Se dau n suprafee ptratice arbitrare (n 2). S se demonstreze c ele
pot fi tiate n pri, astfel nct din prile obinute s se poat construi o nou
suprafa ptratic.
(Golovina L. I., Iaglom I.M. Inducia n geometrie, Ed. Tehnic, Bucureti,
1964)

57
Soluie:
n demonstraia prin inducie este esenial cazul n=2. Fie suprafeele ptratice
[ABCD] i [ABCD] avnd lungimile laturilor l, respectiv l. Dac l=l
ducem diagonalele [AC], [BD] formnd patru suprafee triunghiulare care se
pot compune formnd un ptrat cu latura lV2 ca n figur. Fig 5.3
D C
B
A
S
1
S
2

D C
B A
S
3
S
4

D
C
B
A
S
3
S
2
S
1
S
4

Fig 5.3. Fig. 5.4
n cazul l l. putem lua l > l.
Considerm punctele M(AB), N(BC), P(CD), Q(DA), astfel ca
2
' l l
DQ CP BN AM
+
= = = = . Atunci
2
' l l
QA PD NC MN

= = = =


58
D C
B
A
D
C
B
A
D
C
B
A
Q
P
N
M
O

D
C
B A

Fig. 5.5 fig. 5.6
Triunghiurile dreptunghice AQM, BMN, CNP, DPQ fiind congruente, rezult
QM=MN=NP=PQ, deci MNPQ este romb. Din congruena triunghiurilor AQM
i BMN rezult

( ) ( ) BNM m AMQ m = . Dar ( ) ( ) = + 90 BNM m BMN m , deci
( ) ( ) = + 90 BMN m AMQ m i prin urmare ( ) = 90 QMN m i MNPQ este ptrat
cu lungimea laturii
2
'
2
'
2
'
2 2
2 2
l l l l l l
MN
+
=

+
= i lungimea
diagonalei
2 2
' l l MP + = .
Notnd [MP][NQ]={ } 0 , patrulaterele
inscriptibile AQOM, BMON, CNOP, DPOQ sunt
congruente, deci suprafaa ptratic s-a descompus
n patru suprafee patrulatere congruente. Detam
aceste suprafee i le alturm suprafeei ptratice
[ABCD], astfel ca vrfurile O ale celor patru
patrulatere devin vrfurile ABCD ale noului
ptrat de latur AB= V(l
2
+l
2
) . S demonstrm
implicaia P(n)P(n+1). Presupunem
proprietatea adevrat pentru n suprafee ptratice (n 2). Fie S
1
, S
2
, fig.
5.7 ,S
n
, S
n+1
suprafee ptratice. Conform ipotezei de inducie, S
1
, S
2
, ,S
n
,
pot fi tiate n pri astfel nct din prile obinute s se poat construi o alt
suprafa ptratic S. Procednd cu S i S
n+1
ca i n cazul n=2, obinem o
nou suprafa ptratic S i demonstraia este ncheiat.

R5.1.4. S se arate c numrul prilor n care planul este mprit de n drepte
(n 1) dou cte dou secante i trei cte trei neconcurente este
2
2
2
+ +
=
n n
F
n
.

59
Soluie:
Vom nelege prin parte a planului, fie o anumit suprafa poligonal, fie o
anumit intersecie de semiplane nchise. n cazul n=1, o dreapt mparte planul
n dou semiplane:
2
2 1 1
2
1
+ +
= F . Presupunem c n drepte (n 1), dou cte
dou secante i trei cte trei neconcurente, mpart planul n
2
2
2
+ +
=
n n
F
n

pri. Considerm n plan n+1 drepte n poziia general. Primele n dintre ele
mpart planul n F
n
pri. ( ) a n A +1 dreapt, pe care o notm cu d, se
intersecteaz cu fiecare dintre cele n drepte (din ipotez). Cele n puncte
distincte de pe dreapta d determin pe aceastan-1 segmente i dou semidrepte.
Astfel, dreapta d intersecteaz n+1 pri din F
n
pri cte erau, deci numrul
prilor crete cu n+1. Rezult
( ) ( )
2
2 1 1
2
4 3
1
2
2
1
2 2 2
1
+ + + +
=
+ +
= + +
+ +
= + + =
+
n n n n
n
n n
n F F
n n


R5.1.5. Se d o suprafa poligonal convex [A
1
A
2
A
3
A
n
] (n 5) cu
proprietatea c oricare trei diagonale nu sunt concurente. S se arate c numrul
suprafeelor poligonale n care [A
1
A
2
A
3
A
n
] este descompus de diagonalele
sale este
( )( )( )
24
12 3 2 1
2
+
=
n n n n
f
n
.
(Iaglom A.M., Iaglom I.M. Probleme neelementare tratate elementar)
Soluie:
Pentru n=5, suprafaa [A
1
A
2
A
3
A
n
] se descompune n 11 suprafee (vezi
figura), notate cu S
1
S
11
. Avem
( )
11
24
12 15 25 3 4
=
+
=
n
f . Presupunem
proprietatea adevrat pentru (n 5) i
considerm o suprafa poligonal convex
[A
1
A
2
A
3
A
n
A
n+1
] cu oricare trei diagonale
neconcurente. Ducem diagonalele acestei
suprafee exceptndu-le pe cele care pleac din
vrful A
n+1
. Notm [A
1
A
2
A
3
A
n
A
n+1
]=S
n+1
,
[A
1
A
2
A
3
A
n
]=S
n
.Din ipoteza de inducie
avem c S
n
se descompune n f
n
suprafee
poligonale.
S
n+1
se descompune n f
n
+1 suprafee
poligonale, deoarece am adugat [A
1
A
2
A
3
A
n

A
n+1
] i s ducem diagonalele din vrful A
n+1
n

A
5
A
4
A
3
A
2
A
1
S
8
S
7
S
6
S
11
S
9
S
10
S
1
S
2
S
3
S
4
S
5

Fig. 5.8

60
numr de n-2, apoi s vedem cu ct se mrete numrul f
n
+1.
Considerm diagonala [A
n+1
A
k
] (k2,3,n-1) (vezi figura 5.8). De o parte a ei
se afl k-1 vrfuri (A
1
A
2
A
3
A
k+1
) iar n cealalt n-k vrfuri (A
k+1
A
k+2
A
n
).
Deci diagonala [A
n+1
A
k
] intersecteaz (k-1)(n-k) diagonale ale lui S
n+1
, prin
urmare se obine un adaos de (k-1)(n-k)+1 suprafee poligonale. Raionamentul
este valabil pentru toate cele n-2 diagonale ce pornesc din A
n+1
i putem scrie:
( )( ) [ ] ( ) [ ] ( ) + = + + + = + + + =


=

=
+

=
+
2
1
2
1
1
1
2
2
1
1 1 1 1 1 1 1
n
k
n
k
n
n
k
n n n
k n f k k n f k n k f f

( )( )( )
( )
( )( ) ( )( )( )
+


+ +
+
= +

=
6
3 2 1 2
2
1 2
1
24
12 3 2 1
2
2 2
1
2
n n n n n
n
n n n n
n k
n
k
( )( ) ( ) ( ) ( ) [ ]
24
12 1 3 1 1
24
10 1
1
2 2
+ + +
=
+
= +
n n n n n n n n
n .
Observaie: Formula pentru
n
f
rmne valabil att pentru
suprafee triunghiulare
( )
1
24
12 9 9 1 2
3
=
+
= f , ct i
pentru suprafee patrulatere
convexe cu 2 diagonale
( )
4
24
12 12 16 2 3
4
=
+
= f


R5.1.6. Fie n plan o reea de
linii ce unesc ntre ele punctele
A
1
,A
2
,A
n
i nu au alte puncte
comune. Presupunem reeaua
ca fiind construit dintr-o
singur bucat, adic din fiecare punct A
1
,A
2
,A
n
se poate ajunge n oricare
altul numai de-a lungul liniilor reelei. O astfel de reea de linii o numim hart,
punctele date vrfurile ei, poriunile de curbe dintre dou vrfuri vecine
frontierele (graniele) hrii, poriunile din plan n care ea este descompus de
ctre frontiere rile hrii.
Teorema lui Euler
S notm cu S numrul rilor unei hri arbitrare, cu l numrul frontierelor ei i
cu p numrul vrfurilor. Atunci
S+p=l+2

A
1 A
2
A
3
A
k
A
n
A
n+ 1

fig. 5.9

61
Soluie:
Demonstrm egalitatea prin inducie dup numrul l al frontierelor hrii.
I. Fie l=0, atunci S=1, p=1 S+p=l+2
II. Presupunem c relaia este adevrat pentru orice hart care are n
frontiere. Considerm o hart cu l=n+1 frontiere, S ri i p vrfuri.
Distingem dou situaii:
i) Pentru orice pereche de vrfuri ale hrii exist un drum unic care
le unete de-a lungul frontierelor (exist cel puin unul, deoarece harta este
conex). n acest caz harta nu conine nici un contur nchis. (vezi figura 5.9)



A
1
A
3
A
2
A
4
A
5
A
6
A
8
A
9
A
10
A
12
A
11

fig. 5.10


n acest caz S=1. S artm c pe o astfel de hart se va gsi cel puin
un vrf aparinnd numai unei singure frontiere (de exemplu A
1
sau A
3
)numit
vrf extrem. ntr-adevr s lum un vrf arbitrar. Dac el nu este extrem, atunci
el reprezint captul a cel puin dou frontiere. S parcurgem una dintre
frontiere pn la al doilea vrf al su. Dac nici acest vrf nu este extrem, atunci
el constituie captul unei alte frontiere i parcurgem aceast frontier pn la al
doilea capt al ei i aa mai departe. Deoarece harta nu conine contururi
nchise, nu ne vom ntoarce la nici unul din vrfurile parcurse nainte i dup un
numr finit de pai ajungem la un vrf care va fi extrem. ndeprtnd acest vrf
mpreun cu o frontier care l are drept capt, obinem o nou hart n care
l=l-1, S=S, p=p-1 Din ipoteza de inducie, S+p*=l+2, de unde S+p=l+2.
ii) Exist dou vrfuri unite prin mai multe drumuri (vezi figura 5.10)


62
A
1
A
2
A
5
A
6
A
7
A
4
A
3

fig. 5.11

ndeprtnd una din frontierele acestui contur (fr vrfuri) obinem o
nou hart conex n care l=l-1, p=p. S=S-1. Din ipoteza de inducie
S+p=l+2, de unde S+p=l+2.

R5.1.7. Notm cu v numrul vrfurilor, m numrul muchiilor i f numrul
feelor unui poliedru din spaiul euclidian. Pentru a obine formula v-m+f=2, n
cazul unui poliedru simplu (conex) procedm n modul urmtor: Ne imaginm
c poliedrul este confecionat dintr-o foi de cauciuc subire i observm c
dac nlturm una din fee putem deforma suprafaa rmas ntinznd-o pe un
plan. Obinem astfel o hart planar care ale acelai numr de vrfuri i muchii
ca poliedrul iniial. nlocuind faa nlturat cu faa infinit i utiliznd
rezultatul demonstrat mai sus pentru hri plane conexe gsim relaia lui Euler
n cazul poliedrelor simple.
S artm c exist cinci tipuri de poliedre conexe regulate i anume: tetraedrul,
hexaedrul (cubul), dodecaedrul, octaedrul i icosaedrul regulat.

Soluie:
Fie q numrul muchiilor de pe o fa i p numrul muchiilor ce pleac dintr-un
vrf. Cum fiecare muchie aparine frontierei pentru dou fee i conine dou
vrfuri, rezult c
q
m
f
p
m
v p v q f m
2
,
2
2 = = = = .
Din formula lui Euler 2
2 2
= +
q
m
m
p
m
sau
> + =

+ 0
2
1 1 1
1
2
1 1 1
q p q p
m

63
6
1
3
1
2
1 1
2
1 1
= >
q p
. Deci 6 < p i analog, 6 < q . Dac
> > >
p q
p
1
2
1 1
4 ,
p
1
4
1
2
1
= > deci 4 < q . Obinem c singurele perechi
( ) q p, care verific inegalitatea 0
2
1 1 1
> +
q p
sunt (3,3), (3,4), (3,5), (4,3),
(5,3)

p q v m f Denumire
3 3 4 6 4 tetraedru
3 4 8 12 6 cub
4 3 6 12 8 octaedru
3 5 20 30 12 dodecaedru
5 3 12 30 20 icosaedru


64
6. Metoda vectorial n geometrie

6.1. Consideraii teoretice

Pentru studiul geometriei euclidiene plane se folosesc mai multe
modele: sintetic, analitic, vectorial, complex. Fiecare dintre aceste modele se
dovedete a fi mai eficient ntr-un anumit tip de probleme, de aceea este util s
le cunoatem pe toate i s avem posibiltatea de a trece cu uurin de la unul la
altul. Modelul vectorial se preteaz la probleme n care apar drepte, segmente,
rapoarte, probleme care de multe ori se rezolv mai simplu dect s-ar rezolva pe
cale sintetic. Planul euclidian definit axiometric (modelul sintetic) l
considerm cunoscut, elementele sale fiind puncte.
Modelul analitic l obinem alegnd un sistem de coordonate n plan i
considernd planul ca produsul cartezian a dou drepte (ortogonale) :
( ) { } R R R = = y x y x , \ ,
2
.
Modelul vectorial l obinem alegnd n planul un punct fix, numit
origine i doi vectori necoliniari de baz, de exemplu i
r
i j
r
reprezentai prin
dou sgei (versorii directori ai axelor
x
O i
y
O ).
Fig. 6.1.

Deci { } R R + = = = y x j y i x v , \
2
r r
r
.
Orice punct M din planul are n modelul analitic dou coordonate M(x,y),
deci este unic determinat de dou numere reale xR (abscisa) i
y R (ordonata). Acelai punct M are n modelul vectorial un vector de poziie
j y i x r
M
r r
+ = , deci orice punct este unic determinat de vectorul su de
poziie, care se reprezint printr-o sgeat ce pornete din originea O i se
termin n M.

65
Pentru o pereche de puncte (A,B) , reprezentm printr-o sgeat
ce pornete din A i se termin n B, segmentul orientat (A,B). Fiecrui segment
orientat (A,B) i se ataeaz un vector
2
R AB , definit prin
A B
r r AB = .

Fig. 6.2.


Trei puncte A, B, C sunt coliniare dac vectorii AB i AC sunt
coliniari, adic dac exist un numr real t astfel ca AB t AC =
Mulimea punctelor coliniare cu dou puncte distincte A, B formeaz
dreapta AB.
Dintre ecuaiile dreptei amintim:

a) : D v t r r + =
0
; t R
Ecuaia dreptei ce trece prin vrful vectorului
0
r i este paralel cu vectorul
0 v .

b) : D ( ) ; 1
B A
r t r t r + = R t .
Ecuaia dreptei ce trece prin A i B.

n ecuaia b) a dreptei, punctul M , al crui vector de poziie este
( )
B A M
r t r t r + = 1 , cu [ ] 1 , 0 t se afl pe segmentul [ ] AB i este determinat de
raportul distanelor
( )
( )
AB
AM
B A d
M A d
t = =
,
,
.
n particular mijlocul C al segmentului [ ] AB are vectorul de poziie

66
( )
B A C
r r r + =
2
1
.
Dac ABC este un triunghi, atunci pentru orice punct M din plan, exist
numerele reale , , unic determinate astfel ca:

C B A M
r r r r + + = , cu 1 = + + .
Dac , , sunt pozitive, punctul M se afl n interiorul triunghiului
ABC i numerele , , reprezint rapoarte de arii:
= =
ABC
MBC
S
S
;
ABC
MCA
S
S
= ;
ABC
MAB
S
S
= .
n particular pentru punctele importante din triunghi avem:

( )
C B A G
r r r r + + =
3
1
(centrul de greutate);
c b a
r c r b r a
r
C B A
I
+ +
+ +
= (centrul cercului nscris);
tgC tgB tgA
r tgC r tgB r tgA
r
C B A
H
+ +
+ +
= (ortocentrul triunghiului);
C B A
r C r B r A
r
C B A
O
2 sin 2 sin 2 sin
2 sin 2 sin 2 sin
+ +
+ +
= (centrul cercului circumscris);
H G E
r r r + =
4
3
(centrul cercului lui Euler);




Bibliografie

(1) BRNZEI, D. :Bazele geometriei analitice plane, Editura Paralele 45,
Piteti, 1999
(2) BESOIU, I., BESOIU, E. : Probleme de geometrie rezolvate cu vectori
pentru clasele a IX-a i a X-a, Editura
STAR SOFT, Alba Iulia, 2000.

67
(3) BRNZEI, D., NECHITA,V., ERDEAN, V. : Dicionar de geometrie
elementar, Editura Paralela 45, Piteti,
2001.
(4) BIBOAC, N. : Teme complementare de geometrie, Editura Paralela
45, Piteti, 1999.
(5) BRNZEI,D., ZANOSCHI, A. : Geometrie probleme cu vectori, clasa
a IX-a, Editura Paralela 45, Piteti, 1999.
(6) NECHIL, P. : Algebr vectorial i geometrie analitic, Editura
Paralela 45, Piteti, 2001.
(7) BRNZEI, D., ONOFRA, E., ANIA, S., ISVORANU, GH. : Bazele
raionamentului geometric, Editura
Academiei R.S.R., Bucureti, 1983.
(8) DRANCA, C., VORNICESCU, FL., RADU, L., VORNICESCU, N. :
Probleme i soluii de geometrie
vectorial, analitic i trigonometrie,
Editura Didactic i Pedagogic,
Bucureti, 2002.
(9) SIMIONESCU, GH., D. : Noiuni de algebr vectorial i aplicaii n
geometrie, Editura Tehnic, Bucureti,
1982.
(10) CMPEAN, V., CMPEAN, V. : Vectori geometrie, clasele IX X,
Editura Milenium, Alba Iulia, 2003.
(11) PIMSNER, M., POPA, S. : Probleme de geometrie elementar, Editura
Didactic i Pedagogic, Bucureti, 1979.
(12) NICOLESCU, L., BUMBCEA, AL., CATAN, A., HORJA, P.,
NICULESCU, GH., OPREA, N.N ZARA, C. : Metode de rezolvare a
problemelor de geometrie, Editura Universitii din Bucureti, 1998.

68
(13) ANDRICA, D., VARGA, Cs., VCREU, D. : Teme i probleme alese
de geometrie, Editura Plus, Bucureti, 2002.
(14) NICULA, V. : Geometrie plan (sintetic, vectorial, analitic)
culegere de probleme, Editura Gil, Zalu, 2002.
(15) CSINTA, Th., MODAN, L. : Probleme de matematic date ntre 1998-
2002 la concursul de admitere n grupul GEIPI al colilor superioare
franceze de nalte studii inginereti, vol. I, Editura Gil, Zalu, 2003.
(16) BRNZEI, D., ERDEAN, I., ERDEAN, V. : Olimpiadele balcanice de
matematic pentru juniori, Editura Plas, Bucureti, 2003.




69
6.2. Probleme rezolvate (6.1)

R6.2.1. Fie ABCDE un pentagon i M, N, P, Q, R, S mijloacele segmentelor
AB, BC, CD, DE, MP i NQ. S se arate c RS .
4
1
AE =

Soluie. Avem ( )
B A M
r r r + =
2
1
i analoagele.
Rezult ( )
D C B A R
r r r r r + + + =
4
1
i ( )
E D C B S
r r r r r + + + =
4
1
, deci
( ) AE r r r r RS
A E R S
4
1
4
1
= = = .
Atunci AE RS
4
1
= i n plus segmentele RS i AE sunt paralele.


R6.2.2. Dou segmente | |
1 1
, B A i | |
2 2
, B A alunec pe dou drepte
( )
1
d i ( )
2
d . Fie
3
A mijlocul segmentului | |
2 1
, A A i
3
B mijlocul segmentului
| |
2 1
, B B . S se arate c segmentul | |
3 3
, B A are lungime constant.

Soluie. Dac
X
r este vectorul de poziie al punctului X, avem:
( )
2 1
2
1
A A A
r r r + =
3
, ( )
2 1 3
2
1
B B B
r r r + = i atunci = =
3 3
3 3 A B
r r B A
( ) ( ) c B A B A r r r r
A B A B
= + = + =
2 2 1 1
2
1
2
1
2 2 1 1
, vector constant, cci vectorii
1 1
B A i
2 2
B A ,asociai segmentelor orientate ( )
1 1
, B A i ( )
2 2
, B A , sunt vectori
constani.
Observaie:
a) Mrimea segmentului
3 3
B A este egal cu mediana unui triunghi ABC
n care
1 1
B A AB ,
2 2
B A AC i ( )
2 1
, d d BAC = .
b) Din egalitatea vectorial rezult c n orice dou poziii ale
segmentelor care alunec, segmentele | |
3 3
, B A i | |
'
3
'
3
, B A sunt paralele.
c) Rezultatul se menine dac se definesc punctele
3
A i
3
B ca mprind
segmentele | |
2 1
, A A , respectiv | |
2 1
, B B n acelai raport.



70
R6.2.3. Dou mobile
1
M i
2
M se mic cu viteze constante
1
v i
2
v pe
dreptele ( )
1
d i ( )
2
d . Dac M este mijlocul segmentului | |
2 1
, M M s se arate c
M se deplaseaz pe o dreapt, cu vitez constant.

Soluie. Fie
2 1
, A A poziiile din care pleac cele dou mobile i
( ) ( ) t M t M
2 1
, poziiile lor dup timpul t. Avem:
1
1 1
v t r r
A M
+ = ,
2
2 2
v t r r
A M
+ = i ( ) ( ) ( ) v t r v v r r r r r
A A A M M M
+ = + + + = + =
2 1
2
1
2
1
2
1
2 1 2 1
, unde A
este poziia iniial a lui M , iar ( )
2 1
2
1
v v v + = este media vitezelor. O micare
rectilinie i uniform este definit de legea:
( )
v t r r
M M
t
+ =
) 0 (
, t>0 reprezint
timpul, deci M se mic rectiliniu i uniform.
Observaie: Dac P este un punct de pe segmentul | |
2 1
, M M care l
mparte n raport constant, atunci punctul P se mic pe o dreapt cu viteza
medie ponderat a vitezelor punctelor
1
M i
2
M .


R6.2.4. Fie ABC un triunghi i M un punct n planul su. Notm cu
2 2 2
, , C B A
simetricele lui M fa de mijloacele
1 1 1
, , C B A ale laturilor BC, CA. AB. S se
arate c dreptele
2
AA ,
2
BB ,
2
CC sunt concurente.

Soluie. Avem
M C B M A A
r r r r r r + = =
1 2
2 i analoagele.
Un punct de pe dreapta
2
AA are vector de poziie de forma :
( ) ( ) ( )
M C B A A A
r r r t r t r t r t r AA + + = + = 1 1 :
2
2
, t R .
Analog:
( ) ( )
M A C B
r r r s r s r BB + + = 1 :
2
, R s ;
( ) ( ), 1 :
2 M B A C
r r r u r u r CC + + = ; R u
Pentru
2
1
= = = u s t se obine acelai punct:
( )
M G M C B A N
r r r r r r r
2
1
2
3
2
1
2
1
= + + = ;
Punctul de intersecie N se afl pe dreapta GM.


71
R6.2.5. Pe laturile AB i AC ale triunghiului ABC se iau punctele variabile M
i N astfel ca . CN BM S se arate c punctul P, mijlocul segmentului | | N M,
se mic pe o dreapt paralel cu bisectoarea unghiului A.

Soluie. Alegem originea n A i atunci b AB = , c AC = .
Avem:
b
b
t b r
M
= ,
c
c
t c r
N
= ,
|
|
.
|

\
|
+
+
=
c
c
b
b
t
c b
r
P
2
, R t .
Dar
b
b
i
c
c
sunt versorii laturilor AB i AC, suma lor dnd direcia bisectoarei
din A. Deci
1
'
d t r r
A
P
= R t , P se afl pe dreapta ce trece prin A, mijlocul
lui BC, paralel cu bisectoarea din A.


R6.2.6. Fie triunghiul
3 2 1
A A A i M un punct n planul su. Pentru fiecare
permutare a vrfurilor
) 1 (
A ,
) 2 (
A ,
) 3 (
A , notm cu
) 1 (
M simetricul lui M fa
de
) 1 (
A , cu
) 2 (
M simetricul lui
) 1 (
M fa
) 2 (
A ,
) 3 (
M simetricul lui
) 2 (
M
fa de
) 3 (
A . S se arate c pentru toate cele 6 permutri se obin trei puncte
3 2 1
, , M M M care formeaz un triunghi de arie
3 2 1 3 2 1
16
A A A M M M
S S = .

Soluie. Avem:
M A M
r r r =
) 1 ( ) 1 (
2

;

M A A M
r r r r + =
) 1 ( ) 2 ( ) 2 (
2 2

;

M A A A M
r r r r r + =
) 1 ( ) 2 ( ) 3 ( ) 3 (
2 2 2

;

Cele trei puncte au vectorii de poziie:
( ) ; 2
1 3 2 1
M A A A M
r r r r r + =
( ) ; 2
2 3 1 2
M A A A M
r r r r r + =
( ) ; 2
3 2
2 3
M A A A M
r r r r r + =
Avem: ( )
1 2 2 1
4 4
2 1
A A r r M M
A A
= = , deci laturile triunghiului
3 2 1
M M M sunt
paralele cu ale triunghiului
3 2 1
A A A i de patru ori mai mari. Raportul de
asemnare este 4, iar raportul ariilor 16.




72

R6.2.7. S se determine locul geometric al centrelor de greutate al
triunghiurilor
1 1 1
, , C B A cu vrfurile variabile pe laturile BC, CA, AB ale
triunghiului ABC.

Soluie. Avem: ( )
C B A
r t r t r + = 1
1
, ] 1 , 0 [ t ;
( ) , 1
1
A C B
r s r s r + = | | 1 , 0 s ;
( ) , 1
1
B A C
r u r u r + = | | 1 , 0 u ;

( )
C B A C B A G
r
t s
r
u t
r
s u
r r r r
3
1
3
1
3
1
3
1
1 1 1 1
+
+
+
+
+
= + + = .
Notnd ,
3
1 s u +
= ,
3
1 u t +
=
3
1 t s +
= , avem 1 = + + ,
0 , 0 , 0 , 2 1 0 1 1 = + + s u deci .
3
2
,
3
2
,
3
2
Din simetrie
se obin
3
1
,
3
1
,
3
1
, deci locul geometric este
)
`

= + + + + =
3
2
,
3
1
,
3
2
,
3
1
,
3
2
,
3
1
, 1 \
C B A
r r r r , care
reprezint interiorul i frontiera unui hexagon n care trei laturi sunt treimile din
mijloc ale fiecrei laturi din triunghiul ABC.

Fig. 6.3.


73
7. Probleme de concuren i de coliniaritate rezolvate vectorial

Aceasta tem este o continuare a temei precedente n care au fost amintite
principalele noiuni teoretice , necesare rezolvrii problemelor de geometrie
vectorial.
Pentru demonstrarea problemelor de coliniaritate amintim urmtoarea :
Teorem de coliniaritate: Doi vectori nenuli sunt coliniari dac i
numai dac exist
*
astfel nct . b a =
Ca o consecin a teoremei avem condiia de coliniaritate a trei puncte :
Punctele A, B, C sunt coliniare dac i numai dac vectorii , AB
uuuur
AC sunt
coliniari.





Bibliografie

L. Nicolescu, A. Bumbcea, Metode de rezolvare a problemelor de
geometrie, Ed. Univ. Bucureti, 1998, pag 7-79.
L. Nicolescu, V.Boskoff, Probleme practice de geometrie, Ed. Tehnic,
Bucureti, 1990
D. Andrica, C. Varga, D. Vcreu, Teme i probleme alese de
geometrie, Ed. Plus, Bucureti, 2002.


74
Probleme rezolvate (7.1)

R7.1.1 Teorema lui Menelaus : Fie triunghiul ABC i punctele coliniare
1
A BC ,
1
B AC ,
1
C AB . Atunci are loc relaia 1
1
1
1
1
1
1
=
B C
A C
A B
C B
C A
B A
(*)
Soluie. Considerm cazul cnd dou puncte sunt pe laturi i unul pe
prelungirea unei laturi ( Fig.7.1 )

Din
1
1
1 AC
A B
= rezult
1
1
1
AC
BC
=
+
, deci
1
1
1
CA CB

=
+
uuur uuur
.
Din
1
1
BC
B A
= rezult
1
1
CB CA

=
+
uuur uuur
.
Avem
1
1 1
1
AC
CC CA AC CA AB CA AB
AB

= + = + = +

uuuur uuur uuuur uuur uuur uuur uuur


.
Putem acum exprima vectorii
1 1
B A
uuuur
i
1 1
BC
uuuur
.
1 1 1 1
1
1 1
B A CA CB CB CA


= =
+ +
uuuur uuur uuur uuur uuur

( )= +

+
+
=
+

+ = = CB AC CA CA AB CA CB CC C B
1 1
1
1 1
1 1 1 1



( )( )
.
1 1 1
1
CB CA

+
+

=




Din condiia ca vectorii
1 1
B A
uuuur
i
1 1
BC
uuuur
s fie
coliniari, rezult
( )

1
1 ( )( )
( )( )
1 1
1 1


+
=
+ +
, de unde obinem 1 = , care
este tocmai (*)

Notm
C A
B A
1
1
= ,
,
1
1
A B
C B
=
.
1
1
B C
A C
=
Fig 7.1
75
Observaii 1. Analog se trateaz cazul cnd punctele
1
A ,
1
B ,
1
C sunt pe
prelungirile laturilor.
2.Are loc i reciproca teoremei : Dac pentru punctele
1
A BC ,
1
B AC ,
1
C AB are loc relaia (*) , atunci punctele
1
A ,
1
B ,
1
C sunt colineare.
Pentru demonstraia afirmaiei s considerm configuraia din ( Fig 7.1).
Notnd
1
1
A B
AC
= ,
1
1
BC
B A
= i
1
1
1 C A
C B

= = , avem
1 1 1 1
B A CA CB = =
uuuur uuur uuur

1
1+ 1
CB CA


=
+
uuur uuur
(1)
( )( )
1 1 1 1
1
1 1 1
BC CC CB CA CB


+
= = +
+
uuuur uuuur uuur uuur uuur
i nlocuind
1

= obinem
1 1
1 1
1 1 1
BC CB CA


| | +
=
|
+ +
\ .
uuuur uuur uuur
(2). Din (1) i (2) avem
1 1 1 1
1
1
BC B A

+
=

uuuur uuuur
, deci
1
A ,
1
B ,
1
C sunt puncte coliniare. Cealalt situaie se
trateaz analog.


R7.1.2. Teorema triunghiurilor ortologice
Fiind date dou triunghiuri ABC si ABC situate n acelai plan i
dispuse astfel nct perpendicularele din A, B, C pe laturile BC , CA , AB ,
sunt concurente , atunci i perpendicularele din A, B,C respectiv pe laturile
BC, CA, AB sunt concurente.

Soluie

Fig 7.2
76
Notm cu O punctul de concuren al perpendicularelor din A, B, C pe
laturile triunghiului ABC si cu O punctul de intersecie al perpendicularelor
din A si B pe BC respectiv AC (Fig 7.2 ). Avem :
' ' ' ' ' ' B C O C O B =
uuuuur uuuuur uuuuur

' ' ' ' ' ' C A O A O C =
uuuuur uuuuur uuuuur

' ' ' ' ' ' A B O B O A =
uuuuur uuuuur uuuuur

Folosind aceste relaii obinem ' ' ' ' ' ' O OA B C OB C A OC A B = + + =
uuur uuuuur uuur uuuuur uuur uuuuur

' ' ' ' ' ' O C BA O B AC O A BC = + +
uuuuur uuur uuuuur uuur uuuuur uuur
. Cum ' ' O B AC i ' ' O A BC , avem
' ' 0 O B AC =
uuuuur uuur
, ' ' 0 O A BC =
uuuuur uuur
i deci ' ' 0 O C BA =
uuuuur uuur
, de unde rezult
' ' O C AB . Prin urmare perpendicularele din A, B, C pe laturile triunghiului
ABC sunt concurente n O.

R7.1.3. ( Dreapta lui Gauss)
Fie ABCD un patrulater convex, { } AD BC E = , { } AB DC F = .
Atunci mijloacele M, N, P ale segmentelor (AC), (BD), (EF) sunt puncte
colineare.

Soluie


Notnd p
AB
AF
= i q
AD
AE
= , , AB u AD v = =
uuur r uuur r
(Fig 7.3) atunci
v q AE u p AF = = , ,
( ) ( )
1 1
,
2 2
AN u v AP pu qv = + = +
uuur r r uuur r r
. Notnd
CF
r
CD
= ,
Fig 7.3
77
CB
t
CE
= , avem ,
1 1 t
v tq u
r
v r u p
AC
+
+
=
+
+
de unde
1
1 1
p
r t
=
+ +
,
1 1
r tq
r t
=
+ +
.
Obinem
r
t
pq
= i
( ) 1
1
q p
r
q

=

. nlocuind n 2 AC AM =
uuur uuuur
obinem

( )
( ) ( )
1
1 1
2 1
AM p q u q p v
pq
(
= +

uuuur r r
.
Din NP NA AP = + =
uuur uuur uuur
( ) ( )
1
1 1
2
p u q v
(
= +

r r
,
rezult
( ) 2 1
pq
MP MA AP
pq
= + =

uuur uuur uuur


( ) ( )
( )
1 1
2 1
pq
p u q v NP
pq
(
+ =


r r uuur
,
prin urmare punctele M, N, P sunt coliniare .

Observaie Rezultatul demonstrat mai sus mai are i urmtorul enun :
Mijloacele diagonalelor unui patrulater complet sunt coliniare. (punctele se
afl pe dreapta lui Gauss).

R7.1.4. S se arate c punctele
1 2 3
, , A A A sunt coliniare dac i numai dac
pentru orice punct M din plan (sau din spaiu ) exist numerele reale
1 2 3
, , m m m
nu toate nule , astfel nct
0
3 3 2 2 1 1
= + + MA m MA m MA m ,
1 2 3
0 m m m + + = (*)
Soluie. Fie punctele colineare
1 2 3
, , A A A i M un punct arbitrar . Exist
{ } *\ 1 astfel ca
1 2 1 3
A A A A =
uuuuur uuuur
(1) Dar
1 2 2 1
A A MA MA =
uuuuur uuuur uuuur
,
1 3 3 1
A A MA MA =
uuuur uuuur uuuur
i (1) devine ( ) +
2 1
1 MA MA . Lund
1
1 m = ,
2
1 m = ,
3
m = , relaia este demonstrat.
Reciproc, fie punctele
1 2 3
, , A A A cu proprietatea c pentru orice punct M
din plan (sau spaiu) exist numerele
1 2 3
, , m m m , nu toate nule astfel nct
au loc relaiile (*). Trebuie s artm c
1 2 3
, , A A A sunt coliniare.
Unul din numerele
1 2 3
, , m m m este nenul , fie
2
0 m
Din (*) , putem scrie
2 3 1 2 2 3 3
( ) 0 m m MA m MA m MA + + + =
uuuur uuuur uuuur r
, sau
0
3 1 3 2 1 2
= + A A m A A m (2) . Dac
3
0 m = , ar rezulta
2
0 m = sau
1 2
A A = ,
imposibil , deci
3
0 m .
78
Artm c
3 2
m m . Dac
2 3
m m = , atunci
( ) 2 1 2 1 3
0 m A A A A =
uuuuur uuuur r
de
unde rezult
2 3
A A = , imposibil , aadar
3 2
0 m m . Avem atunci
{ }
3
2
* 1
m
m
i relaia (2) se scrie
3
1 2 1 2
2
m
A A A A
m
=
uuuuur uuuuur
, deci punctele
1 2 3
, , A A A sunt coliniare.


79
8. Metoda geometric n rezolvarea problemelor de algebr

O cale deosebit de elegant de aflare a soluiei n cazul unor probleme
de algebr, este cea geometric. Formula distanei dintre dou puncte n plan i
spaiu, teorema cosinusului, exprimarea n diferite moduri a unei arii sau a unui
volum, sunt cteva dintre ideile care pot conduce la probleme interesante de
algebr. Frumuseea acestor probleme const n simplitatea i naturaleea
soluiei care de cele mai multe ori nu se ntrevede la prima vedere.
Vom prezenta cteva probleme de algebr n care apar inegaliti,
probleme de maxim i minim, ecuaii funcionale i sisteme de ecuaii rezolvate
prin metoda geometric.




Bibliografie

L.Panaitopol, V.Bndil, M. Lascu, Inegaliti, Ed.GIL, Zalu, 1996
V.Tudor, Probleme de algebr cu rezolvri ingenioase, Ed. Carminis,
Piteti, 1999, pag 12-18
M. Prajea, Demonstrarea unor inegaliti prin metode geometrice,
G.M. 1/2001, pag 12-18.
G. Csongor, S. Groznev, I. Kortezov, Best Practices in Education,2002



80
Probleme rezolvate (8.1)

A. Inegaliti

R8.1.1. S se demonstreze c
( ) , 2
2 2 2 2 2 2
c b a a c c b b a + + + + + + + c b a , , R

Soluie. Dac a, b, c sunt pozitive putem considera un model geometric sugerat
de expresiile din primul membru care sunt distanele i de membrul doi n care
apare diagonala unui ptrat de latur a+b+c. Considerm aadar ptratul ABCD
cu latura a+b+c ca n figura 8.1


C
B
A
D
b a c
a
c
b
c a b
b
c
a
N
P
M
Q

Fig. 8.1

Avem ,
2 2
c b BM + =
2 2
c a MN + = , ,
2 2
b a ND + =
2 ) ( c b a BD + + = . Inegalitatea de demonstrat este echivalent cu
BM+MN+NDBD, care este evident.
Observaii: Pornind de la figura putem deduce i alte inegaliti.
BP+PDBD 2 ) ( ) ( ) (
2 2 2 2
c b a b a a c b c + + + + + + +
BN+NDBD 2 ) ( ) ( ) (
2 2 2 2
c b a b a c b c a + + + + + + +

AQ+QP+PCAC 2 ) ( 2 2
2 2 2
c b a c a b + + + +

Dac unul dintre numerele a, b, c este negativ, de exemplu c0, aplicm
modelul de mai sus numerelor a, b, -c i avem

81
, 2 ) ( 2 ) (
2 2 2 2 2 2
c b a c b a a c c b c a + + + + + + + + deci
inegalitatea este adevrat pentru orice numere reale a, b, c, cu egalitate
a=b=c.
Inegalitile de mai sus admit i generalizri.

R8.1.2. S se demonstreze inegalitatea lui Minkowski.
2
2 1
2
2 1
2 2 2
2
2
2
2
1
2
1
) ... ( ) ... ( ...
n n n n
b b b a a a b a b a b a + + + + + + + + + + + +
n i R b a
i i
, 1 , , = .

Soluie:
Membrul nti ne sugereaz o sum de segmente, iar membrul doi,
diagonala unui dreptunghi. Considerm dreptunghiul de laturi
n
a a a + + + ...
2 1

i
n
b b b + + + ...
2 1
ca n figura 8.2

D C

b
n
M
n-1


M
2

b
2
M
1

b
1


A a
1
a
2
a
n
B
Fig. 8.2
Se formeaz n
2
dreptunghiuri de dimensiuni a
i
i b
j
, i,j=1,n. Dintre acestea
considerm dreptunghiurile de dimensiuni a
i
i b
j
, i,j=1,n ale cror diagonale
sunt n i b a
i i
, 1 ,
2 2
= + . Diagonala dreptunghiului ABCD este
2
2 1
2
2 1
) ... ( ) ... (
n n
b b b a a a AC + + + + + + + =
Inegalitatea de demonstrat este echivalent cu
AM
1
+M
1
M
2
++M
n-1
AC, care este evident. Egalitatea are loc dac i numai
dac toate dreptunghiurile formate sunt asemenea cu ABCD,
adic n i
AD
AB
b
a
i
i
, 1 , = =

82
Observaii:
1. Fcnd un raionament analog pentru diagonala BD, obinem
2 2
2 1
2 2 2
2
2
2
2
1
2
1
) ... ( ... + + + + + + + + + +
n n n
a a a b a b a b a
) ... (
2 1 n
b b b + + + +
2. Dac ABCD este ptrat, inegalitatea devine

= = =
= +
n
i
i
n
i
n
i
i i i
b a b a
1 1 1
2 2
) ( 2 ) ( 2
3. Modelul folosete numere a
i
, b
i
pozitive, celelalte situaii se deduc uor din
prima.
R8.1.3. S se arate c dac a, b, cR
+
, are loc inegalitatea
2 3 3 3
2 2 2 2 2 2
a ac c a bc c b ab b a + + + + +
Soluie:
Expresiile de sub radicali ne sugereaz folosirea teoremei cosinusului.
Considerm modelul geometric din figura, n care OA=OD=a,
OB=b, OC=c, . 30 ) ( ) (
0
= = COD m BOC m Din teorema cosinusului avem:
3
2 2
ab b a AB + = 3
2 2
bc c b BC + = 3
2 2
ac c a CD + =
Scriind AB+BC+CDAD obinem inegalitatea cerut. Egalitate se obine dac
B(AD) i C(AD). n acest caz avem b=c i teorema sinusurilor aplicat n
ABO, obinem relaia ) 1 3 ( = a b , prin urmare inegalitatea devine
egalitate ) 1 3 ( = = a c b .
n exemplul urmtor, expresiile de sub radical ne sugereaz un model spaial.
O
A
B
C
D
a
a
b
c

fig. 8.3

R8.1.4. S se arate c oricare ar fi , , R y x
i i
, , 1 n i = are loc inegalitatea:

= = = =
|
.
|

\
|
+ |
.
|

\
|
+ |
.
|

\
|
+ +
n
i
n
i
i
n
i
i
n
i
i i i i
z y x z y x
1
2
1
2
1
2
1
2 2 2


83
Soluie:
Considerm un paralelipiped dreptunghic cu un vrf n originea unui sistem de
axe ortogonale n spaiu, iar axele OX, OY, OZ, de-a lungul celor trei muchii ce
pleac din O. Considerm pe OX punctele x
1
,x
2
,,x
n
, pe OY punctele
y
1
,y
2
,,y
n
iar pe OZ punctele z
1
,z
2
,,z
n
. Ducnd prin punctele x
i
, n i , 1 = plane
paralele cu YOZ, prin punctele y
i
, n i , 1 = plane paralele cu XOZ, iar prin
punctele z
i
, n i , 1 = , plane paralele cu XOY, se formeaz o reea spaial de
paralelipipede. Exprimnd faptul c diagonala din O a paralelipipedului este
mai mic dect suma diagonalelor paralelipipedelor de dimensiuni x
i
,y
i
,z
i
,
n i , 1 = , obinem inegalitatea cerut.
Comentariu:
Pentru inegalitatea
+ + + + + + + + + + + +

2
2
2
1
2 2
1
2 2
1
2
4
2
3
2
2
2
3
2
2
2
1
... a a a a a a a a a a a a
n n n

( ) 3 ...
2 1 n
a a a + + +
cu egalitate dac i numai dac a
1
= a
2
== a
n
0 putem folosi ca model de
pornire un cub i parcurgem un raionament similar cu cel prezentat n
problema R - 8.1.4.

Exemplele urmtoare folosesc ca model geometric noiunile de arie i
volum.
R8.1.5. Dac x, y, z, t[0,1], s se arate c
. 2 ) 1 ( ) 1 ( ) 1 ( ) 1 ( + + + z t y x x y t x
Soluie:
Pe laturile ptratului ABCD, de latur 1, considerm punctele M[AB],
N[BC], P[CD], Q[AD], astfel nct AM=x, BN=y, CP=z, DQ=t (vezi
figura).
Suma ariilor triunghiurilor AMQ, BMN, CNP i DQP este cel mult egal
cu aria ptratului ABCD, ceea ce se scrie echivalent:

2
) 1 (
2
) 1 (
2
) 1 (
2
) 1 ( z t y z x y t x

D 1-z z
C
1-y
y
N
B 1-x x
M
A
1-t
Q
t

84
Egalitatea se realizeaz pentru } { . ) 1 , 0 , 1 , 0 ( ), 0 , 1 , 0 , 1 ( ) , , , ( t z y x

R8.1.6. Fie ) 1 , 0 ( , , , t z y x . tiind c aceste numere variaz independent n
intervalul ( ) 1 , 0 , s se arate c
. 1 ) 1 )( 1 )( 1 ( ) 1 )( 1 ( ) 1 ( < + + + t u z t v y x uv xyz

Soluie:
Considerm cubul ABCDABCD, de latur 1 i punctele M(AB),
N(AA), P(AD), R(CC), S(BC), Q(CD), astfel nct AM=x, AN=y,
AP=BS=z, AQ=t, TC=v. Exprimm faptul c suma volumelor
paralelipipedelor ce conin vrfurile A, A, C i C este cel mult egal cu
volumul cubului, obinem tocmai inegalitatea cerut.
A
B
C
D
A
B
C
D
Q
Q
M
M
S
T
P
v
1-x x
z
t
1-z
1-t
1-v
u
1-u



B. Probleme de extrem

R8.1.7. S se arate c dac a, b, cR i exist relaia a
2
+b
2
=2(a+b); c
2
+d
2
+4=4(c+d), atunci

1. ) 2 2 4 ( 2 2 2 4 + + + + d c b a
2. ) 2 2 3 ( 4 ) ( ) ( 0
2 2
+ + d b c a .

85
Soluie:
Cele dou relaii din ipotez se scriu echivalent
, 2 ) 2 ( ) 2 ( ; ) 2 ( ) 1 ( ) 1 (
2 2 2 2 2 2
= + = + d c b a ceea ce ne sugereaz
s folosim un model geometric cu cercurile ) 2 , (
1 1
O C i
2 4 , 4 ), 2 , (
2 1
= = OC AB o C (vezi figura).

O
A
O
1
O
2
M(a,b)
B
C
M (c,d)
2



Deoarece ) 2 2 ( , 2 2 2
2
=

= A
OA OC
OA i ). 2 2 , 2 2 ( + + B
Avem OM
1
OO
2
0 a
2
+b
2
8.
Din ( ) 1 ... 4 0
2
2 2
+
+
= + b a
b a
b a .
Avem OAOM
2
OB OA
2
c
2
+d
2
OB
2

2 18 12 2 8 12
2 2
+ + d c . Din
4
4
2 2
+ +
= +
d c
d c
). 2 ...( 2 2 4 2 2 4 + + d c
Adunnd (1) cu (2) obinem
. 2 2 8 2 2 4 + + + + d c b a


86
2. ) 2 , O ( C M , ) ( ) ( M M
1 1 1
2 2
2 1
+ = d b c a i ) 2 , O ( C M
2 2 2
.
Cele dou cercuri au dou puncte comune, deci min M
1
M
2
=0, maxM
1
M
2
=OB.
Deci 0 M
1
M
2
2
OB
2
0(a-c)
2
+(b-d)
2
4(3+2 2 )

R8.1.8.
a) Aflai valoarea minim a funciei
b) ( ) ( ) ( ) 16 4 1 4 ,
2 2 2
+ + + + + = y x y x y x f
c) Aflai pR astefel nct minimul funciei
d) ( ) ( ) ( ) 1 16 9 ,
2 2 2
+ + + + + = y p x y x y x g
S fie acelai cu minimul lui f(x,y)
Soluie:
a) Fie A(x,2), B(y,3), C(4,7), (vezi figura), deci
( ) 65 7 4 ,
2 2
= + = + + = OC BC AB OA y x f

Avem egalitate dac A,BOC


7 C 8 F
7 E


3 3 D
2

0 x y 4 0 x y p
Fig. Fig.
b) Fie D(x,3), E(y,7), F(p,8) (vezi figura). Atunci
( ) 64 ,
2
+ = + + = p OF EF DE OD y x g
Din
Obinem p=1. 65 64
2
= + p

R8.1.9. S se determine valoarea maxim i valoarea minim a expresiei
y x y x y x E 10 6 ) , (
2 2
+ = dac 0 2
2 2
+ y y x .
7
12
,
7
8
7
4
3 2
= = = = y x
y x

87
Soluie:
Condiia 0 2
2 2
+ y y x se mai scrie 1 ) 1 (
2 2
+ y x , deci punctele
M(x,y) se afl n interiorul discului de centru C(0,1) i raz R=1. Dac
notm A(3,5) atunci
. 34 34 ) 5 ( ) 3 ( ) , (
2 2 2
= + = MA y x y x E
Deci punctele situate n discul D(C,1) la distan minim i maxim sunt
situate pe cerc la intersecia cu dreapta AC. Avem AC=5 i deci AP=5-1,
AQ=5+1 (vezi figura). Obinem E
max
=2 i E
min
=-18





5 A




0 3
Fig.

C. Ecuaii funcionale

R8.1.10. Determinai funciile care verific ecuaia funcional
. ) ( ), ( ) ( ) ( R x y f x f y x f + = +
Soluie: Relaia dat se scrie echivalent
( ) ( )
( )
R z x
x y x
x f y x f
=
+
+
, , 1 ( ) 1
Reamintim c o condiie a punctelor ) , ( ); , (
2 2 2 1 1 1
y x M y x M ; ) , (
3 3 3
y x M fie
coliniare este
3 1 2 1
M M M M
m m = care este echivalent cu
1 3
1 3
1 2
1 2
x x
y y
x x
y y



(am notat
AB
m panta dreptei AB). Dac n (1) n locul lui x lum k, kZ,
obinem
( ) ( )
( )
R y x Z k
kx y kx
kx f y kx f
=
+
+
, , , 1
Obinem c punctele ( ) ) ( ,
1
y x f y x M + + , ( ) ) ( ,
2
x f x M , ( ) ) ( ,
jk
kx f kx M sunt
coliniare i aparin unei drepte de pant m=1. Prin urmare b x x f + = ) ( , xR,
care verific ecuaia funcional.

88
D. Sisteme de ecuaii

R8.1.11. S se rezolve sistemul

( ) ( ) ( ) ( ) ( ) ( )

< > = + = +
= + + + + + + + +
o y o t z x t y z x
t y t z y x z x
, , , ; 2
6 1 1 1 1
2 2 2 2
2 2 2 2 2 2

Soluie:
Prima ecuaie a sistemului ne sugereaz considerarea urmtoarelor
puncte: ) 1 , 1 ( ); 1 , 1 ( ); , ( ); , ( D C t y B z x A . Din a doua i a treia relaie rezult c
punctele se afl pe cercul 2 , 0 ( C (vezi figura). Avem
( ) ( ) ( ) ( ) ( ) ( )
2 2 2 2 2 2
1 1 , , 1 1 + + + = + = + + = t y AB t z y x BC z x AD
CD=2=latura ptratului (l
4
) nscris n 2 , 0 ( C ). Dar patrulaterul inscriptibil de
perimetru maxim nscris ntr-un cerc dat este ptratul. Din ipotez avem c se
atinge maximul perimetrului, deci ABCD ptrat. innd cont de egalitile a
doua i a treia din ipotez, soluiile sistemului sunt (1,-1,1,1), (-1,-1,-1,1).

x
y
O
B(y,t)
C(-1,-1)
A(x,z)
D(1,-1)


Fig.

n exemplul urmtor, modelul geometric l constituie un triunghi n care se ia un
punct interior. Esenial este faptul c pentru unghiurile sub care se vd laturile
triunghiului, din acest punct interior, se cunosc valorile funciei trigonometrice
cosinus. O soluie algebric n acest caz necesit eforturi mari, nu ntotdeauna
ncununate de succes.

89

R8.1.12. Numerele x, y, zR
+
verific urmtoarele relaii:
(i) x
2
+xy+y
2
=9
(ii) y
2
+yz+z
2
=16
(iii) z
2
+xz+x
2
=25
S se calculeze valoarea expresiei xy+yz+zx.
Soluie:
Prima condiie este echivalent cu existena unui triunghi cu laturile x,y,3 i
unghiul dintre x i y de 120
0
(teorema cosinusului). Un raionament similar
pentru celelalte dou relaii ne sugereaz urmtorul model (vezi figura), n care
ABC are laturile 3,4 i 5 iar unghiurile cu vrful n T au msurile de 120
0
.

120
120
T
A
B
C
x
z
y
4
3
5


Fig.
Avem
( ) ( ) zx yz xy zx yz xy S
ABC
+ + = + + =
4
3
120 sin 120 sin 120 sin
2
1
0 0 0

Pe de alt parte, triunghiul ABC fiind dreptunghic, 6
2
4 3
=

=
ABC
S . Deci
xy+yz+zx= 3 8 .
Comentariu: Punctul T cu proprietatea

m( ATB)=m( BTC)=m( CTA)= 120
0

exist pentru orice triunghi care are unghiurile mai mici de 120
0
i se numete
punctul lui Torricelli. Acest punct realizeaz minimul sumei TA+TB+TC i se
afl la intersecia cercurilor circumscrise triunghiurilor echilaterale construite
pe laturile triunghiului ABC, n exterior.

90
9. Funcii speciale

ntr-o expunere fcut de Euler n anul 1749 se menioneaz de mai
multe ori funcia ca o mrime variabil care depinde de o alt mrime variabil.
Pentru unele scopuri, o astfel de definiie a funciei este suficient. n
dezvoltarea ulterioar a matematicii s-a impus necesitatea de a da noiunii de
funcie un coninut mai general i mai abstract. Nu dependena variabilelor (prin
care de obicei se neleg numere care pot fi comparate n ceea ce privete
mrimea) este esenial n coninutul noiunii de funcie, ci corespondena prin
care anumitor obiecte li se ataeaz alte obiecte. n felul acesta, noiunea de
funcie se fundamenteaz pe noiuni ale teoriei mulimilor. n consideraiile ce
urmeaz se consider funcii care nu sunt nominalizate n trunchiul comun, ns
care faciliteaz rezolvarea unor probleme de teoria mulimilor sau teoria
numerelor.


9.1. Definiii i proprieti ale funciilor speciale

9.1.1.Definiie: Fie E o mulime nevid fixat i E A . Aplicaia
E f
A
: R, ( )

=
A E x
A x
x f
A
\ , 0
, 1
, se numete funcia caracteristic mulimii
A.

9.1.2.Teorem: Fie A, B submulimi ale unei mulimi E.
Atunci A = B f
A
= f
B
.

Demonstraie: Dac mulimile sunt egale, din definiia 9.1.1.
deducem imediat f
A
= f
B
. Reciproc, presupunem f
A
= f
B
i vom demonstra
identitatea A = B prin dubl incluziune. Fie A x , deci ( ) ( ) 1 = = x f x f
B A
, de
unde obinem B x ; astfel B A . Fie B x , deci ( ) ( ) 1 = = x f x f
B A
, de
unde obinem A x ; astfel . A B

9.1.3. Observaie: A demonstra egalitatea a dou mulimi este
echivalent cu a demonstra c funciile lor caracteristice sunt egale.

9.1.4. Teorem: Fie B A, P (E). Atunci
91
( )
B A B A B A
B A B A
B A B A
B A B A B A
A
A
f f f f f
f f f
f f f
f f f f f
f f
2 . 5
1 . 4
. 3
. 2
1 . 1
\
+ =
=
=
+ =
=

I
U

Demonstraie:
1. Pentru E x sunt posibile doar situaiile:
(i) A x , deci ( ) 1 = x f
A
; aceasta implic A x , deci ( ) 0 = x f
A
. Astfel
identitatea este verificat.
(ii) A x , deci ( ) 0 = x f
A
, ceea ce implic A x , deci ( ) 1 = x f
A
. Astfel
formula 1 este verificat i n acest caz.
2. Fie E x i pentru sistematizarea demonstraiei prezentm datele pe
cazuri:
1. A x i B x , B A x U : ( ) x f
B AU
=0 ( ) ( ) ( ) ( ) x f x f x f x f
B A B A
+ = 0
2. A x i B x , B A x U : ( ) x f
B AU
=1 ( ) ( ) ( ) ( ) x f x f x f x f
B A B A
+ = 1
3. A x i B x , B A x U : ( ) x f
B AU
=1 ( ) ( ) ( ) ( ) x f x f x f x f
B A B A
+ = 1
4. A x i B x , B A x U : ( ) x f
B AU
=1 ( ) ( ) ( ) ( ) x f x f x f x f
B A B A
+ = 1
Deducem E x , ( ) ( ) ( ) ( ) ( ) x f x f x f x f x f
B A B A B A
+ =
U
.
3. Se demonstreaz analog, lundu-se n discuie aceleai patru cazuri.
4. Putem utiliza identitatea B A B A I = \ i vom aplica succesiv
proprietile 3 i 1; obinem ( )
B A
B
A
B A
B A
f f f f f f = = = 1
\
I
.
5. Prin definiie ( ) ( ) A B B A B A \ \ U = . Utilizm proprietile 2 i 4. Astfel
B A
f

=
A B B A A B B A
f f f f
\ \ \ \
+ =
= ( ) ( ) ( ) ( )
A B B A A B B A
f f f f f f f f + 1 1 1 1 =
B A B A
f f f f + 2 , cci
pentru orice mulime E M i orice numr natural 0 > k ,
M
k
M
f f = .

9.1.5. Exerciii:
1. Artai c diferena simetric este asociativ.
Soluie: Va trebui s artm c ( ) ( ) C B A C B A = , C B A , , P
(E).Conform teoremei 9.1.2. este suficient s artm c
( ) ( ) C B A C B A
f f

= , iar
pentru aceasta utilizm formula 5 din teorema 9.1.4.
( ) C B A
f

=
C B A C B A
f f f f +

2 =
= ( )
C B A B A B A B A
f f f f f f f f f + + 2 2 2 =
92
= ( )
C B A C B C A B A C B A
f f f f f f f f f f f f + + + + + 4 2 .
Calculnd
( ) C B A
f

se obine acelai rezultat.

2. Studiai injectivitatea funciei caracteristice unei mulimi A, E A ,
card(E) 1.
Soluie: Fie A P(E), E f
A
: R. Dac E are un element, atunci P(E)
={, E}. Funciile f

, f
E
sunt injective. Dac E are dou elemente, E = {a,
b}, atunci P(E)= {, {a}, {b}, E}. Deducem f
{a}
i f
{b}
sunt injective. Fie card (E)
3. Pentru orice submulime A a lui E, Im f
A
{0, 1} nu are mai mult de dou
elemente, n schimb domeniul de definiie al funciei are cel puin 3 elemente;
astfel f
A
nu poate fi injectiv. n concluzie, funcia caracteristic unei mulimi
E A este injectiv n ipotezele
(i) card (E) = 1
(ii) card (E) = 2 i card (A) = 1.

3. Fie A, B submulimi disjuncte ale lui E. Determinai X P(E) nct
( ) X B X B A U U = \ .
Soluie: Prin utilizarea funciei caracteristice, ecuaia dat se poate scrie
( ) ( )
X B B A X B B A
f f f f f f f f + =
X B X B
f f f f + .
Din A B = deducem 0 =
B A
f f . Dup reducerea termenilor asemenea
obinem
X A
f f = , adic X = A, soluie unic a ecuaiei.

4. Fie A, B submulimi disjuncte ale lui E. Determinai X, Y P(E) astfel
nct

=
=
=
B Y X
A Y X
E Y X
\
I
U

Soluie: Aplicnd funcia caracteristic i proprietile acesteia sistemul
devine:

=
=
= +
B Y X X
A Y X
Y X Y X
f f f f
f f f
f f f f 1


nlocuind relaia a doua n relaia a treia i utiliznd faptul c A
B = , ceea ce nseamn 0 =
B A
f f .
93
Vom avea
B A B A B A B A
f f f f f f f
U
= + = + i astfel relaia a treia
conduce la
B A X
f f
U
= , adic B A X U = . nlocuim relaia a doua n prima i
utilizm soluia gsit pentru X care implic
B A X
f f f + = . Dup reducerea
termenilor asemenea se obine
B Y
f f =1 , adic
B
Y
f f = , ceea ce
nseamn B Y = . Soluia sistemului este ( ) B B A , U .

9.1.6. Definiie: Funcia f : R Z, dat de legea ( ) | | x x f = , unde | | x
reprezint cel mai mare ntreg mai mic dect x, se numete funcia parte
ntreag.

9.1.7. Observaie: Au loc inegalitile | | | | 1 + < x x x .

9.1.8. Definiie: Funcia f : R | ) 1 , 0 dat de legea ( ) | | x x x f = se
numete funcia parte fracionar.

9.1.9. Proprieti ale funciei parte ntreag:
Fie f : R Z, ( ) | | x x f = . Atunci au loc proprietile:
1. ( ) ( ) ( ) y f x f y x f + + , x, y R ;
2.
| |
|
.
|

\
|
= |
.
|

\
|
n
x
f
n
x
f , n Z*, x R;
3. ( ) ( ) x f x f x f = |
.
|

\
|
+ 2
2
1
, x R;
4. ( ) ( ) 0
1
...
2 1
= |
.
|

\
|
+ + + |
.
|

\
|
+ + |
.
|

\
|
+ + nx f
n
n
x f
n
x f
n
x f x f ,
x R, n Z*.

Demonstraie:
1. Se scrie inegalitatea sub forma: | | | | | | y x y x + + . Ori pentru x,
y R avem | | { } x x x + = , | | { } y y y + = , deci | | | | { } { } y x y x y x + + + = + . Cum
{ } { } 0 + y x , obinem | | | | + y x Z i | | | | | | y x y x + + . Dar | | y x + este cel mai
mare ntreg ce nu depete pe x + y. Deci | | | | | | y x y x + + .
94
2. Egalitatea se mai scrie:
| |
(

=
(

n
x
n
x
. Plecm de la | | { } x x x + = i de
la | | r qn x + = , 1 0 n r cnd obinem
| |
n
r
q
n
x
+ = sau
| |
q
n
x
=
(

, ceea ce
ncheie demonstraia proprietii.
3. Are forma | | | | x x x =
(

+ 2
2
1
cu | | { } x x x + = . Avem cazurile:
a) { }
2
1
0 < x . Avem | | { }
2
1
2
1
+ + = + x x x i deci | | x x =
(

+
2
1
,
| | | | { } | | | | x x x x 2 2 2 2 = + = .
b) { } 1
2
1
< x . Cum | | { }
2
1
2
1
+ + = + x x x avem: | | 1
2
1
+ =
(

+ x x ;
| | | | { } | | x x x 2 2 2 + = = | | 1 2 + x i relaia 3 este verificat.
4. Ca i n celelalte cazuri rescriem proprietatea cu ajutorul simbolului [
] i avem: | | | | 0
1
...
2 1
=
(


+ + +
(

+ +
(

+ + nx
n
n
x
n
x
n
x x . Aceast egalitate
este cunoscut sub numele de identitatea lui Hermite. Notm {x} = a, deci x
=[x] + a. Partiionm intervalul [0,1] n n subintervale , mai exact fie
| )
U
1
0
1
, 1 , 0

=
(


=
p
k
n
k
n
k
Deoarece | ) 1 , 0 a , va exista k {0, 1,, n 1},
nct 1
1

+
<
n
k
a
n
k
n k a n k + < 1 .Atunci
=
(

+
n
i
x = | |
(

+ +
n
i
a x = | |
(

+
+
n
i n a
x . Pentru primele n k valori
ale lui i i anume i = 0, 1,, n k 1 deducem 1 0 <
+

n
i p n
, adic
0 =
(

+
n
i p n
. Folosind aceasta va rezulta ( ) | |


=
=
(

+
1
0
k n
i
x k n
n
i
x . Pentru
urmtoarele k valori ale lui i, i anume ( ) 1 , 1 , + + = k k n k n k n i
deducem:
n
n k
n
i a n +
<
+
1 =
n
k
+ 1 , adic 1 =
(

+
n
i a n
. Folosind aceasta
avem | |

=
+ =
(

+
1 n
k n i
k x k
n
i
x . Prin urmare | |

=
+ =
(

+
1
0
n
i
k x n
n
i
x i pe de alt
95
parte | | | | | | | | | | | | k x n a n x n a n x n x n + = + = + = , deoarece | | k a n = , deci

=
(

+
1
0
n
i
n
i
x =| | x n .

9.1.10. Definiie: Funcia sgn : R { 1, 0, 1} definit prin

<
=
>
=
0 , 1
0 , 0
0 , 1
sgn
x
x
x
x
se numete funcia signum (funcia indicator de semn).

9.1.11. Proprieti ale funciei signum
1. Funcia signum este surjectiv, dar nu este injectiv.
2. Funcia signum este impar.
3.Pentru orice (x, y) R R avem: ( ) y x y x sgn sgn sgn = .

9.1.12. Observaie: Funcia signum mai poate fi definit astfel:
sgn : R R,

=
0 , 0
0 ,
sgn
x
x
x
x
x .

9.1.13. Definiii:
(1) Funcia f : R R R, definit prin ( )

<

=
y x y
y x x
y x f
,
,
, se numete
funcie maximum din x i y (prescurtat, funcie max) i se noteaz
( ) { } y x y x f , max , = .
Altfel , funcia max se mai scrie ( )
2
,
y x y x
y x f
+ +
= .
(2) Funcia f : R R R, definit prin ( )

>

=
y x y
y x x
y x f
,
,
, se numete
funcie minimum dintre x i y (funcie min) i se noteaz ( ) { } y x y x f , min , = .
Funcia minimum se mai poate scrie { }
2
, min
y x y x
y x
+
= .

9.1.14. Proprieti ale funciilor maximum i minimum:
(1) Funciile max, min nu sunt injective, dar sunt surjective.
96
(2) Fie funcia ( ) y x f , , unde | | I b a x = , , | | J d c y = , . Atunci au loc
relaiile:
a) ( ) ( ) y x f y x f
J y I x J y I x
, max max , min min

;
b) ( ) ( ) y x f y x f
I x J y I x J y
, max max , min min

.

Demonstraie:
(2) a) Pentru I x , x fixat, fie ( ) ( ) J y y x f y g = , , : Avem
( ) ( ) y g y g
J y J y
max min ( ) ( ) y x f y x f
J y J y
, max , min

. Dar ( ) y x f
J y
, min

este un
numr ce depinde de valoarea fixat pentru x. Fie ( ) ( ) y x f x h
J y
, min

= i
( ) ( ) y x f x l
J y
, max

= . Atunci ( ) ( ) I x x l x h , dac i numai dac


( ) ( ) x l x h
I x I x
max max .
Dar ( ) ( ) x l x h
I x I x
min max , deci ( ) ( ) ( ) x l x h x h
I x I x I x
max max min , i deci
( ) ( ) y x f y x f
J y I x J y I x
, max max , min min

.

9.1.15. Definiii i proprieti (alte funcii):

(1) Funcia f : R R, ( ) ( )
| | { }
| | { }

|
.
|

+
|
.
|


= =
1 ,
2
1
, 1
2
1
, 0 ,
x x x
x x x
x x f se numete
funcia distan la cel mai apropiat ntreg.
Funcia distan la cel mai apropiat ntreg este o funcie par, mrginit,
(

=
2
1
, 0 Im f , periodic, cu perioada principal 1
0
= T .
(2) Funcia f : R {0, 1}, f (x) = T (x) =

<
1 , 1
0 , 0
x
x
se numete funcia
lui Heaviside sau funcia treapt unitate.
Este funcia caracteristic a mulimii R
+
.
(3) Funcia f : R {0, 1} definit prin ( )

=
Q R
Q
\ , 0
, 1
x
x
x f se numete
funcia lui Dirichlet.
97
Este o funcie par, mrginit, surjectiv, care admite ca perioad orice numr T
Q*. Funcia lui Dirichlet este funcia caracteristic a mulimii Q.

9.2. Aplicaii ale funciilor speciale

Probleme rezolvate

R9.2.1 Fie A, B, C mulimi cu proprietatea ( ) ( ) A C B B C A \ \ U U = .
Determinai B A .
Soluie: Conform teoremei 9.1.2. vom avea
( ) ( ) A C B B C A
f f
\ \ U U
= . Calculm
fiecare membru utiliznd formulele teoremei 9.1.4:
( ) B C A B C A B C A
f f f f f
\ \ \
+ =
U
=
C B A C A B C C A
f f f f f f f f f + .
( ) A C B A C B A C B
f f f f f
\ \ \
+ =
U
=
C B A C B A C C B
f f f f f f f f f + + .
Ipoteza conduce la f
A
= f
B
, adic A = B, ceea ce implic B A = .

R9.2.2 S se demonstreze cu ajutorul funciei caracteristice c dac
X B X A U U = i X B X A I I = , atunci A = B.
Soluie: Avem:
( ) ( ) ( ) ( ) ( ) ( ) ( ) ( ) x f x f x f x f x f x f x f x f
X B X B X A X A
+ = + i
( ) ( ) ( ) ( ) x f x f x f x f
X B X A
= . Din aceste relaii rezult ( ) ( ) x f x f
B A
= , deci A
= B.

R9.2.3 S se arate c pentru orice x > 0 i n N, avem:
( )
| |
n
n
k
kx
x n
I
1
2
1
=

+
.
Soluie: Avem x > 0 i k N, de unde rezult | | 0 kx i se tie c:
| |
| |
n
n
k
n
k
kx
n
kx
I
1
1
=
=

.
Dar | | kx kx , rezult
| |
n
kx
n
kx
n
k
n
k

= =

1 1
.
Deci | |
n
n
k
n
k
kx
n
kx
I
1
1
=
=


( )
| |
n
n
k
n
k
kx
n
n n
x
n
k x
I
1
1 2
1
=
=

+
=


( )
| |
n
n
k
kx
x n
I
1
2
1
=

+
.

98

R9.2.4 S se arate c pentru orice x > 0, avem:
| |
( )
| |

=
+
+ +
n
k
x
n n
n kx
1
2
1
1 .
Soluie:
| | | | | | | |

=
+ + + + + + = +
n
k
nx x x kx
1
1 ... 2 1 1 1 . Dar | | | | | | | | x k kx kx + + = + 1 1 1 .
Deci | | | |
( )
| |

= =
+
+ = + +
n
k
n
k
x
n n
n k x n kx
1 1
2
1
1 .

R9.2.5 S se determine exponentul numrului 7 n numrul 90!.
Soluie: tim c exponentul unui numr prim p n numrul n! este egal
cu ...
3 2
+
(

+
(

+
(

p
n
p
n
p
n
n cazul nostru exponentul cutat este:
13 0 1 12 ...
7
90
7
90
7
90
3 2
= + + = +
(

+
(

+
(

.

R9.2.6 Dac a este un numr real, notm { } | | a a a = partea sa fracionar.
Fiind dat m ( ) 2 m un numr natural, s se arate c funcia f : N R,
definit prin ( ) { } 2 =
n
m n f este injectiv.
(C. Ni)
Soluie: Presupunem prin absurd c funcia f n-ar fi injectiv, adic ar
exista
2 1
x x , x
1
, x
2
N, astfel ca ( ) ( )
2 1
x f x f = . Aceasta din urm este
echivalent cu:
{ } { } 2 2
2 1
=
x x
m m sau | | | | 2 2 2 2
2 2 1 1
=
x x x x
m m m m sau
| | | |
2 1
2 1
2 2
2
x x
x x
m m
m m


= , deci 2 Q, fals. Prin urmare f este injectiv.

R9.2.7 S se arate c singura funcie f : R R cu proprietatea c:
( ) ( ) ( ) ( ) ( ) y x f y x f y x f , min , max + = + , y x, R este funcia identic.
Soluie: Schimbnd pe x cu y n relaia din enun obinem:
( ) ( ) ( ) ( ) ( ) + = + y x y x f x y f y x f , , , min , max R.
Adunnd aceast relaie cu cea din enun i folosind relaia
( ) ( ) + = + y x y x y x y x , , , min , max R
obinem:
99
( ) ( ) ( ) y f x f y x y x f + + + = + 2 , y x, R.
Pentru y = 0 avem ( ) ( ) + = x f x x f , 0 R. Fie ( ) a f = 0 , atunci relaia din
enun devine : ( ) ( ) + + + = + + y x x a y y a x a y x , , , min , max R . Pentru x =
0, y = a rezult ( ) 0 , max 0 + = a a , de unde a = 0 i deci ( ) = x x x f , R.

9.3. Ecuaii funcionale

9.3.1. Exemple de ecuaii funcionale

La fel ca noiunea de mulime, pentru noiunea de ecuaie funcional
este dificil de dat o definiie care s cuprind toate tipurile de ecuaii
funcionale. Vom nelege prin ecuaie funcional, o relaie de egalitate n care
apare una sau mai multe funcii necunoscute, care trebuie s satisfac relaia
identic pentru valori atribuite variabilelor dintr-o mulime dat.
Dintre ecuaiile funcionale clasice amintim, ca exemplu:
9.3.1.1. Ecuaia lui Cauchy
:
( ) ( ) ( ) , ,
f
f x y f x f y x y

+ = +


9.3.1.2. Ecuaia lui Jensen
:
( ) ( )
, ,
2 2
f
x y f x f y
f x y

+ + | |
=
|

\ .


9.3.1.3. Ecuaia general liniar
:
( ) ( ) ( ) , ,
f
f a x b y c A f x B f y C x y

+ + = + +


unde , , , , , a b c A B C .
9.3.1.4. Ecuaia lui dAlambert
:
( ) ( ) 2 ( ) ( ) , ,
f
f x y f x y f x f y x y

+ + =


9.3.1.5. Ecuaia lui Pexider
, , :
( ) ( ) ( ) , ,
f g h
f x y g x h y x y

+ = +


9.3.1.6. Ecuaia lui Hossz
:
( ) ( ) ( ) ( ) , ,
f
f x y xy f x f y f xy x y

+ = +



100
9.4. Probleme rezolvate (9)

Atribuirea unor valori particulare variabilelor

R9.3.2.1. S se decid dac exist un numr real a i o funcie : f cu
proprietatea ( ) ( ) f x f a x x + = , pentru orice x .
Soluie. Dac ar exista, nlocuim x cu a x obinem o nou relaie
( ) ( ) f a x f x a x + = , pe care comparnd-o cu prima, obinem x a x =
pentru orice
2
a
x x = , pentru orice x , care este fals, deci nu exist
a i f.
R9.3.2.2. S se rezolve ecuaia funcional
: ,
( ) ( ) ( ) , , .
f
f x y f x f y x y


Soluie.
2
0 (0) ( (0)) (0) {0,1} x y f f f = = = .
Dac (0) 0 f = , punnd n ecuaie 0 ( ) 0 y f x = = , x , deci 0 f = .
Dac (0) 1 f = , facem n ecuaie schimbrile 2 x x i y x , i
rezult
( ) (2 ) ( ) f x f x f x =
(2)
Dac exist a astfel ca ( ) 0 f a = , din ecuaie, pentru y a =
0 f = .
Dac ( ) 0 f x pentru orice x, din (2) rezult (2 ) 1 f x = , x deci
1 f = .
Singurele funcii care verific ecuaia sunt funciile constante 0 f = i 1 f = .
R9.3.2.3. Determinai toate funciile : f care satisfac relaia
2 2
( )( ( ) ( )) ( ) ( ) x y f x f y f x f y + = ,
pentru orice x i y .
Soluie. Pentru 0 y = i 1 y = obinem:
2
( ( ) (0)) ( ) (0) x f x f f x f = ,
2
( 1)( ( ) (1)) ( (1)) x f x f f x f + = ,
din care prin scdere:
( ) ( (1) (0)) (0) f x f f x f = + ,
deci ( ) , f x ax b x = + , unde a, b sunt constante reale arbitrare.
Se verific uor c toate aceste funcii satisfac relaia.

101
Substituii de funcii i schimbri de variabile

R9.3.2.4. S se rezolve ecuaia funcional
: ,
4 ( ) 3 ( ) 2 , .
f
f x f x x x


Soluie. nlocuim x cu x i obinem relaia: 4 ( ) 3 ( ) 2 f x f x x = ,
adic sistemul:
4 ( ) 3 ( ) 2
3 ( ) 4 ( ) 2
f x f x x
f x f x x
=

+ =


cu necunoscutele ( ) f x i ( ) f x .
nmulim prima relaie cu 4, a doua cu 3 i le adunm. Rezult:
7 ( ) 14 ( ) 2 , f x x f x x x = = .
R9.3.2.5. Fie : f o funcie care verific relaia
( )
2
3 3
( ) ( ) 1 3 ( ( )) ( ) f x a f x f x f x + + = + ,
pentru orice x , unde
*
a este o constant arbitrar. S se arate c
funcia f este periodic.
Soluie. Scriem relaia sub forma
( )
3
3 3 3
( ) 1 ( ) ( ) 1 ( ) f x a f x f x a f x + = + = .
Notnd
3
( ) ( ) g x f x = obinem:
( ) 1 ( ) g x a g x + = ,
( 2 ) 1 ( ) 1 (1 ( )) ( ) g x a g x a g x g x + = + = = ,
deci

3 3
( 2 ) ( ) ( 2 ) ( ) , f x a f x f x a f x x + = + = .
Funcia f are perioada 2 T a = .
R9.3.2.6. S se determine funciile : f i : g care pentru orice
x verific sistemul
( 6) 2 (2 15) 6 40,
2
( 5) 2 6.
2
f x g x x
x
f g x x
+ + + = +

+ | |
+ + = +
|

\ .

Soluie. Dac
2
6
2
y
x
+
= + atunci 5 2 15 y x + = + . nlocuind n relaia a
doua pe x cu 2 10 x + obinem sistemul
102
( 6) 2 (2 15) 6 40
( 6) (2 15) 4 26
f x g x x
f x g x x
+ + + = +

+ + + = +


(2 15) 2 14, ( ) 1, g x x x g x x x + = + = .
( 6) 2 12, ( ) 2 , f x x x f x x x + = + = .
R9.3.2.7. Fie a i b dou numere reale fixate. S se determine toate funciile
: f care verific relaia:
( ) (2 ) ( 2 ) , f x f a x x x a b x = + .
Soluie. Dac n locul lui x punem 2 x a x , obinem relaia:
(2 ) ( ) ( 2 ) f a x f x x x a b = + ,
care dac o adunm cu prima, obinem 0 b = , deci pentru 0 b nu exist
funcii care s verifice relaia dat.
Pentru 0 b = rmne o singur relaie:
( ) (2 ) ( 2 ) , f x f a x x x a x = .
Dac 0 0 x a = = , deci ( ) f a este arbitrar.
Dac 2 x a a x a < > , ntre valorile lui f pentru fiecare din
intervalele ( , ) a i ( , ) a + nu exist nici o relaie. Obinem:
( ) ,
( )
( 2 ) (2 ) ,
h x x a
f x
x x a h a x x a

=

>


unde : ( , ] h a este o funcie arbitrar.

Folosirea unor proprieti de monotonie, surjectivitate, injectivitate

R9.2.3.8. S se determine toate funciile : f i : g care
verific condiiile:
a) g este injectiv;
b) ( ( ) ) ( ( )) f g x y g x f y + = + , pentru orice x i y .
Soluie. Pentru 0 x = obinem: ( (0) ) ( ( )) f g y g f y + = , deci:
( ( )) ( ) , g f x f x b x = + unde (0) b g =
(1)
Pentru 0 y = , obinem:
( ( )) ( ) , f g x g x a x = + , unde (0) a f = .
(2)
Din (1) i (2) rezult
( ( )) ( ( )) ( )( ( )) g g x a g f x g f g x + = = = o

injectiv )
( ( ) ) ( ( )) ( ) ( )
g b
f g x b g x f b g x a x f b = + = + + = +


103

(2)
( ) ( ) g x x c f x c x a c = + + = + +
i obinem:
( ) , f x x a x = + i ( ) , g x x b x = + ,
unde a i b sunt parametri reali arbitrari.
R9.3.2.9. S se decid dac exist numerele naturale m, n i funciile
: f i : g astfel ca:
2 1
( )( )
m
f g x x
+
= o i
2 2
( )( )
n
g f x x
+
= o .
Soluie.
2 1 2 1
1 2 1 2 1 2 1 2
( ) ( ) ( ( )) ( ( ))
m m
g x g x f g x f g x x x x x
+ +
= = = = ,
deci funcia g este injectiv.
Avem:
2 1 2 2
(( )( )) ( )( ( )) ( ) ( ( ))
m m
g f g x g f g x g x g x
+ +
= = o o .
Pentru 0, 1 x x = = i 1 x = , obinem:
2 2 2 2 2 2
(0) ( (0)) , (1) ( (1)) , ( 1) ( ( 1))
n n n
g g g g g g
+ + +
= = =
deci
(0) {0,1}, (1) {0,1}, ( 1) {0,1} g g g ,
adic numerele (0) , (1) g g i ( 1) g nu sunt toate distincte. (Contradicie cu
injectivitatea funciei g). n concluzie, rspunsul este negativ.
R9.3.2.10. S se arate c nu exist funcii : f astfel ca:
- funcia ( ) (3 1) , g n f n n n = + s fie cresctoare, iar
- funcia ( ) (5 2) , h n f n n n = + s fie descresctoare.
Soluie. Dac prin absurd g ar fi cresctoare i h descresctoare atunci
funciile
1
( ) (5 2) g n g n = + i
1
( ) (3 1) h n h n = + ar fi cresctoare, respectiv
descresctoare, deci funcia
1 1 1
( ) ( ) ( ) f n g n h n = ar fi cresctoare. Dar:
1
( ) (15 7) 5 2 (15 7) 3 1 2 1 f n f n n f n n n = + + + + = ,
care este o funcie descresctoare
R9.3.2.11. S se determine funciile injective : f cu proprietatea
( )( ) ( ) 1 0, f f x f x x = o .
Soluie. Dac ( ) Im y f x f = atunci ( ) 1 f y y = + . Artm c
Im f = , deci ( ) 1, f y y y = + .
( f este surjectiv) i atunci unica soluie este funcia
( ) 1, f x x x = + .
Dac Im 1 Im y f y f + deci { , 1, , , } Im y y y n f + + K K .
Dac presupunem c Im f atunci n Im f ar exista un element
minim
0
Im y f .
104
Fie
0
x cu
0 0
( ) f x y = . Deoarece pentru
0
y y > avem ( 1) f y y =
rezult c
0
x y i pentru orice
0
x x < , ( ) Im f x f deci ( ) ( 1) f x y f y = =
cu 1 x y , deci f nu ar fi injectiv.

Folosirea unor inegaliti

R9.3.2.12. Fie : f o funcie bijectiv, strict cresctoare. S se
determine funciile : g pentru care
( )( ) ( )( ) , f g x x g f x x o o .
Soluie. Funcia invers
1
f

este i ea strict cresctoare
1 1 1
( ( )) ( ( ( ))) ( ) ( ) ( ) f g x x f f g x f x g x f x

.
Punem n relaia ( ( )) x g f x n loc de x pe
1
( ) f x

i avem:
1
( ) ( ) f x g x

. Din cele dou relaii avem:


1
( ) ( ) f x g x

. Din cele dou relaii


rezult
1
( ) ( ) g x f x

= , x , deci singura funcie este


1
g f

= .
R9.3.2.13. S se determine funciile : (0, ) (0, ) f cu proprietatea
1 1
( ) ( ) ( )
4 4
f ax by f x f y
a b
+ + ,
pentru orice numere pozitive a, b, x, y.
Soluie. Pentru
1
2
a = i
1
2
x
b
y
= obinem
1 1
( ) ( ) ( ) ( ) ( )
2 2
y
f x f x f y x f x y f y
x
+ .
Pentru
1
2
y
a
x
= i
1
( ) ( )
2
b y f y x f x = deci ( ) ( ) x f x y f y =
pentru orice x, y.
Rezult c ( ) ,
c
f x x
x
= unde (0, ) c este o constant arbitrar.
Avem:
1 1
( ) , ( ) ( )
4 4 4 4
c c c
f ax by f x f y
ax by a b a x b y
+ = + = +
+

i inegalitatea din enun este echivalent cu
2 1 1 1
2 ax by ax by
| |
+
|
+
\ .
care este
cunoscut (inegalitatea dintre media aritmetic i armonic).


105
Metode recursive i inductive

R9.3.2.14. S se determine funciile : f care ndeplinesc simultan
condiiile:
a) ( ) ( ) ( ) 2 f m n f m f n mn + = + + , oricare ar fi , m n ;
b) Pentru orice n , numrul ( ) f n este ptrat perfect.
Soluie. Pentru 0 m n = = , din a) rezult (0) 0 f = . Pentru 1 m n = = , din
a) rezult (2) 2 (1) 2 f f = + . Din b) exist , a b astfel ca
2 2
(1) , (2) f a f b = = . Dac a ar fi un numr par atunci numrul
2
2 2 a + ar fi de
forma 4 2 M + , deci nu poate fi ptrat perfect. Rmne c a este impar
2 1 a k = + , i b este un numr par 2 b p = i avem relaia:
2 2 2 2 2 2
(2 ) 2(2 1) 2 4 4 4 4 1 p k p k k p k k = + + = + + = + +
din care rezult p k > deci
2 2 2
1 2 1 1 p k p k k k k + + + > + + , dac 0 k .
Deci condiia
2 2
1 p k k = + + poate fi ndeplinit pentru 0 k = i
1 p = (1) 1 f = i
2
(2) 4 2 f = = .
Artm prin inducie c
2
( ) , f n n n = .
Avem: ( 1) ( ) (1) 2 f n f n f n + = + + i din ipoteza de inducie
2 2 2
( ) ( 1) 2 1 ( 1) f n n f n n n n = + = + + = + .
R9.3.2.15. S se determine funciile : f care satisfac condiiile
a) ( ( )) , f f n n n = ;
b) ( ( 2) 2) , f f n n n + + = ;
c) (0) 1 f = .
Soluie. ( ( )) , f f n n n f = este bijectiv. ( ( )) n f f n = =
( ( 2) 2) f f n = + +
( ) ( 2) 2 f n f n = + +
(1)
Din (1) pentru 0 (2) 1 n f = = i prin inducie
(2 ) 1 2 , f k k k = ;
( (2 )) 2 (1 2 ) 2 1 (1 2 ) , f f k k f k k k k = = = .
Deci, ( ) 1 , f n n n = este singura funcie care satisface cele trei condiii.




107
10.2. Probleme rezolvate (10)

R10.2.1. Inegalitatea lui Cauchy-Buniakovski-Schwarz. Fie numerele reale a
1
,
a
2
,, a
n
,b
1
, b
2
,, b
n
. Atunci:

Cu egalitate dac i numai dac p,qR astfel nct
i i
qb pa = pentru orice
i1,2,,n`
Demonstraie. Inegalitatea este verificat cu egalitate pentru
0
2 1
= = = =
n
a a a L Presupunem c exist i1,2,,n` cu a
i
diferit de zero.
Considerm funcia
Observm c f(x)0 pentru orice x R. Pe de alt parte avem

Cum
deducem f0 sau
care conduce la (1). Egalitatea n (1) este echivalent cu f=0. Atunci exist
rR, cu f(r)=0, rezult
i i
b r a = , pentru orice { } n i , , 2 , 1 K i numerele p,q din
enun sunt p=r, q=1.

R10.1.2. Inegalitatea lui Aczel. Fie a
1
, a
2
,, a
n
,b
1
, b
2
,, b
n
i a
1
2
>
a
2
2
++ a
n
2
. Atunci
(a
1
b
1
a
2
b
2
-- a
n
b
n
)
2
( a
1
2
- a
2
2
-- a
n
2
)(b
1
2
- b
2
2
-- b
n
2
)(2)
egalitatea obinndu-se dac i numai dac exist p,qR astfel nct pa
i
=qb
i

pentru orice i1,2,n`.
Demonstraie: Considerm funcia f:RR,
) 1 (
1
2
1
2
2
1
|
.
|

\
|
|
.
|

\
|
|
.
|

\
|

= = =
n
i
i
n
i
i
n
i
i i
b a b a
( )

=
=
n
i
i i
b x a x f R R f
1
2
) ( , :

= = = =
+ |
.
|

\
|
|
.
|

\
|
= + =
n
i
n
i
i
n
i
i i
n
i
i i i i i
b x b a x a b x b a x a x f
1 1
2
1
2
1
2 2 2 2
2 ) 2 ( ) (
0
1
2
>

=
n
i
i
a
0 4 4
1
2
1
2
2
1
|
.
|

\
|
|
.
|

\
|
|
.
|

\
|

= = =
n
i
i
n
i
i
n
i
i i
b a b a

108
f(x)=(a
1
x-b
1
)
2
-(a
2
x-b
2
)
2
--(a
n
x-b
n
)
2
. Pentru
1
1
a
b
r = , a
1 0
, avem f(r)=-(a
1
r-b
1
)
2
-
(a
2
r-b
2
)
2
--(a
n
r-b
n
)
2
0. Dar
f(x)= ( a
1
2
- a
2
2
-- a
n
2
)x
2
-2(a
1
b
1
a
2
b
2
-- a
n
b
n
)x+ b
1
2
- b
2
2
-- b
n
2

i cum a
1
2
- a
2
2
-- a
n
2
>0, deducem f0, inegalitate echivalent cu cea din
enun.
Dac n (2) avem egalitate, atunci f=0, deci f(x) 0, oricare ar fi x real. Rezult
f(r) 0 cu
1
1
a
b
r = Dar am demonstrat c f(r) 0, adic a
i
r=b
i
, i1,2,n`.
R10.2.3. Fie a
1
, a
2
,, a
n
,b
1
, b
2
,, b
n
numere reale pozitive i a
i
b
i
, oricare ar
fi i1,2,n`. Atunci:
Demonstraie:
Considerm funcia
Din a
i
b
i
, rezult

Pe de alt parte,
Cum coeficientul lui x
2
este pozitiv,folosind varianta II a principiului, rezult
f0, inegalitate echivalent cu cea din enun.
n urmtorul exemplu vom prezenta un alt raionament pentru deducerea
semnului lui f ,dect variantele prezentate. Fie f
1,
f
2
funcii de gradul al doilea
cu coeficienii dominani pozitivi. Atunci f=f
1
+f
2
este o funcie de gradul doi cu
coeficientul dominant pozitiv. Dac f
1
+f
2
ia i valori nepozitive, atunci au loc
una din inegalitile f0, f 0. ntr-adevr, presupunnd f
1
<0 i f
2
<0
obinem f
1
(x)>0 i f
2
(x)>0, oricare ar fi xR i atunci (f
1
+f
2
)(x)>0 oricare ar fi
xR, contrar ipotezei.
R10.2.4. Fie
n n
b b b b a a a a
2 2 1 2 2 1
, , , , , , , , , K K R
( )
|
|
.
|

\
|
|
|
.
|

\
|

+

n j i
j i
n j i
j i
n j i
j i j i
a b b a b b a a
p p p 1 1
2
1
4
( ) ( )( )


=
n j i
j j i i
a x b b x a x f
p 1
( )( )


=
n j i
j j i i
a b b a f
p 1
0 ) 1 (
( )


+
|
|
.
|

\
|
+
|
|
.
|

\
|
=
n j i
j i
n j i
j i j i
n j i
j i
a b x b b a a x b a x f
p p p 1 1
2
1
) (

109
Dac
2
1 2
2
2
2
1
2
2 2 2

+ + + >
n
a a a a L i
2
2
2
4
2
2
2
2 2 2
n
a a a a + + + > L ,atunci
( ) { }
2 2 1 1
2
2 2 2 2 1 1
, min C A C A b a b a b a ab
n n
L , unde
2
1 2
2
3
2
1
2
1
2 2 2

=
n
a a a a A L ,
2
1 2
2
3
2
1
2
1
2 2 2

=
n
b b b b C L
2
2
2
4
2
2
2
2
2 2 2
n
a a a a A = L ,
2
2
2
4
2
2
2
2
2 2 2
n
b b b b C = L .
Demonstraie: Fie : f RR,
( ) ( ) ( ) ( ) ( ) = =
2
2 2
2
2 2
2
1 1
2
n n
b x a b x a b x a b ax x f L
( ) ( )
2
2
2
1
2
2 2 1 1
2 2
2
2
1
2
2
n n n n
b b b x b a b a ab x a a a + L L L =
( ) ( ) x f x f
2 1
+ = unde : ,
2 1
f f RR
( ) ( ) ( ) + =

x b a b a b a ab x a a a a x f
n n n 2 2 2 2 1 1
2 2
1 2
2
3
2
1
2
1
2 2 2
2
1
L L +
2
1

( )
1
2
1
2
1 2
2
3
2
1
2
2
1
2
1
2 2 2 C Bx x A b b b b
n
+ =

L ,
( ) ( ) ( ) + = x b a b a b a ab x a a a a x f
n n n 2 2 2 2 1 1
2 2
2
2
4
2
2
2
2
2 2 2
2
1
L L +
( ) .
2
1
2 2 2
2
1
2
2
2
2
2
2
4
2
2
2
C Bx x A b b b b
n
+ = L
Din ipotez, coeficienii dominani ai funciilor f
1
i f
2
sunt pozitivi, deci i f are
coeficientul dominant pozitiv. Cum

deducem f
1
0 sau f
2
0, adic B
2
A
1
C
1
sau B
2
A
2
C
2
, atunci B
2
min(A
1
C
1
,
A
2
C
2
), care este tocmai inegalitatea (3).
R10.2.5. Fie (a
n
) n1 un ir de numere reale pozitive astfel nct ( )
1 1 2 n n
a este
monoton i a
2n-1
+ a
2n+1
< 2a
2n
oricare ar fi numrul n1. Atunci
Demonstraie: Fie f:RR
Vom folosi varianta III de aplicare a principiului trinomului. S evalum
semnul produsului f(-1)f(1). Avem
0 ...
2
2 2
2
1 1

|
.
|

\
|

|
.
|

\
|
=
|
.
|

\
|
n n
b
a
b
a b
a
b
a
a
b
f
( ) ( ) ( ) ( ) ( ) ( ) ) 4 ( 1 1 1
2
1
2
1
2
1
2
2
2
1
1
|
.
|

\
|
|
.
|

\
|
> |
.
|

\
|


=
+
= =
+
n
k
k
k
n
k
k
k
n
k
k k
k
a a a a
( ) ( )
2
2
1
1
1 ) (

=
+
+ =
n
k
k k
k
a x a x f

110
n prima parantez dreapt avem o sum de n expresii de forma

p
= - (-a
2p-1
+a
2p
)
2
+(-a
2p-1
+a
2p
)
2
, p1,2,,n`
iar n a doua parantez dreapt avem o sum de n expresii de forma

p
= - (a
2p-1
+a
2p
)
2
+(a
2p-1
+a
2p
)
2
,p1,2,,n `.
Avem

p
=( )( )
p p p p p
a a a a a
2 1 2 1 2 1 2 1 2
2 +
+ +
, i

p
= ( )( )
p p p p p
a a a a a
2 1 2 1 2 1 2 1 2
2 + +
+ +
.
Folosind ipoteza problemei deducem c
p

p
<0, de unde obinem c f(-
1)f(1)<0 i prin urmare f>0. Scriind funcia dat sub forma:
condiia f>0 este echivalent cu inegalitatea (4).

( ) ( ) ( ) ( )
(

+
(

+ =

=
+
=
+
n
k
k k
k
n
k
k k
k
a a a a f f
2
1
2
1
2
1
2
1
1 1 ) 1 ( ) 1 (
( ) ( ) ( )

=
+
=
+
=
+ + =
n
k
k
k
n
k
k k
k
n
k
k
k
a x a a x a x f
2
1
2
1
2
1
1
2
1
2 2
1 1 2 1 ) (
111
11. Principiul includerii i excluderii

n acest capitol vom urmri prezentarea unei metode prin care s se
poat numra elementele unei mulimi.

11.1. Mulimi finite. Principiul includerii i excluderii

11.1.1. Definiie: Spunem c o mulime A este finit dac exist o funcie
bijectiv A n f } ,..., 2 , 1 { : . Atunci numrul n este unic i spunem c A este
finit cu n elemente sau A este o mulime de cardinal n.

11.1.2 Notaie: Vom nota cu A cardinalul (sau numrul de elemente ) al
mulimii A.

11.1.3 Teorem (PRINCIPIUL INCLUDERII I EXCLUDERII)
Fie
n
A A ,...,
1
o familie de mulimi finite. Atunci cardinalul mulimii
n
A A A U U U ...
2 1
este dat de formula:

= <

< <

+ + + + =
=
n
i n j i
n
n
i i
i i
k
j i i
n
A A A A A A A A
A A A
k
k
1 1
2 1
1
...
1
2 1
... ) 1 ( ... ... ) 1 ( ...
| ... |
1
1
I I I I I I
U U U


numit formula lui Boole-Sylvester.

Demonstraie:
Procedm prin inducie dup n. Pentru n=2 trebuie s demonstrm relaia:
2 1 2 1 2 1
A A A A A A I U + = .
Aceasta rezult din faptul c
2 1
A A U este reuniunea mulimilor disjuncte
1
A i
) ( \
2 1 2
A A A I , iar
2
A este reuniunea mulimilor disjuncte ) ( \
2 1 2
A A A I i
2 1
A A I . Din egalitile:
2 1 2 1 2 2
) ( \ A A A A A A I I + =
) ( \
2 1 2 1 2 1
A A A A A A I U + =
rezult egalitatea de demonstrat. Presupunem c formula din enun este
adevrat pentru familii de n-1 mulimi i o vom demonstra pentru familii de n
mulimi:
112
. ... ) 1 (
... ) 1 ( ... ... ) 1 ( ...
... ) 1 ( ... ... ) 1 (
...
1 1
1
1 1
1
1
1 2 1
2
1 ... 1
1
1 1
1
1
2 1
2
1 ... 1
1
1
1 1
1
1
1
1
1
1 1
1
1
1
1
n n
n
n j i
n j i
n
i
n i
n
n
n i i
i i
k
n j i
j i
n
i
i
n
i
n i n
n
n i i
i i
k
n
i n j i
j i n i n
n
i
i n
n
i
i
n
i
n i
n
i
i
A A A A A A A A
A A A A A A A A
A A A A A A
A A A A A A A A A A A
k
k
k
k
I I I I I I
I I I I I
I I I I I
I I U
U
U U U U

<

< <

< =

< <

= <

= =
+ +
+ + + + =
= + + +
+ + + =

+ = =



Grupnd sumele cu acelai numr de factori n intersecie se obine formula din
enun.

11.1.4. Observaii:
(1) Formula lui Boole-Sylvester se mai scrie sintetic astfel:
(2) O form mai general a principiului includerii i excluderii este
urmtoarea: Fie A o mulime finit i ) , 0 [ : + B f o funcie.
Pentru orice submulime A B notm:

=
B x
x f B f ) ( ) ( , iar
0 ) ( = f . n aceste ipoteze, dac
U
n
i
i
A A
1 =
= atunci are loc egalitatea:

Dac f este funcie constant 1 ) ( = x f , A x atunci se obine
11.1.4.(1).

11.1.5. Teorem (indicatorul lui EULER)
Dac
m
m
p p p n

= ...
2 1
2 1
, unde 2
m
p p p < < < < ...
2 1
sunt numere prime, iar
N ,..., ,
2 1

m
atunci )
1
1 ( ... )
1
1 ( ) ( | 1 ) , ( , N
1
*
m
p p
n n n k n k k = = =
( ) (n numindu-se indicatorul lui Euler aferent numrului natural 2 n ).
( )


+
=
=
} ,..., 2 , 1 {
1
1
) 1 (
n I I i
i
I
n
i
i
A A
I U
( )


+
=
} ,..., 2 , 1 {
1
) 1 ( ) (
n I I i
i
I
A A f
I
113
Demonstraie:
Pentru un ntreg pozitiv n se noteaz prin ) (n funcia lui Euler, care este
egal cu numrul ntregilor pozitivi mai mici sau egali cu n i primi cu n.
S notm cu
i
A mulimea numerelor naturale mai mai mici sau egale cu n, care
sunt multipli de
i
p . Deducem:
i
i
p
n
A = ;
j i
j i
p p
n
A A

= I , deoarece numerele
i
p i
j
p fiind numere prime sunt prime ntre ele. Numerele naturale mai mici
sau egale cu n i care sunt prime cu n sunt numerele din mulimea
} ,..., 2 , 1 { n X = care nu aparin nici uneia dintre mulimile
i
A pentru m i 1 .
Deci = =
m
A A A n n U U U ... ) (
2 1



= <
= < =
=

+ =
= + + =
m
i m j i m
m
j i i
m
i
i
m
m j i
j i
m
i
i
p p
n
p p
n
p
n
n
A A A A n
1 1 1
1 1 1
...
) 1 ( ...
) 1 ( ...
I
I

)
1
1 ( ... )
1
1 ( )
1
1 (
2 1 m
p p p
n = .

11.1.6. Proprieti ale indicatorului lui Euler

(1) Dac (m,n) = 1 atunci ) ( ) ( ) ( n m n m = , adic funcia lui Euler este o
funcie aritmetic multiplicativ.
(2) Pentru orice n avem

=
n d
n n
|
) ( .

Demonstraie:
Vom demonstra (2). Presupunem c 0 , ...
2 1
2 1
> =
m m
m
p p p n

. Dup o idee
a lui Gauss vom demonstra afirmaia prin inducie dup suma
m
s + + + = ...
2 1
. Dac s = 1 rezult c n este numr prim i are doar
divizorii 1 i n; prin urmare suma devine n n n = + = + ) 1 ( 1 ) ( ) 1 ( .
Presupunnd proprietatea adevrat pentru numerele n care suma exponenilor
este mai mic dect s, o vom demonstra pentru s. mprim mulimea D a
divizorilor lui n n dou clase disjuncte:
1
D este mulimea divizorilor n care
1
p apare la puteri mai mici dect
1
, iar
2
D este mulimea divizorilor n care
1
p apare la puterea
1
. Avem
2 1
D D D U = i =
2 1
D D I . Prin urmare:
114
( )


= + = + =
D d D d D d
p
n
d
p
n
d
d p d d d d
1 2
1
1
1
1
| |
1
) ( ) ( ) ( ) ( ) (




n
p
n
n
p
n
p
n
p
p
p
n
= + =

+ =
1 1 1 1
1
1
1
1
1
1

.

n continuare vom prezenta cteva exemple clasice de utilizare a principiului
includerii i excluderii.

115
11.2. Probleme rezolvate (11)

R11.2.1. (PR1) Cte numere naturale mai mici sau egale cu 1000 sunt
divizibile cu 2 sau cu 3 sau cu 5.
Soluie: Fie } 1000 2 , | 2 { = n N n n A ; } 1000 3 , | 3 { = n N n n B ;
} 1000 5 , | 5 { = n N n n C . Se observ c
} 1000 30 , | 30 { },..., 1000 5 , | 6 { = = n N n n C B A n N n n B A I I I . Evident
numrul cutat este C B A U U . Aplicnd formula Boole-Sylvester avem:
= + + + = C B A A C C B B A C B A C B A I I I I I U U
=
(

+
(

+
(

+
(

30
1000
12
1000
15
1000
10
1000
6
1000
3
1000
2
1000

734 33 66 100 166 200 333 500 = + + + .

R11.2.2. Calculul numrului numerelor prime dintr-o mulime

Fie n>1 un numr natural nenul i ne propunem s calculm numrul
numerelor prime din mulimea A = {1,2,,n}. Ideea este de a proceda ca n
ciurul lui Eratostene: se pun numerele de la 1 la n ntr-un ir (cresctor) din care
se elimin cele divizibile cu 2, exceptndu-l pe 2, se reine 3 i se terg multiplii
lui 3 .a.m.d. Pentru numere n mai mici se poate proceda astfel, dar pentru
numere mari este necesar o analiz mai atent. Se observ c orice numr
compus din A este multiplu al unuia din numerele prime cel mult egale cu | | n .
Fie 3 , 2
2 1
= = p p irul numerelor prime cel mult egale cu | | n . Pentru fiecare
i
p din acest ir notm } | , 1 | { a p n a a A
i p
i
= . Atunci numrul numerelor
prime cel mult egale cu n este
U
k
i
p
i
A k n
1
1
=
+ .
De exemplu, s calculm numrul numerelor prime mai mici dect 250.
Deoarece | | 15 250 = rezult c irul numerelor prime cu care vom lucra este:
2,3,5,7,11,13 i avem de calculat cardinalul unei reuniuni de 6 mulimi:

+ =
=
l k j i k j i j i i
A A A A A A A A A A
A A A A A A
I I I I I I
U U U U U
13 11 7 5 3 1
.
Formula este aceasta deoarece din inegalitatea 250 11 7 5 3 2 >
rezult c intersecia a cinci mulimi este vid. innd seama de faptul c
116
(

=
i
p
p
n
A
i
,
(
(

=
j i
p p
p p
n
A A
j i
I .a.m.d. obinem =
i
A U 202. Prin urmare
rezult c n mulimea {1,2,3,,250} sunt 53 numere prime.

R11.2.3. ( PR3) ntr-un institut de cercetare se afl 67 de cercettori. Dintre
acetia 47 vorbesc limba englez, 35 vorbesc limba germana i 23 ambele limbi.
Ci cercettori din institut nu vorbesc nici una dintre cele dou limbi?

Soluie: Pentru rezolvarea problemei nu se poate aplica sub forma dat
principiul includerii i excluderii i prin urmare l reformulm. Fie A o mulime
cu n elemente avnd sau nu cel puin una dintre proprietile notate 1,2,...,p.
Notm cu A
i
mulimea elementelor ce au proprietatea i(1i p) (indiferent dac
elementele mulimii mai au i alte proprieti); A
i
A
j
mulimea elementelor ce
au proprietile i i j (chiar dac elementele din A
i
A
j
mai au i alte proprieti),
etc. De asemenea, notm cu i A mulimea elementelor ce nu au proprietatea
i(1 p i )
j i
A A mulimea elementelor ce nu au proprietile i i j etc.
Atunci principiul includerii i excluderii are forma:
p
A A A ...
2 1
=
=n -
p
A A A ...
2 1
= n - + +

< =
...
1 1 p j i
j i
n
i
i
A A A
I
p
i
i
p
A
1
) 1 (
=

n cazul nostru A
1
este mulimea cercettorilor ce vorbesc limba englez, A
2

mulimea cercettorilor care vorbesc limba german i se cere
2 1
A A . Avem:

2 1
A A =67- 8 23 35 47 67
2 1 2 1
= = + A A A A .
Acesta este numrul celor care nu vorbesc nici limba englez i nici
limba german.

R11.2.4. ( PR4 ) Cte numere compuse din n cifre exist, care nu conin
cifrele 1,2,3 dar pe fiecare dintre acestea cel puin o dat?
Soluie: Fie A
i
mulimea numerelor de n cifre care nu-l conin pe
i(i=1,2,3). Atunci pe baza principiului includerii i excluderii
3 2 3 3 3
3 2 1 3 2 1
+ = =
n n n
A A A A A A .

R11.2.5. Problema coincidenelor ( a punctelor fixe )
117
Definiie : O aplicaie bijectiv { } { } n n , , 2 , 1 , , 2 , 1 : K K cu ( ) i i = se
spune c admite o coinciden n i. Numrul i se numete punct fix al aplicaiei .
Numrul total de aplicaii bijective ( permutri ) este egal cu ! n .
Se pot defini urmtoarele mulimi ( ) { } i i A
i
= = cu ( )! 1 = n A
i
,
( ) ( ) { } i j j i i A A
j i
, , = = = I < j, cu ( )! 2 = n A A
j i
I , .a.m.d.
Problema coincideelor : Un lucrtor la pot are n fa n plicuri cu
adresele a n persoane diferite i n scrisori adresate fiecrei persoane ( cte una
de fiecare persoan ) . El trebuie s introduc n fiecare plic cte o scrisoare.
Discutnd cu oamenii de la ghieu lucrtorul este neatent i introduce oricum
scrisorile n plicuri.
n cte moduri poate introduce cele n scrisori n plicuri astfel ca nici o
persoan s nu-i primeasc scrisoarea ? Dar numai una s-i primeasc
scrisoarea ?

Soluie : Pentru a determina numrul de modaliti n care nu avem
permutri cu coincidene aplicm formula includerii i excluderii i avem :
( ) ( ) ( )
( )
|
|
.
|

\
|

+ + =
=
|
|
.
|

\
|
+
|
|
.
|

\
|
+
|
|
.
|

\
|
=
!
1
! 2
1
! 1
1
1 !
1 ! 2
2
! 1
1
! ...
2 1
n
n
n
n
n
n
n
n
n A A A
n
n
n
L
L

i reprezint numrul de posibiliti de a introduce scrisorile n plicuri n aa fel
nct nici una dintre persoane s nu-i primeasc scrisoarea. Pentru a determina
n cte moduri doar o persoan i poate primi un plic cu scrisoarea care i era
adresat procedm astfel : dac unul dintre plicuri are scrisoarea adresat ,
atunci n celelalte 1 n plicuri se pot introduce astfel nct nici unul s nu
conin scrisoarea cu acelai numr n ( )
( )
( )
|
|
.
|

\
|

+ +

! 1
1
! 2
1
! 1
1
1 ! 1
1
n
n
n
L moduri.
Deoarece un plic se poate alege din cele n plicuri n n moduri deducem c
numrul total de moduri este ( )
( )
( )
|
|
.
|

\
|

+ +

! 1
1
! 2
1
! 1
1
1 ! 1
1
n
n n
n
L .



118
12. Metoda apropierii de extrem, sau metoda variaiei pariale a lui Sturm

12.1. Introducere
n cadrul acestei teme ne propunem s descriem o metod elementar de
demonstrare a unor inegaliti, propus n 1884 de matematicianul german R.
Sturm.
Metoda este prezentat pe larg n revista sovietic Kvant
(nr. 1/1981) i se refer la inegaliti cu funcii de mai multe variabile, n care se
fixeaz pentru moment o parte din variabile, lsnd s varieze una sau dou
dintre acestea. De aceea metoda este cunoscut i sub numele de metoda
variaiei pariale.
Metoda lui Sturm de demonstrare a unor inegaliti n care apar n, n2
numere reale const n a studia comportarea unei expresii mai simple, de dou
variabile, atunci cnd acestea iau valori apropiate, n ipoteza c suma sau
produsul lor este o mrime constant. Raionamentul continu n cazul a n
numere n care cel puin dou sunt distincte i se aplic rezultatul obinut
anterior de mai multe ori.
Studiul riguros al funciilor de mai multe variabile se face cu ajutorul
analizei matematice i depete nivelul cunotinelor de liceu.
Primul exemplu, preluat din revista citat, are ca scop descrierea
metodei la o problem de geometrie.



Bibliografie

Gh. Eckstein, Metoda apropierii de extrem a lui Sturm, RMT 1/1984 pag
3-10
M. Ganga, Teme i probleme de matematica,Ed tehnica, Bucureti,
1991, pag 117-123


119
Probleme rezolvate (12)

R12.2.1 Dintre toate poligoanele de n laturi nscrise ntr-un cerc dat, s se
gseasc poligonul de arie maxim.
Soluie. Considerm A,B,C trei vrfuri consecutive ale poligonului i
presupunem AB>BC (vezi figura).

B
1

B B

A H
1
H C




Fig.
S studiem cum variaz aria triunghiului ABC, dac fixm A i C, iar
B se deplaseaz pe arcul ABC , astfel nct lungimile AB i BC s se apropie.
Bac B aparine arcului AC cu BA=BC, atunci a apropia lungimile AB i BC
nseamn a nlocui B cu orice punct B
1
aparinnd arcului BB. nlimea
B
1
H
1
>BH, deci aria triunghiului crete. Trecnd la analiza cazului general, s
observm c nici un poligon neregulat nu este soluie a problemei, deoarece
dac [AB] i [BC] sunt laturi neegale, nlocuind B cu B
1
, B
1
aparinnd arcului
BB, obinem un poligon cu arie mai mare.
S demonstrm c poligonul regulat are aria mai mare dect aria oricrui
alt poligon nscris n acelai cerc. S observm c dac nlocuim B cu B,
obinem un poligon cu aceeai arie i aceleai laturi, doar c ordinea a dou
laturi vecine s-a schimbat. Repetnd de un numr finit de ori aceast operaie,
putem schimba ntre ele oricare dou laturi. Fie un poligon diferit de
poligonul regulat cu n laturi. Cea mai mic latur subntinde un arc mai mic
dect 360
0
/n, iar cea mai mare un arc mai mare dect 360
0
/n. Permutm laturile
lui astfel nct cea mai mic i cea mai mare latur s fie vecine. n figur,
acestea sunt [BC] i [AB]. Alegem pe arcul AB punctul B
1
, astfel ca
m(ABC)=360
0
/n. Obinem poligonul
1
de arie mai mare i n care cel puin o
latur are lungimea laturii poligonului regulat.


120
B
1

B
A C




F D


E
Fig.
Dac poligonul
1
nu este regulat, repetm raionamentul i obinem poligonul

2
cu aria i mai mare i cel puin dou laturi egale cu lungimea laturii
poligonului regulat. Dup cel mult n-1 pai, ajungem la poligonul regulat i am
demonstrat c poligonul regulat cu n laturi nscris ntr-un cerc are aria strict mai
mare dect orice alt poligon cu n laturi nscris n acelai cerc. n urmtorul
exemplu vom demonstra o inegalitate cu metoda lui Sturm.
R12.2.2. Pentru orice numere reale a
1
, a
2
,,a
n
, este adevrat inegalitatea
2
2 1
2 2
2
2
1

+ + +

+ + +
n
a a a
n
a a a
n n
L L
(1)
Soluie. S observm c dac 0
2 1
= + + +
n
a a a L atunci inegalitatea este
satisfcut, cu egalitate dac i numai dac 0
2 1
= = = =
n
a a a L . Dac
0
2 1
= + + + S a a a
n
L atunci (1) se mai
scrie
n s
a
s
a
s
a
n
1
2 2
2
2
1

+ +

L . Notnd { } n k
s
a
b
k
k
, , 2 , 1 , K = problema
devine:
tiind c b
1
+b
2
++b
n
=1, s se demonstreze c
n
b b b
n
1
2 2
2
2
1
+ + + L (2)
S studiem cum se modific suma ptratelor a dou numere reale a i b dac le
modificm fr a schimba suma lor. nlocuim a cu a+x iar b cu b-x. Dac a<b,
iar x(0,b-a), numerele se apropie, iar dac x<0, numerele se deprteaz.
S presupunem c apropiem numerele. Avem:
( ) ( ) ( ) . 2
2 2 2 2 2 2
b a x a b x b a x b x a + < + = + +
Deci apropiind numerele, suma ptratelor lor scade. n concluzie, dac nu toate
numerele b
1
,b
2
,,b
n
sunt egale, atunci suma

121
2 2
2
2
1 n
b b b + + + L poate fi micorat. Vom demonstra c dac b
1
+b
2
++b
n
=1,
atunci
n n n n
b b b
n
1 1 1 1
2 2 2
2 2
2
2
1
=

+ +

+ + + L L
ntr-adevr, dac nu toate numerele b
1
,b
2
,,b
n
sunt sunt egale cu
n
1
, atunci
unul dintre numere este mai mic dect
n
1
, altul mai mare dect

n
1
. Putem presupune c aceste numere sunt b
1
i b
2
. nlocuind b
1
cu
n
1
, iar pe b
2
cu
n
b b
1
2 1
+ , noile n numere au tot suma 1, dar suma ptratelor mai mic. n
plus, cel puin unul dintre aceste numere este egal cu
n
1
. Repetnd de cel mult
n-1 ori aceast operaie, inegalitatea (2) este demonstrat i prin urmare (1) este
adevrat.


Observaii:
1. n exemplele R 12.2.1 i 12.2.2. am cutat extremul unei funcii de n
variabile fcnd s varieze doar un vrf (Ex. 1) sau dou numere (Ex.2).
2. n etapa a doua a demonstraiei, (Ex. 1) am apropiat variabilele astfel: la
trecerea de la la
1
am ales B
1
astfel ca [AB
1
] s aib lungimea laturii
poligonului regulat. Dac alegeam B
1
la mijlocul arcului AC, mream aria,
dar nu ajungeam la poligonul regulat dup un numr finit de pai.
3. Exemplul R-12.2.2. a fost ales pentru exemplificarea metodei lui Surm,
inegalitatea (1) putndu-se demonstra i prin alte metode. De exemplu:
pornim de la x
2
-2xs+s
2
0 x
2
-s
2
2s(x-s) (3) nlocuim succesiv x cu
a
1
+a
2
++a
n
i adunnd inegalitile obinute, apoi nlocuim s cu media
aritmetic a numerelor a
1
+a
2
++a
n
i obinem (1).
Urmtorul exemplu trateaz aplicarea metodei la o problem de
Trigonometrie.
R12.2.3. Dac a,b,c,d0, iar a+b+c+d=, atunci
4
1
sin sin sin sin d c b a
Soluie. S analizm produsul sinasinb, cu a+b=constant, dac numerele
a i b se apropie (a+b<). Fie a<b, 0<2x<b-a.
n identitatea sinasinb= ( ) ( ) [ ], cos cos
2
1
b a b a + nlocuim

122
a i b cu a+x, respectiv b-x. Identitatea devine:
( ) ( ) ( ) ( ) [ ] = + + = + b a x b a x b x a cos 2 cos
2
1
sin sin
= ( ) ( ) [ ] b a x a b + cos 2 cos
2
1

Cum < < < a b x a b 2 0 i funcia cosinus este descresctoare pe [0,].
Deducem cos(b-a-2x)>cos(b-a), deci sin(a+x)sin(b-x)>sinasinb. Prin urmare,
dac nlocuim numerele a i b de sum constant cu altele mai apropiate
avnd de asemenea suma constant, produsul sinusurilor crete. Vom
demonstra c maximul produsului are loc cnd numerele a,b,c,d sunt egale.
Dac ntre unghiurile a,b,c,d, exist dou inegale, atunci unul este strict mai mic
dect
4

, iar altul strict mai mare dect


4

. Fie
4

< a i
4

> b . nlocuind aceste


unghiuri cu
4

i respectiv
4

+b a , suma rmne constant, iar produsul de


sinusuri s-a mrit, dup cum rezult din raionamentul fcut anterior. Repetnd
raionamentul, obinem n final c pentru
4

= = = = d c b a , produsul este
maxim.
Observaie: Problema admite urmtoarea generalizare:
Dac , 0
i
x i1,2,,n cu =

=
n
i
i
x
1
, atunci are loc inegalitatea
n
n
i
i
n
x

sin sin
1
, n n , 2 N.
n exemplul urmtor extremele nu se ating pentru valori egale ale
variabilelor aa cum s-a ntmplat n celelalte exemple.

R12.2.4. S se gseasc maximul i minimul expresiei:
unde a,b,c,d sunt numere nenegative, cel mult dou egale cu zero.
(O.I.M.)
Soluie. Dac s d c b a = + + + atunci ( )

=
s
d
s
c
s
b
s
a
E d c b a E , , , , , , .
Notnd
, , , ,
, , , d
s
d
c
s
c
b
s
b
a
s
a
= = = = , avem a+b+c+d=1, deci putem
presupune a+b+c+d=1 i n acest caz avem
d c a
d
d c b
c
c b a
b
d b a
a
d c b a E
+ +
+
+ +
+
+ +
+
+ +
= ) , , , (

123
Observm c expresia este simetric n a i c, n b i d, iar perechile (a,c) i
(b,d) joac i ele un rol simetric. S presupunem a<c i s studiem cum se
modific E(a,b,c,d) dac nlocuim a cu a+x, c cu c-x, astfel nct a+x<c-x.
Avem:

Deducem de aici c dac a i c se apropie ( ) 0 > x , expresia E crete, iar dac a
i c se ndeprteaz ( ) 0 < x , valoarea expresiei E scade. n situaiile extreme
cnd se apropie, a=c i b=d, iar cnd se ndeprteaz, a0, ca+c, b0,
db+d. Dac notm a+c=2u, b+d=2v,
obinem ( ) ( ) ( ) 1 2 , 2 , 0 , 0 , , , , , , = v u E d c b a E v u v u E . Deci minimul lui E este 1.
Pentru maxim s lum ( )
v
v
u
u
v u v u E

=
1
2
1
2
, , , i s cutm maximul lui E,
cnd 2(u+v)=1. Urmrind un raionament analog ajungem la concluzia c E
crete cnd u se ndeprteaz de v. Ajungem n final s obinem c maximul lui
E(a,b,c,d) este E(0,2,0,2)=2.
Observaie:
n acest exemplu, nici minimul nici maximul nu s-au atins n cazul n
care toate variabilele sunt egale. n aceast situaie, a=b=c=d, iar
expresia ( )
3
4
, , , = a a a a E ,care nu este nici minim, nici maxim pentru E.
n exemplele precedente am variat dou dintre variabile, celelalte
rmnnd fixe. Se poate ns varia i numai o singur variabil, aa cum este
cazul prezentat n urmtorul exemplu.

R12.2.5. Inegalitatea lui Schweitzer.
Dac b a < < 0 iar [ ] b a x x x
n
, , , ,
2 1
K , atunci
b
d
d
b
a
c
c
a
d c b a E

=
1 1 1 1
) , , , (
=



+
+
+
= +
a
c
c
a
x a
x c
x c
x a
d c b a E d x c b x a E
1 1 1 1
) , , , ( ) , , , (
) 1 )( 1 )( 1 )( 1 (
) )( 2 )( 1 (
) 1 )( 1 (
) 1 (
) 1 )( 1 (
) 1 (
a x a c x c
x a c c a c a x
a x a
c a x
c x c
c a x
+

=

+

=
( )
2
2
2 1
2 1
4
1
...
1 1
) ... ( n
ab
b a
x x x
x x x
n
n
+

+ + + + + +

124
Soluie. Studiul variaiei unei singure necunoscute ne sugereaz
considerarea funciei ( ) , 0 : f R , ( ) ( )

+ + =
x
d x c x f
1
,
. 0 , > d c Avem ( ) ( ) . 1 2 1 1
2
+ = + + + + + = cd cd cd
x
c
xd cd x f Minimul
funciei are loc pentru ,
d
c
x = , deci f este descresctoare pe

d
c
, 0 i
cresctoare pe , ,

+
d
c
prin urmare, maximul funciei f pe intervalul [a,b]
este atins la unul din capete. Fixnd n-1variabile i variind-o pe a n-a, deducem

( ) ( )

+ +

+ + + + + +
b
q
a
p
qb pa
x x x
x x x
n
n
1 1 1
2 1
2 1
L L ,
cu p,q naturale i p+q=n (n p puncte variabilele iau valoarea a i n q puncte
valoarea b). Folosind inegalitatea 4AB(A+B)
2
, obinem
n continuare propunem aplicarea metodei lui Sturm la rezolvarea
urmtoarelor probleme, care au fost propuse la diverse concursuri.

2
2 2
4
) (
4
) (
4
) )( ( 4
) ( n
ab
b a
ab
qa pb qb pa
ab
qa pb qb pa
b
q
a
p
qb pa
+
=
+ + +

+ +
=

+ +

125
13. Baricentre i centre de greutate


n cadrul acestei teme prezentm o metod de rezolvare a problemelor
de geometrie plan i n spaiu care i are originea n mecanic.
Vom prezenta mai nti consideraiile de mecanic care au condus la
noiunea de baricentru.
Prin punct material n fizic nelegem cuplul (punct, numr): (A,m).
Altfel spus, n punctul A se afl masa m > 0. Pentru (A,m) scriem simplu mA. S
considerm dou bile de mase m
1
i m
2
la capetele unei bare A
1
A
2
(vezi figura
13.1). Exist un punct G n care dac bara este legat, atunci sistemul este n
echilibru. Punctul G se numete centrul masei sistemului de puncte materiale S,
sau baricentrul sistemului { }
2 2 1 1
, A m A m S =



A
1
G A
2

m
1
m
2

Fig. 13.1
Noiunea poate fi extins la o configuraie spaial cu sferele de mase m
1
, m
2
,,
m
n
. Pentru aplicarea conceptului la rezolvarea problemelor de geometrie trebuie
s inem seama de urmtoarele reguli:
1. Orice sistem format din n puncte materiale are un baricentru i acesta este
unic.
2. Baricentrul a dou puncte materiale este situat pe segmentul determinat de
cele dou puncte. Poziia acestuia este determinat de regula prghiei:
produsul masei punctului material cu distana dintre acesta la baricentru este
acelai pentru cele dou puncte materiale, adic m
1
d
1
=m
2
d
2
.
3. Dac ntr-un sistem format dintr-un numr finit de puncte materiale marcm
cteva puncte materiale i masele lor le mutm n baricentrul acestor puncte
materiale, atunci poziia baricentrului sistemului iniial nu se schimb.

13.1 Definiia matematic a baricentrului

Vom analiza pentru nceput cazul a dou puncte materiale m
1
A
1
, m
2
A
2

avnd n G baricentrul (regula1) Egalitatea
2 2 1 1
d m d m = (regula 2) se poate
scrie sub forma:
2 2 1 1
GA m GA m = . Cum vectorii
1
GA ,
2
GA

au sensuri opuse,
relaia se scrie vectorial
2 2 1 1
GA m GA m = ( ) 1 .

126
Fie acum trei puncte materiale m
1
A
1
, m
2
A
2
, m
3
A
3
i G baricentrul
acestui sistem. (vezi figura 13.2).

m A
3 3
m A
1 1
m A
2 2 (m + m )G
1 2 1
G


Fig. 13.2
Fie G
1
baricentrul sistemului format din punctele m
1
A
1
, m
2
A
2
. Conform (1)
putem scrie: 0
2 1 2 1 1 1
= + A G m A G m ( ) 2 innd cont de regula 3, baricentrul
sistemului iniial format din punctele materiale m
1
A
1
, m
2
A
2
, m
3
A
3
coincide cu
baricentrul sistemului format din punctele materiale

(m
1
+m
2
)G
1
, m
3
A
3
i innd
seama de (1) avem

( ) 0
3 3 1 2 1
= + + GA m GG m m ( ) 3 . Dar ( ) + = +
1 1 1 2 1
GG m GG m m

( ) ( ) + = + = +
2 2 1 1 2 1 2 2 1 1 1 1 1 2
GA m GA m A G GA m A G GA m GG m
( )
2 2 1 1 2 1 2 1 1 1
GA m GA m A G m A G m + = + .
Prin urmare (3) devine: 0
3 3 2 2 1 1
= + + GA m GA m GA m ( ) 4
n mod asemntor se poate deduce o relaie pentru mai multe puncte materiale.
Cu acestea putem da urmtoarea:

13.1.1. Definiie: Baricentrul sistemului de puncte materiale m
1
A
1
, m
2
A
2
,,
m
n
A
n
este punctul G pentru care are loc egalitatea:
0
2 2 1 1
= + + +
n n
GA m GA m GA m L
Masele m
1
m
2
,,m
n
se numesc ponderi. Dac aceste ponderi sunt egale
obinem un caz particular de baricentru, numit centru de greutate.

13.1.2. Definiie: Un punct G se numete centru de greutate al punctelor A
1
,
A
2
, ,A
n
din spaiu dac
0
2 1
= + + +
n
GA GA GA L

127
Orice sistem de puncte din spaiu are un unic centru de greutate (din regula 1)
Notnd prin { }
n n
A m A m A m S , , ,
2 2 1 1
K = sistemul de puncte materiale, avem
urmtoarea teorem de caracterizare a baricentrului unui sistem de puncte
materiale:

13.1.3. Teorem: a) Dac G este baricentrul sistemului de puncte materiale S,
atunci oricare ar fi punctul O, are loc egalitatea
n
n m
m m m
OA m OA m OA m
OG
+ + +
+ + +
=
L
L
2 1
2 2 1 1
( ) 5
b) Dac pentru orice punct O are loc (5), atunci G este baricentrul sistemului S.
Demonstraie
Vom trata cazul n=2, pentru n>2 procedndu-se analog.
a) Fie O un punct arbitrar. Egalitatea 0
2 2 1 1
= + GA m GA m se scrie sub forma
( ) ( ) 0
2 2 1 1
= + OG OA m OG OA m sau = +
2 2 1 1
OA m OA m
( )OG m m
2 1
+ = , care este relaia de demonstrat.
b) Afirmaia se deduce parcurgnd n sens invers egalitile de la punctul a).
Din teorema enunat rezult c orice sistem S format dintr-un numr finit de
puncte materiale are un baricentru unic (rezultatul corespunde regulii 1). S
artm c definiia baricentrului ne conduce la regula 2.

13.1.4. Teorem: Baricentrul a dou puncte materiale este situat pe segmentul
ce unete cele dou puncte. Poziia acestuia se determin prin regula prghiei:
m
1
d
1
= m
2
d
2
.
Demonstraie: Considerm punctele materiale m
1
A
1
, m
2
A
2
cu baricentrul G.
Atunci 0
2 2 1 1
= + GA m GA m , adic vectorii
1
GA ,
2
GA sunt coliniari i de sens
contrar, deci G se afl ntre A
1
i A
2
, G[A
1
A
2
]. Din
2 2 1 1
GA m GA m = rezult
m
1
d
1
= m
2
d
2
. Verificarea regulii numrul 3 este dat de urmtoarea

13.1.5. Teorem de grupare a punctelor materiale: Fie sistemul de puncte
materiale { }
n n
A m A m A m S , , ,
2 2 1 1
K = n care marcm k puncte materiale
m
1
A
1
,m
2
A
2
,,m
k
A
k
i fie G baricentrul punctelor marcate. Dac ntreaga mas
a punctelor marcate o concentrm n G, atunci baricentrul sistemului de puncte
materiale (m
1
+m
2
++m
k
)G, m
k+1
A
k+1
,,m
n
A
n
coincide cu baricentrul
sistemului S.
Demonstraie:
Fie G baricentrul sistemului S, deci are loc egalitatea
0
1 1 2 2 1 1
= + + + + + +
+ + n n k k k k
GA m GA m GA m GA m GA m L L

128
Deoarece G este baricentrul sistemului { }
k k
A m A m A m , , ,
2 2 1 1
K , avem
i obinem
n
n n
m m m
OA m OA m OA m
OG
+ + +
+ + +
=
L
L
2 1
2 2 1 1

( ) 0 '
1 1 2 1
= + + + + + +
+ +
n
n k k k
GA m GA m GG m m m L L
prin urmare G este baricentrul sistemului de puncte materiale
(m
1
+m
2
++m
k
)G, m
k+1
A
k+1
++ m
n
A
n
i teorema este demonstrat. Formula
(%) are loc pentru orice punct O. Putem folosi o scriere simplificat
n
nk n
m m m
A m A m A m
G
+ + +
+ + +
=
...
...
2 1
2 2 1 1
r r r

De exemplu scrierea
7
4 1 2 C B A
G
+ +
=
Arat c G este baricentru a trei puncte materiale 2A, 1B, 4C. Analog,
5A+3B=8G arat c G este baricentrul punctelor materiale 5A, 3B. Implicaia
[ ] ED G
D E D C B A
G
+
=
+ + +
=
10
4 6
10
4 ) 1 3 2 (

are urmtoarea semnificaie: G este baricentrul punctelor materiale 2A, 3B, 1C,
4D. Dac concentrm masele a trei puncte materiale 2A, 3B, 1C n baricentrul
E, atunci G[ED].
Observaii:
1. Dac G este baricentrul maselor m
1
, m
2
, m
3
, plasate n vrfurile A, B i
respectiv C (vezi figura 13.3). Atunci dreapta AG intersecteaz latura BC n A
care este baricentrul punctelor materiale m
1
B i m
3
C.
2. Baricentrul G se poate determina astfel: n vrfurile A,B,C ale triunghiului
ABC sunt plasate masele m
1
, m
2
i m
3
(vezi figura 13.4). Dac B este
baricentrul sistemului { } C m A m
3 1
, i C baricentrul sistemului { } B m A m
2 1
, ,
atunci G=BBCC este baricentrul sistemului de puncte materialem
1
A,m
2
B,
m
3
C.

m B
2
m A
1
m C
3 A
G
m A
1
m B
2
m C
3
B
C
G

Fig. 13.3 Fig. 13.4

129
Aceast observaie este important, servind la demonstrarea concurenei a
dou sau mai multor drepte prin gruparea convenabil a punctelor materiale
(regula 3).
3. Utilizarea metodei baricentrelor n rezolvarea problemelor de geometrie
const n: i) alegerea convenabil a punctelor; ii) punerea maselor
corespunztoare n aceste puncte, astfel nct problema s fie uor rezolvabil.


13.2. Baricentrul unui sistem de puncte materiale n care apar i mase
negative

Datorit faptului c am definit centrul de mas al unui sistem de puncte
materiale printr-o relaie vectorial, putem lua coeficienii vectorilor i numere
negative. De exemplu, centrul de mas al sistemului (-3)A, 4B este acel punct G
pentru care are loc relaia vectorial
( ) 0 3 3 = + GB GA . Condiia pentru ca un sistem de puncte materiale
{ }
n n
A m A m A m , , ,
2 2 1 1
K s aib un baricentru este ca suma maselor
m
1
+m
2
++m
n
s fie nenul. Au loc rezultatele din paragraful precedent i
putem afirma c: Centru G a dou mase m
1
i m
2
cu m
1
+m
2
0, aezate la
capetele segmentului A
1
A
2
se afl pe dreapta A
1
A
2
i verific condiia
m
1
d
1
=m
2
d
2
, unde d
1
=GA
1
, d
2
=GA
2
. Avem G[A
1
A
2
] dac m
1
,m
2
au
acelai semn sau G aparine exteriorului segmentului [A
1
A
2
] dac numerele
m
1
,m
2
au semne diferite. n cazul maselor negative, centrul de mas este mai
aproape de acea mas care este mai mare n valoare absolut.

13.3. Coordonate baricentrale n plan

n paragrafele anterioare am artat c orice mase m
1
,m
2
,m
3
aezate n
vrfurile triunghiului A
1
A
2
A
3
definesc un punct unic G care este baricentrul
sistemului de puncte materiale m
1
A
1
,m
2
A
2
,m
3
A
3
. Are loc i proprietatea
reciproc, adic pentru orice punct G din planul triunghiului A
1
A
2
A
3
exist trei
mase m
1
,m
2
,m
3
plasate n vrfurile A
1
A
2
A
3
ale triunghiului astfel nct sistemul
de puncte materiale s aib centrul de mas G. Cele trei numere m
1
,m
2
,m
3
nu
definesc n mod unic punctul G, deoarece (km
1
)A
1
, (km
2
)A
2
, (km
3
)A
3
are
acelai baricentru. Lund k=1/( m
1
+m
2
+m
3
) i notnd
i
=k
i
, { } 3 , 2 , 1 = i , avem
c G este baricentrul punctelor materiale
i
A
i
, { } 3 , 2 , 1 = i . Putem spune c
pentru orice punct Gint[A
1
A
2
A
3
] exist trei numere pozitive
I
,
2
,
3
cu

I
+
2
+
3
=1 pentru care G este baricentru sistemului
1
A
1
,
2
A
2
,
3
A
3
,
adic G=
1
A
1
+
2
A
2
+
3
A
3
. Numerele
I
,
2
,
3
care verific proprietile de

130
mai sus se numesc coordonatele baricentrice ale punctului G relative la
triunghiul A
1
A
2
A
3
.
n cazul particular n care G este punctul de intersecie al medianelor
triunghiului A
1
A
2
A
3
, atunci G este centrul de mas al sistemului cu suma
ponderilor 1:
1
3
1
A ,
2
3
1
A ,
3
3
1
A deci coordonatele baricentrice ale lui G sunt
3
1
1
= ,
3
1
2
= ,
3
1
3
= .
Dac extindem coordonatele baricentrice i la mase negative atunci
centrul de mas G aparine planului (A
1
A
2
A
3
). Are loc urmtoarea:

13.3.1. Teorem: Fie A
1
A
2
A
3
un triunghi dat i M un punct n planul
triunghiului. Atunci exist numerele reale unice
I
,
2
,
3
cu
I
+
2
+
3
=1 i M
baricentrul sistemului
1
A
1
,
2
A
2
,
3
A
3
, deci M=
1
A
1
+
2
A
2
+
3
A
3
.
n urmtoarea teorem vom raporta coordonatele baricentrice la ariile
unor triunghiuri.

13.3.2. Teorem: Fie punctul P situat n interiorul triunghiului A
1
A
2
A
3
i fie
S,S
1
,S
2
,S
3
ariile triunghiurilor A
1
A
2
A
3
, P
1
A
2
A
3
, A
1
PA
3
, PA
1
A
2
. Atunci
coordonatele baricentrice ale punctului P sunt
S
S
S
S
S
S
3
3
2
2
1
1
, , = = =
Deci, dac punem n vrfurile A
1
A
2
A
3
masele egale cu S
1
,S
2
,S
3
atunci
baricentrul acestor puncte este P.
Demonstraie:
Fie (
I
,
2
,
3
) coordonatele baricentrice ale punctului P din interiorul
triunghiului A
1
A
2
A
3
(vezi figura 13.5). Sistemul format din punctele materiale

2
A
2
i
3
A
3
au baricentrul n punctul B
1
[A
2
A
3
], prin urmare punctele A
1
, P,
B
1
sunt coliniare. Are loc relaia

3 1 3 2 1 2
A B A B = sau
3 1
2 1
2
3
A B
A B
=



131
A
1
A
2 A
3
P
B
1
S
2 S
3



Fig. 13.5
Triunghiurile A
1
A
2
B
1
i A
1
B
1
A
3
au aceeai nlime (din A
1
), la fel triunghiurile
PA
2
B
1
i PB
1
A
3
. Atunci
3 1
2 1
1 3
1 2
1 3 1
1 2 1
A B
A B
S
S
S
S
B PA
B PA
B A A
B A A
= = 3
De unde deducem
3 1
2 1
3 1 1 3 1
1 2 1 2 1
A B
A B
S S
S S
A PB B A A
B PA B A A
=

sau
3 1
2 1
2
3
A B
A B
S
S
=
Din (1) i (2) avem
3
3
2
2
2
3
2
3
S S S
S

= =
Analog obinem relaia
S S S S S S S
deci
S S
1
,
3 2 1
3 2 1
3
3
2
2
1
1
2
2
1
1
=
+ +
+ +
= = = =


Rezult c
S
S
S
S
S
S
3
3
2
2
1
1
, , = = =
Care sunt coordonatele baricentrice ale punctului P.
Consecine:
1. Cu ajutorul teoremei 13.3.2. putem deduce coordonatele baricentrice ale
ortocentrului unui triunghi A
1
A
2
A
3
ascuitunghic, dac
m(A
1
)=, m(A
2
)= m(A
3
)=. Dac (
I
,
2
,
3
) sunt coordonatele baricentrice a
lui H atunci PH i A
1
B
1
A
2
B
3
. Din (1) avem

tg
tg
tg ctg
A B
B A
B A
A B
A B
A B
= = = =
3 1
1 1
1 1
2 1
3 1
2 1
2
3

Deducem analog

tg
tg
=
2
1
i obinem coordonatele baricentrice ale lui H:






tg tg tg
tg
tg tg tg
tg
tg tg tg
tg
+ + + + + +
, ,
innd seama de identitatea tg+tg+tg= tgtgtg, coordonatele se mai scriu
(ctgctg, ctgctg, ctgctg).

132

2. Coordonatele baricentrice ale centrului nscris n triunghiul ABC
avnd laturile a,b,c sunt a/(a+b+c), b/(a+b+c), c/(a+b+c). ntr-adevr, dac r
este raza cercului nscris n triunghiul ABC, avem:
2
1
ar
S = ,
2
2
br
S = ,
2
3
cr
S =
iar
( )
2
r c b a
S
+ +
= . Coordonatele lui I sunt
c b a
a
S
S
+ +
= =
1
1
,
c b a
b
S
S
+ +
= =
2
2
,
c b a
c
+ +
=
3








Bibliografie

V. Nicula, Revista Arhimede, nr 1-2/2002 pag 6-19
M. Botesch, D. Isac, Baricentre i centre de greutate G.M. 7-8/2001 pag
257
M. Ganga, Probleme elementare de matematic, Ed. Mathpress, Ploieti,
2003 pag 196


133
13.4. Probleme rezolvate (13)

R13.4.1. S se arate c ntr-un triunghi medianele sunt concurente ntr-un
punct G care mparte fiecare median n raportul 2:1, de la vrfuri. Punctul G se
numete centrul de greutate al triunghiului.
b) Segmentul determinat de un vrf al unui tetraedru i centrul de greutate al
feei opuse se numete median. Artai c ntr-un tetraedru medianele sunt
concurente ntr-un punct G, care se numete centrul de greutate al tetraedrului.
Soluie:
a) Fie triunghiul ABC i A
1
, B
1
, C
1
, mijloacele laturilor [BC], [AC] i
respectiv[AB] (vezi figura 13.6). Cum A
1
este mijlocul segmentului [BC],
punem n B i C aceeai mas. Un raionament similar ne conduce la aezarea n
A,B i C a aceleiai mase, de exemplu 1. Avem deci sistemul ( ) C B A S 1 , 1 , 1 = .
Exist baricentrul sistemului S (regula 1), notat cu G, deci
3
1 1 1 C B A
G
+ +
= .
Fie A
1
baricentrul sistemului { } C B1 , 1 , deci
2
1 1
1
C B
A
+
= i A
1
B= A
1
C
1
.
Aplicnd regula 3, deducem c S nu-i schimb centrul de greutate G, deci
( )
3
2 1
3
1 1 1
1
A A C B A
G
+
=
+ +
= , de unde G[AA
1
] i 2GA
1
=1*GA. Analog
obinem GB/GB
1
=GC/GC
1
=2/1.
b) Considerm sistemul de puncte materiale { } D C B A S 1 , 1 , 1 , 1 = (vezi figura
13.7). Exist G, centrul de greutate al sistemului S,
4
1 1 1 1 D C B A
G
+ + +
= .
Grupm { } D C B 1 , 1 , 1 i fie G
A
centrul de greutate al sistemului, deci
D C B G
D C B
G
A A
1 1 1 3
3
1 1 1
+ + =
+ +
= .
Din teorema de grupare a punctelor materiale, deducem c sistemul 3G
A
,1A are
acelai centru de greutate G ca i sistemul S.

1A
1B 1C
2B
1
2A
1
2C
1
G
1A
1B 1C
1D
G
2B
1
3G
A

Fig. 13.6 Fig. 13.7
134
Avem deci c
( )
4
1 3
4
1 1 1 1 A G A D C B
G
A
+
=
+ + +
= =, ceea ce nseamn c
G[AG
A
] i 3GG
A
=1G
A
, adic
1
3
=
A
GG
GA
. Deci G mparte mediana tetraedrului
n raportul 3:1 plecnd de la vrf. Avem i
( )
| |
D
D
DG G
D G D C B A
G
+
=
+ + +
=
4
1 3
4
1 1 1 1
i
1
3
=
D
GG
GD
. Analog
obinem c G[CG
C
] i
1
3
=
C
GG
GC
; G[BG
B
] i
1
3
=
A
GG
GB
.
Observaie: Problema are o rezolvare simpl folosind chestiuni elementare.
Scopul alegerii problemei a fost exemplificarea metodei baricentrelor pe o
problem clasic.

R13.4.2. Pe laturile (AB), (BC), (CA) ale triunghiului ABC se consider
punctele M, N respectiv P, astfel nct
5
1
=
MB
MA
,
4
1
=
NB
NC
i
6
1
=
PA
PC
. S se
determine raportul n care punctul de intersecie al segmentelor [AN] i [MP]
mparte fiecare din aceste segmente.
Soluie: Vom pune mase convenabile n vrfurile triunghiului astfel ca punctele
M, N i P s fie baricentre. Din NB=4NC rezult c trebuie s lum punctele 1B
i 4C (vezi figura 13.8). n A punem masa x astfel ca M s fie baricentrul
punctelor materiale 1B, xA.
A
1B
5N 4C
P
G
M

Fig. 13.8

Din relaia 5MA=1MB rezult x=5, deci
6
5 1 A B
M
+
= . De asemenea n A
punem masa y astfel ca P s fie baricentrul punctelor materiale 3C,yA. Din
135
6PC=PA i 4PC=yPA deducem
3
2
6
4 4
= = =
PA
PC
y . Deci P este baricentrul
punctelor 4C, A
3
2
, cnd
3
14
3
2
4 A C
P
+
= . Fie G baricentrul sistemului
)
`

= A A C B S
3
2
, 5 , 4 , 1 .
Deci
2
32
2
3
5 4 1 A A C B
G
+ + +
=
Aplicm teorema de grupare a punctelor materiale i avem:
( )
3
32
3
14
6
3
32
3
2
4 5 1
P M
A C A B
G
+
=
|
.
|

\
|
+ + +
= ,
deci G[MP] i GP GM
3
14
6 = , adic
9
7
6
3
14
= =
GP
GM
.
Folosind alt grupare, avem
( )
3
32
3
17
5
3
32
2
3
5 4 1
A N
A A C B
G
+
=
|
.
|

\
|
+ + +
= , deci G[AN] i GA GN
3
17
5 = , adic
17
15
13
17
5
= =
GN
GA
.

R13.4.3. Teorema lui Pappus.
Pe laturile triunghiului ABC se consider punctele M(AB), N(BC) i
P(AC), astfel ca. Atunci triunghiurile ABC i MNP au acelai centru de
greutate.
Demonstraie:
Vom alege mase convenabile pentru vrfurile triunghiului astfel ca punctele M,
N i P s fie baricentre (vezi figura 13.9).


136
A
B C
M
N
P
G


Fig. 13.9
Din MA=rMB rezult c trebuie s lum punctele 1A i rB, iar
r
rB A
M
+
+
=
1
1
.
Din NB=rNC rezult punctele !B, rC cnd
r
rC B
N
+
+
=
1
1
. Din PC=rPA rezult
punctele materiale 1C, rA, iar
r
rA C
P
+
+
=
1
1
.
Fie G baricentrul sistemului { } rA C rC B rB A S , 1 , , 1 , , 1 = , deci
( ) r
rA C rC B rB A
G
+
+ + + + +
=
1 3
1 1 1
G. Folosind teorema de grupare a punctelor
materiale, avem:
( ) ( ) ( )
( )
=
+
+ + + + +
=
r
rA C rC B rB A
G
1 3
1 1 1

( ) ( ) ( )
( ) 3 1 3
1 1 1 P N M
r
P r N r M r + +
=
+
+ + + + +
= , deci G este centrul de greutate al
triunghiului MNP. Grupnd altfel avem:
( ) ( ) ( )
( )
3
1 3
1 1 1 C B A
r
rC C rB B rA A
G
+ +
=
+
+ + + + +
= , prin urmare G este centru de
greutate i pentru triunghiul ABC.

R13.4.4. Teorema lui Pappus generalizat
Pe laturile triunghiului ABC considerm punctele M(AB), N(BC) i
P(AC), astfel nct
1
2
x
y
MB
MA
= ,
1
2
y
z
NC
NB
= ,
1
2
z
x
PA
PC
= , unde x
1
,x
2
,y
1
,y
2
,z
1
,z
2

sunt numere pozitive oarecare. Atunci baricentru sistemului de puncte materiale
( ) ( ) ( ) { } C z z B y y A x x S
2 1 2 1 2 1 1
, , + + + = este acelai cu baricentrul sistemului
137
( ) ( ) ( ) { } P x z N z y M y x S
2 1 2 1 2 1 2
, , + + + = .
Demonstraie:
Considerm sistemul de puncte materiale
{ } C z C z B y B y A x A x S
2 1 2 1 2 1
, , , , , = . Conform regulii 1, S are baricentrul G, cu
2 1 2 1 2 1
2 1 2 1 2 1
z z y y x x
C z C z B y B y A x A x
G
+ + + + +
+ + + + +
= .
Folosind regula 3 avem:
( ) ( ) ( )
2 1 2 1 2 1
2 1 2 1 2 1
z z y y x x
C z z B y y A x x
G
+ + + + +
+ + + + +
= , deci G este baricentrul sistemului S
1
.
Din x
1
MA= y
2
MB rezult c trebuie considerate punctele materiale x
1
A, y
2
B i
2 1
2 1
y x
B y A x
M
+
+
= . Analog obinem
( )
2 1
2 1
z y
C z B y
N
+
+
= i
1 2
1 2
z x
C z A x
P
+
+
= .
Folosind din nou teorema de grupare a punctelor materiale, avem:
( ) ( ) ( )
=
+ + + + +
+ + + + +
=
1 2 2 1 2 1
1 2 2 1 2 1
z x z y y x
C z A x C z B y B y A x
G
( ) ( ) ( )
2 1 2 1 2 1
1 2 2 1 2 1
z z y y x x
P z x N z y M y x
+ + + + +
+ + + + +
= , prin urmare G este baricentru i pentru
S
2
. n cazul particular x
1
=y
1
=z
1
=u. x
2
=y
2
=z
2
=v, att A,B,C ct i M,N,P au
aceeai pondere u+v, deci baricentrul comun al celor dou sisteme devine
centrul de greutate comun i am obinut teorema lui Pappus.
Observaie:
Ca o consecin imediat avem urmtorul rezultat: Fie G centrul de greutate al
triunghiului ABC. Atunci pentru orice M(AB) exist N(BC) i P(CA)
astfel nct G s fie centrul de greutate al triunghiului MPN.

138
14. Probleme de geometrie combinatorica

14.1. Partiii ale mulimii numerelor naturale

Problemele care se refer la partiii ale mulimii numerelor naturale pot
aprea n contexte aparent diferite.
n general, o astfel de problem poate avea enunul de forma:
Dac partiionm mulimea numerelor naturale ntr-un numr finit de
submulimi, atunci una din mulimi are o proprietate dat. Aceast proprietate
poate fi legat strict de mulimea numerelor naturale (de exemplu, dac ea este
legat de operaiile cu numere naturale), legat doar de faptul c mulimea
numerelor naturale este numrabil (ideea de ir) sau de faptul c este o
mulime discret i le putem parcurge pe toate pas cu pas.
Una din problemele celebre, cu aplicaii foarte interesante, este
problema lui Baudet, rezolvat de Van der Waerden. Demonstraia teoremelor
legate de aceast problem ar ocupa mult spaiu (dar, ar merita urmrit din
bibliografie), noi vom da doar enunurile i cteva aplicaii.
14.1.1. Teorem. Pentru orice partiie a lui n dou clase
1 2
A A = U ,
1 2
( ) A A = I i pentru orice 3 k exist o clas care conine k
numere n progresie aritmetic.
14.1.2. Observaii. a) Orice partiie a lui n dou clase, are o clas
care conine progresii aritmetice finite orict de lungi.
b) Existena progresiilor cu orict de muli termeni nu implic existena
n una din clase a unei progresii aritmetice infinite dup cum rezult din
exemplul
1 2
A A = U unde
1
2
:1, , 4, 5, 6, ,
: 2, 3 7, 8, 9,10,
A
A
K
K

c) n general, n teoria numerelor, considerm c 0 nu este numr
natural, deci {1, 2, 3, } = K .
14.1.3. Teorem. Pentru orice partiie a mulimii ntr-un numr
finit de mulimi
1 n
A A = UKU i pentru orice 3 k esxist k numere n
progresie aritmetic, toate aflate n aceeai submulime a partiiei.
14.1.4. Teorem. (Van der Waerden). Pentru orice numr natural n
i 3 k exist un numr natural (minim) ( , ) N W n k = cu proprietatea c
pentru orice partiie a mulimii {1, 2, , } N K n n mulimi, cel puin o mulime
conine k numere n progresie aritmetic.
14.1.5. Teorem. (Kakeya, Morimoto). Pentru orice numere naturale
k i d exist un numr natural N cu proprietatea c orice mulime de N numere
naturale
139
1 2 N
a a a < < < K
cu
1
, 1, 1
i i
a a d i N
+
= , conine o progresie aritmetic cu k termeni.
14.1.6. Observaie. Nu se poate trage concluzia c dac irul infinit de
numere naturale
1 2 n
a a a < < < < K K verific condiia
*
1
,
i i
a a d i
+
,
atunci el ar conine progresie aritmetic infinit. (Mulimea progresiilor infinite
este numrabil, o progresie fiind determinat de primul termen i de raie, iar
mulimea este numrabil).
Dac presupunem c am ordonat toate progresiile:
1 2
, , , ,
n
P P P K K i
alegem
1 1 2 2
, , , ,
n n
b P b P b P K K atunci irul
{ }
\
n
b n verific
condiia
1
2
i i
a a
+
, dar nu conine nici o progresie aritmetic infinit.
O interpretare interesant a teoremei lui Van der Waerden a fost dat de
R.Rado [2] i anume: oricum am mpri numerele naturale ntr-un numr finit
de clase, sistemul de ecuaii
1 2 2 3 1
0
n n
x x x x x x
+
= = = K
are soluie n una din clase.
Aceast interpretare a deschis un tip de probleme:
O ecuaia de forma
1 2
( , , , ) 0
n
F x x x = K sau un sistem de astfel de ecuaii
are soluie n una din mulimile partiiei, oricare ar fi aceast partiie?
14.1.7. Definiie. Dac o ecuaie sau un sistem de ecuaii are
proprietatea c oricum am mpri mulimea numerelor naturale ntr-un numr
finit de clase, ecuaia sau sistemul are soluie cu toate componentele n aceea
clas, spunem c ecuaia sau sistemul este n normal.
14.1.8. Observaii. a) Dac o ecuaie este n normal i k n < atunci ea
este k normal.
b) O ecuaie care este n normal pentru orice
*
n se numete ecuaie
normal.
c) Din teorema lui Van der Waerden rezult c ecuaia 2 x y z + = este
normal (chiar n ipoteza suplimentar c numerele x, y i z sunt distincte).
Un rezultat general, interesant, n acest domeniu este
14.1.9. Teorem. Ecuaia ax by cz + = este 2-normal pentru orice
*
, , a b c (n general ns ea nu este n normal pentru orice n).




140
Bibliografie

1. G., Sudan, Cteva probleme matematice interesante, Editura Tehnic,
Bucureti, 1969
2. P., Erds, R., Rado, Combinatorial theorem on classification of subsets of
a given set, Proc.of London Math.Soc., vol.2, 1952
3. L.,Moser, Notes on number theory. On a theorem of Van der Waerden,
Canadian Bull. 1960
4. I., Creang, Introducere n teoria numerelor, Editura p. Bucureti, 1965









141
14.2. Probleme rezolvate (14)

R14.2.1. S se arate c pentru orice partiie a punctelor planului ntr-un numr
finit de mulimi, exist trei puncte n una din mulimi, care sunt vrfurile unui
triunghi echilateral.
Soluie. Pentru partiia n dou mulimi vom da o soluie elementar:


















Considerm dou puncte A
1
, A
2
din aceeai mulime C
1
. Cele dou
puncte A
3
i A
4
care sunt vrfuri ale triunghiurilor
1 2 3
A A A i
1 2 4
A A A sunt n C
2

(n caz contrar se formeaz triunghi echilateral n C
1
). Punctul A
5
situat pe
semidreapta
1 2
[ A A pentru care triunghiul
3 4 5
A A A este echilateral va fi n C
1
.
Punctul A
6
pentru care triunghiul
2 5 6
A A A este echilateral (situat n aceeai parte
a dreptei A
1
A
5
ca i A
3
) va fi n C
2
. Analizm n ce mulime este punctul A
7

(pentru care triunghiurile
3 6 7
A A A i
1 5 7
A A A sunt simultan echilaterale). Dac
7 1
A C avem triunghiul
1 5 7
A A A n C
1
, iar dac este n C
2
, atunci
3 6 7
A A A este
triunghi echilateral n C
2
. Pentru partiia n n mulimi, aceast metod nu pare a
avea anse. Vom folosi teorema Van der Waerden.
Considerm n plan o reea de triunghiuri echilaterale (de latur 1).
Considerm nodurile reelei situate pe una din dreptele d
1
ale reelei.
Considerm nodurile reelei situate pe una din dreptele d
1
ale reelei.
Exist pe ea suficient de multe (orice numr finit) puncte n aceeai mulime a
partiiei C
1
, echidistante (n progresie aritmetic). Dac L
1
este distana ntre

A
5

A
6

A
7
(1,2)
A
2
A
3
A
4

A
1
142
dou astfel de noduri consecutive, considerm toate triunghiurile echilaterale de
laturi L
1
situate n acelai semiplan fa de d
1
, cu cte dou vrfuri pe d
1
. Dac
unul din aceste noduri este n C
1
se formeaz triunghi n C
1
. Dac toate sunt n
celelalte ( 1) n mulimi, se pare c putem continua inductiv, dac la nceput am
luat suficient de multe puncte pe d
1
. Acest numr este
n
N a = unde irul ( )
k k
a
este definit astfel:
1
( 1, 1) , 1,
k k
a W a k k n
+
= + + = i
1
2 a = ,
unde am notat ( , ) W p q numrul minim pentru care dac partiionm mulimea
{1, 2, ( , )} W p q K n q mulimi exist p numere n progresie aritmetic n aceeai
mulime.
R14.2.2. Fie 1 x > , 1 y > numere reale. S se arate c mulimile
{ }
*
[ ] A n x n = ,
{ }
*
[ ] B n y n = formeaz o partiie a mulimii
*
,
dac i numai dac , \ x y i
1 1
1
x y
+ = .
Soluie. Fie
{ }
*
[ ]
n
A k k x n = ,
{ }
*
[ ]
n
B k k y n = . Dac
max
n n
k A = , max
n n
l B = atunci ,
n n n n
A k B l = = i condiia
*
1
( )
n n
n
A B

=
= U U revine la: pentru orice 0 > , orict de mic, exist
*
N
astfel ca
1 1
n n
l k
n

+
< < pentru toi n N .
Avem: [ ] 1 k x n k x n < + deci
1 n
k
x
+
< i atunci
1 1
1
n
n n
k
x x
+ +
< i analog,
1 1
1
n
n n
l
y y
+ +
< .
Adunnd relaiile obinem:
1 1 1 2 1 1 1
n n
k l n n
n x y n n n x y
| | | | + + +
+ < +
| |
\ . \ .
,
relaii care au loc pentru orice n numai dac
1 1
1
x y
+ = .
Rmne s artm c , \ x y .
143
Dac, prin absurd, x atunci i y , fie , x y
q p q

= =

. Avem:
[ ] [( ) ] q x p q y = = deci A B I .
Reciproc, dac , \ x y i
1 1
1
x y
+ = trebuie s artm c orice
numr natural
*
n se afl doar n una din mulimile A sau B. Fie
n
A k k
x

=
`
)
,
n
B l l
y

=
`
)
i
0 0
min , min k A l B = = . Dac
0 1
n
k
x
= + ,
0 2
n
l
y
= + , cu
1 2
, (0,1) , atunci
0 0 1 2 1 2
1 1
k l n n
x y

| |
+ = + + + = + +
|
\ .
, deci
1 2
1 + = sau
1 2
1
x y
x y

+ = . Unul din numerele
1 2
, x y este mai mic ca 1, iar altul mai
mare (dac am avea
1 2
1 x y = = atunci
0
1 x k n = + sau x ). Dac
1
0 1 x < < i
2
1 y > atunci
0 1
x k n x = + i
0
[ ] x k n = , iar
0
[ ] y l n > .
R14.2.3. a) S se arate c oricum am partiiona mulimea numerelor naturale
n dou mulimi, exist numerele x, y, z n una din mulimile partiiei astfel ca
ele s verifice relaia
3 x y z + = .
b) Rmne afirmaia adevrat dac partiia se face n mai multe
mulimi?
Soluie. Observm c n
*
ecuaia 3 x y z + = are soluia (2,1,1) i
(2 , , ) k k k ,
*
k .
Notm cu C
1
mulimea care conine pe
1
3 a = i cu C
2
cealalt mulime.
Dac n irul
{ }
3 , 3 n n n toi termenii ar fi din C
1
am avea soluia
1
(18, 9, 9) ( ) S C . Fie deci , 3 n n astfel nct
2 2
3 a n C = i notm
3( 1) k n = . n irul
{ }
*
m k m exist termeni din C
2
(n caz contrar am
avea soluia
1
(2 , , ) ( ) k k k S C , fie
3 2
a m k C = i considerm numerele
4
( 1) a m n n = + i
5
( 1) a m k = + . Tripletele
3 2 4
( , , ) a a a i
5 1 4
( , , ) a a a verific
ecuaia 3 x y z + = i din
2 3 2
, a a C rezult
4 1
a C iar din
1 4 1
, a a C rezult
5 2
a C . Atunci, din ipoteza
2
m k C a rezultat
2
( 1) m k C + i, prin inducie,
2 2
( 2) , , 2 m k C mk C + K i am obinut soluia
2
(2 , , ) ( ) mk mk mk S C .
144
b) Vom arta c dac, de exemplu, partiionm n 4 clase, ecuaia
3 x y z + = nu are soluie n nici una din clase.
Pentru aceasta punem n eviden o astfel de partiie:
Se tie c orice numr natural se poate scrie sub forma 5
p
N q = , unde
q nu se divide cu 5. Definim clasele:

{ }
1
(5 1) 5 ,
m
C n n m = + ;

{ }
2
(5 2) 5 ,
m
C n n m = + ;

{ }
3
(5 3) 5 ,
m
C n n m = + ;

{ }
4
(5 4) 5 ,
m
C n n m = + .
Dac ecuaia ar avea soluie n una din mulimi am avea:
3 1 2
1 2 3
(5 ) 5 (5 ) 5 3(5 ) 5
m m m
n c n c n c + + + = + .
Distingem cazurile:
a) Unul din
1 2 3
, , m m m este strict mai mic dect celelalte dou;
b) Unul din
1 2 3
, , m m m este strict mai mare dect celelalte dou;
c)
1 2 3
m m m = = .
Toate, dup simplificare cu puterea maxim a lui 5, dau contradiciile de forma
0(mod5) C sau 2 0(mod5) C , sau 3 0(mod5) C .
Observaie. Ecuaia 3 x y z + = este 2normal dar nu este 4normal
(nici nnormal pentru 4 n ).
R14.2.4. Se consider n plan
1 1 1
1 ! 1 , 1
1! 2! !
n
a n n
n
| |
= + + + + +
|
\ .
K puncte
oricare trei necoliniare, iar segmentele ce unesc aceste puncte se partiioneaz
n n mulimi (se coloreaz n n culori). S se arate c se formeaz cel puin un
triunghi cu laturile n aceeai mulime a partiiei (monocolor).
Soluie. Vom ncerca o inducie dup n.

1
1; 3 n a = = evident;

2
2; 6 n a = =
3
3; 17 n a = =
(Problem dat la Olimpiada Internaional de Matematic, 1964
(17 oameni corespondeaz ntre ei pe trei teme. S se arate c exist 3
care corespondeaz pe aceeai tem).
Pentru inducie, n general, este util o relaie de recuren de tipul
1
( )
n n
a f a

= . Avem
145

1 1
1 ! 1
1! ( 1)!
n
a n
n
| |
= + + +
|

\ .
K

1 1
1 ( 1)! 1 1
1! ( 1)!
n
a n n
n
( | |
= + + + +
( |

\ .
K

1
1 ( 1) 1
n n
a n a

= +
Unim punctul A
1
cu celelalte 1
n
a puncte cu segmente de n culori,
1
1 ( 1) 1
n n
a n a

= + cel puin
1 n
a

segmente sunt de aceeai culoare
1
1 1
( 1) 1
n
n
n a
C a
n

+ | |
>
|
\ .
.
Dac unul din segmente ce unesc aceste
1 n
a

puncte ar fi de culoare C
1

am avea un triunghi de culoare C
1
.
Dac nu, problema s-a redus la pasul de inducie (
1 n
a

puncte cu
segmentele ce le unesc partiionate n ( 1) n mulimi).
Alt formulare: A
n
mulime cu a
n
elemente i
2
( )
n
A P mulimea prilor
cu dou elemente. Dac partiionm
2
( )
n
A P n n mulimi exist o mulime C
i
a
partiiei i , ,
n
A distincte astfel ca { , },{ , },{ , }
i
C
(Pentru idee n se poate lua
2
2, 6 n a = = )
R14.2.5. Numerele naturale de la 1 la
1 1 1
! 1
1! 2! !
n
b n
n
| |
= + + + +
|
\ .
K se
partiioneaz n n mulimi. S se arate c ecuaia
x y z + =
are soluie n cel puin una dintre mulimi.
(exist : , ,
i i
C x y z C neaprat distincte astfel ca x y z + = ).
Soluie. Vom proceda n mod analog cu problema anterioar.
1
2, 1 1 2 b = + = deci (1,1,2) este soluie.
Relaia de recuren este
1
1
n n
b n b

= + .
Din relaie suntem inspirai la o prim concluzie: cel puin
1
1
n
N b

= +
numere sunt n aceeai mulime C
1
a partiiei. Fie ele
1 2
1
N
< < < K
i s considerm diferenele
2 1 3 2 1
1
N
< < < K
n numr de
1 n
b

.
Dac una din aceste diferene ar fi n C
1
problema este rezolvat.
146
Dac nu, cum
1 2
( 1) 1
n n
b n b

= + cel puin
2
1
n
b

+ sunt n C
2
.
Fie ele
2
1 2 1
1
n
b


+
< < < K i considerm cele
2 n
b

diferene
2
2 1 3 2 1 1
n
b

+
< < < K .
Dac nu ar fi n C
2
s-a terminat (soluie n C
2
)
Dac nu ar fi n C
1
, din nou s-a terminat cci
1
( ) ( )
i j k k
= = (soluie n C
1
).
Dup n pai ajungem la
1
2 b = elemente (diferene) din care dac unul
este n mulimile
1 1
, ,
n
C C

K s-a terminat. Dac ambele sunt n C
n
oriunde ar fi
diferena lor avem o soluie n aceast mulime.
Avem:
2 3 4 5 6
5, 16, 65, 326, 1957 1978 b b b b b = = = = = < .
Pentru 6 n = Olimpiada Internaional de Matematic, Bucureti 1978
s-a dat problema: Membrii unei Societi Internaionale fac parte din 6 ri.
Lista membrilor conine 1978 de nume, numerotate de la 1 la 1978. S se arate
c exist un membru care are numrul de ordine egal cu suma numerelor de
ordine a doi compatrioi (sau dublul unuia).
Observaii. a) n limbajul partiiilor lui , problema afirm c ecuaia
x y z + = este n-normal pentru orice
*
n , deci este o ecuaie normal.
Metoda de rezolvare a fost dat de I. Schur.
b) ntre problemele R.13.2.4 i R.13.2.5 din relaia 1
n n
a b = + se
ntrevede o legtur. Soluia problemei R.13.2.5 s-ar putea obine din cea a
problemei 13.2.4.
Dac
1 2
{1, 2, , }
n n
b M M M = K U UKU considerm n plan punctele
0 1 2
, , , ,
n
b
A A A A K (a
n
puncte) oricare trei necoliniare i colorm segmentul
[ ]
i j
A A cu culoarea C
k
corespunztoare mulimii M
k
n care se afl numrul
{1, 2, , }
n
i j b K .
Faptul c un triunghi
i j k
A A A este monocolor este echivalent cu faptul c
numerele , x j i y k j = = i z k i = sunt n aceeai mulime i atunci ele
verific ecuaia x y z + = .
R14.2.6. S se arate c oricum am partiiona planul n n mulimi, exist patru
puncte n aceeai mulime care sunt vrfurile unui dreptunghi.
Soluie. Considerm reeaua laticeal. Pe dreapta 0 y = avem o
infinitate de noduri, dintre care o infinitate n aceeai mulime a partiiei C
1
.
Considerm nodurile de deasupra lor situate pe dreapta 1 y = , dac dou din
ele ar fi n C
1
s-a format dreptunghi n C
1
, dac nu rmnem cu o infinitate de
puncte de pe dreapta 1 y = , aflat n C
2
, deasupra unor puncte de pe dreapta
147
0 y = , din C
1
. Considerm nodurile de deasupra acestora pe dreapta 3 y = , o
infinitate sunt n C
3
. Raionnd inductiv ajungem pe dreapta y n = cu o
infinitate de puncte n una din mulimile
1 2
, , ,
n
C C C K . Dac acesta este C
k
se
formeaz dreptunghi cu vrfurile n C
k
(dou noduri pe dreapta y n = i dou
noduri pe dreapta 1 y k = ).
R14.2.7. Vrfurile unui poligon regulat sunt colorate 2n n rou i 2n n negru.
S se arate c exist dou poligoane cu ( 1) n + vrfuri, unul nergru i unul rou,
congruente.
Soluie. Ne imaginm c am construit dou roi identice n care pe
cercurile de pe margine am fcut 4n guri. n gurile corespunztoare punctelor
roii avem cte o surs de cldur iar n cele negre avem cartue. Putem roti
roata de sus peste cea de jos, care rmne fix. Dac se suprapune o gaur
negar peste una roie atunci puc un cartu. Este suficient s artm c exist
o rotaie la care se produc ( 1) n + pucturi.
Avem 4 1 n rotaii i fiecare punct negru ajunge peste fiecare punct
roi, n total avem
2
2 2 4 n n n = pucturi. Deoarece
2
4
4 1
n
n
n
>

, exist o rotaie
la care se produc ( 1) n + pucturi.

Potrebbero piacerti anche